Download as pdf or txt
Download as pdf or txt
You are on page 1of 202

THERMODYNAMICS

For
MECHANICAL ENGINEERING
THERMODYNAMICS
SYLLABUS
Thermodynamic systems and processes; properties of pure substances, behaviour of
ideal and real gases; zeroth and first laws of thermodynamics, calculation of work and
heat in various processes; second law of thermodynamics; thermodynamic property
charts and tables, availability and irreversibility; thermodynamic relations.
Power Engineering: Air and gas compressors; vapour and gas power cycles, concepts of
regeneration and reheat.
I.C. Engines: Air-standard Otto, Diesel and dual cycles.
Refrigeration and air-conditioning: Vapour and gas refrigeration and heat pump cycles;
properties of moist air, psychrometric chart, basic psychrometric processes.

ANALYSIS OF GATE PAPERS


Exam Year 1 Mark Ques. 2 Mark Ques. Total
2003 5 6 17
2004 3 6 15
2005 2 7 16
2006 - 7 14
2007 2 6 14
2008 2 9 20
2009 3 4 11
2010 2 4 10
2011 2 7 16
2012 1 2 5
2013 1 4 9
2014 Set-1 2 4 10
2014 Set-2 2 2 6
2014 Set-3 1 3 7
2014 Set-4 - 4 8
2015 Set-1 4 3 10
2015 Set-2 3 4 11
2015 Set-3 2 3 8
2016 Set-1 3 3 9
2016 Set-2 3 2 7
2016 Set-3 1 2 5
2017 Set-1 1 4 9
2017 Set-2 2 4 10
2018 Set-1 2 3 8
2018 Set-2 2 4 10

© Copyright Reserved by Gateflix.in No part of this material should be copied or reproduced without permission
CONTENTS
Topics Page No
1. BASIC CONCEPTS OF THERMODYNAMICS

1.1 Thermodynamic 01
1.2 Microscopic Approach and Macroscopic Approach 01
1.3 System 01
1.4 Property of System 02
1.5 State 03
1.6 Thermodynamic Equilibrium 04
1.7 Zeroth Law of Thermodynamics 04
1.8 Ideal gas Equation and Process 06
1.9 Work Transfer 08
1.10 Heat Transfer 10
1.11 Example 11

2. FIRST LAW OF THERMODYNAMICS

2.1 First Law of Thermodynamics for a Cycle 13


2.2 Application of First Law of Thermodynamics 13
2.3 Meyer’s Formula 14
2.4 Heat Transfer in Different Process 15
2.5 Free Expansion 15
2.6 First Law of Thermodynamics for Flow Process 16
2.7 First Law of Thermodynamics for Variable Flow Process 17
2.8 Examples 18

3. SECOND LAW OF THERMODYNAMICS - ENTROPY

3.1 Thermal Reservoir 23


3.2 Equivalence of Kelvin Plank and Clausius Statement 25
3.3 Carnot Cycle 25
3.4 Refrigeration & Heat Pump working on Reversed Carnot Cycle 26
3.5 Clausius Theorem 26
3.6 Entropy 28
3.7 Combined First Law and Second Law of Thermodynamics 28
3.8 Entropy change for an ideal gas 29
3.9 Entropy change for finite body 29
3.10 Slope of isobaric process & isochoric process on T-S Diagram 29
3.11 Example 29

4. AVAILABLE ENERGY & THERMODYNAMIC RELATIONS

4.1 Available Energy 34

© Copyright Reserved by Gateflix.in No part of this material should be copied or reproduced without permission
4.2 Unavailable Energy 35
4.3 Loss in Available Energy 35
4.4 Available Function 35
4.5 Irreversibility 36
4.6 Exact Differential Equations 36
4.7 Maxwell’s Equations 36
4.8 T – DS Equations 36
4.9 Energy Equation 38
4.10 Enthalpy Equation 38
4.11 Difference in Heat Capacities 38
4.12 Joule Kelvin Coefficient 39
4.13 Examples 39

5. PROPERTIES OF PURE SUBSTANCE & GAS MIXTURE

5.1 Pure substance 44


5.2 P – V diagram of Pure substance 44
5.3 Triple Point 45
5.4 Gibbs phase rule 45
5.5 Phase Change of Pure substance 45
5.6 T – S diagram of pure substance 46
5.7 Properties of Pure Substance 47
5.8 Specific volume, Enthalpy and Entropy of different Phases 48
5.9 Equation of State 49
5.10 Properties of Gas mixture 51
5.11 Examples 51

6. REFRIGERATION

6.1 Introduction 56
6.2 Air Standard Refrigeration Cycle 56
6.3 Refrigerator working on reversed brayton Cycle 58
6.4 Vapour Compression Refrigeration Cycle 60
6.5 Effect of Parameters on COP of Vapour Compression
Refrigeration cycle 62
6.6 Vapour Absorption Refrigeration Systems 63
6.7 Refrigerant 65
6.8 Designation of refrigerants 66
6.9 Examples 66

7. PSYCHOMETRY

7.1 Properties of Moist Air 72


7.2 Psychometric Chart 73
7.3 Significance, Different Lines on Psychometric Chart 74
7.4 Different Process on Psychometric Chart 78
7.5 Examples 79

© Copyright Reserved by Gateflix.in No part of this material should be copied or reproduced without permission
8. POWER PLANT ENGINEERING

8.1 Stirling cycle 84


8.2 Ericson cycle 84
8.3 Gas Power Plant 84
8.4 Steam Power Plant 88
8.5 Nozzle and Diffuser 90
8.6 Compressor 92
8.7 Steam turbine 93
8.8 Examples 93

9. INTERNAL COMBUSTION ENGINE

9.1 Heat Engine 101


9.2 Applications of IC Engines 101
9.3 Classifications of IC Engines 101
9.4 Basic components of IC Engine 101
9.5 Terms used in internal combustion engine 102
9.6 Difference between four stroke and two stroke Engine 103
9.7 Performance Parameters 103
9.8 Air standard cycle and efficiency 103
9.9 Comparison of Otto, Diesel, dual Cycle 106
9.10 Examples 107

10. GATE QUESTIONS 112

© Copyright Reserved by Gateflix.in No part of this material should be copied or reproduced without permission
1 BASIC CONCEPTS OF THERMODYNAMICS

1.1 THERMODYNAMICS 1.3 SYSTEM

Thermodynamics is the science of energy A quantity of matter or region in space


transfer and its effect on the physical upon which attention is concentrated is
properties of the substances. known as thermodynamic System. The
Some students have difficulties with system and the boundary are specified by
thermodynamics because of global nature the analyst, these are not specified in a
of its applicability. Most students are used problem statement.
to courses that focus on a few specific
topics like statics, dynamics, fluid flows etc. 1.3.1 SURROUNDING
all deals with the limited range of topics.
Thermodynamics, on the other hands, deals Anything external to the system is called as
with many issues that are inherent in every surrounding or Environment. The
engineering system. A thermodynamic combination of the system and surrounding
analysis can span from analyzing a makes universe. It means in the universe if
complete power plant to analyzing the anything is specified by the analyst as a
smallest component in power plant. system the things except that system are
We begin by introducing some basic considered as the surrounding.
thermodynamic terms and definitions.
Some of these terms are already in our
everyday vocabulary as a result of the
broad result of thermodynamics concepts
in non engineering concepts (for example
cooling process of tea in the open
environment is a thermodynamic process).

1.2 MICROSCOPIC APPROACH AND Fig.1.1system, surrounding and boundary


MACROSCOPIC APPROACH
1.3.2 BOUNDARY
In microscopic approach, a certain
quantity of matter is considered with The separation line which separates the
considering the event occurring at system from surrounding is called as
molecular level. It is called as statistical Boundary.
Thermodynamics. In Macroscopic  Boundary may be fixed or rigid, may be
approach, a certain quantity of matter is real or imaginary.
considered without considering the event
occurring at molecular level. In this, 1.3.3 UNIVERSE
average behaviour of molecules is
considered. It is called as Dynamic The combination of system and surrounding
Thermodynamic. is called as Universe.
At the higher altitude or where the density Universe = system + Surrounding
of the system is low, the microscopic
approach is used for checking the 1.3.4 TYPE OF SYSTEM
behaviour of the system.
There are three types of thermodynamic
systems.

© Copyright Reserved by Gateflix.in No part of this material should be copied or reproduced without permission
1) Closed System: fluid id transmitted from the outlet valve
(for example the process of compression in
The system in which only energy can compressor),the mass with energy leaves
transfer without transferring the mass from the system it means in this system
across the system boundary is called as mass and energy both crosses the
closed system. An example of closed system boundary so this is considered as the Open
is mass of gas in the piston cylinder System.
without valve.
Let us consider the piston cylinder 3) Isolated System:
arrangement as shown in fig 1.2. When
heat is supplied to system, gas inside the The system in which, neither energy nor
cylinder expands and thermal energy is mass can transfer across the system
converted into mechanical work. But mass boundary is called as isolated system.
of the gas will be the same after the process Example: Universe, Insulated thermal flask.
in cylinder it means there is no mass which Let consider a proper insulated thermal
crosses the boundary of the system so it is flask and hot water is filled in the thermal
considered as closed system. flask after passing of the time mass and
temperature are same which shows neither
energy nor mass crosses the thermal
boundary of the flask, so it is considered as
an isolated system.

fig.:1.2 closed system

2) Open System:
The system in which energy and the mass Fig. 1.4 isolated system
can transfer across the system boundary is
called as open system. Most of engineering 1.4 PROPERTY OF SYSTEM
devices are open systems. Examples:
Turbine, Pump, Boiler, condenser Etc. A property of the system is a characteristic
of system that depends upon the state of
the system .as long as state is fixed,
property of system is fixed. It does not
depend how the state is reached. So
properties depend upon end points only.
These are point functions and exact
differential.
Examples: Pressure, temperature, entropy etc.
Fig. 1.3 open system There are two types of properties:
Let us consider the piston cylinder
arrangement with inlet and outlet valve. (1) Extensive property:
When mass of a fluid enters in control These properties depend upon the mass of
volume of system, the mass and energy of the system. If the system mass is changed
the system change and then this mass of then these types of properties will change.

© Copyright Reserved by Gateflix.in No part of this material should be copied or reproduced without permission
For example, volume, internal energy,  Because in a cycle initial and the final
enthalpy, entropy etc. point are same. So for the
thermodynamic cycle, a thermodynamic
2) Intensive Property: property remains same.
These properties do not depend upon the  Difference of properties for reversible
mass of the system. For example, specific and irreversible process is same.
volume, specific internal energy, specific
enthalpy, specific entropy, pressure,
Temperature etc.
Most of the extensive properties can be
converted in to intensive properties by
dividing the extensive property by system
mass or number of moles in the system.
Intensive properties created in this way are
called as specific property. It means all the
specific properties are intensive property.
1.5.1 PROCESS
Let us we an example: we know that
volume of the system is considered as the A process is said to be done by the system
extensive property and its unit is m3. when it change its state. Process may be
V
But Specific volume (v) = m . The specific non flow process in mass does not transfer
volume is given by m3/kg. It is given as or it may be flow process in which mass
intensive property. and energy both will transfer. Type of
Process: A Process may be reversible
1.5 STATE process or irreversible process.

It is the condition of a system at the instant 1.5.1.1 REVERSIBLE PROCESS


of time. Each property has its single value
The process is said to be reversible process
at every state. As long state is fixed the
if when it is reversed in direction it follows
property at that state is fixed. It means that
the same path as former path without
the property does not depend the
leaving any effect on system and
parameter other than the state or the
surrounding. Reversible process is most
specific point.
efficient process.
Let us consider a state 1 on P- V diagram as
shown in fig 1.5. the properties at point 1
1.5.1.2 IRREVERSIBLE PROCESS
are P1 , V1 . The state change from 1 to 2 and
the properties at state 2 are P2, V2. then we A process which does not follow the
follow the same points but change the path condition of the reversible process is called
between the state 1 and state 2 .but the as the irreversible process. The friction is
properties remains same. It means main reason for the process being the
properties do not depends upon the path. it irreversible process.
only depends upon the point, So result of
this discussion is 1.5.1.3 QUASISTATIC PROCESS
 All properties are the point function.
 Differentiation of all the properties is Quasi means ‘almost’ static means
exact differential (dT, dP etc.) ‘slowness’. When process is carried out in
 Properties do not depend upon the past very slow manner, it is called as Quasistatic
history. These only depends the present process. Infinite slowness is characteristics
condition of the system of a Quasistatic process. Friction less

© Copyright Reserved by Gateflix.in No part of this material should be copied or reproduced without permission
quasistatic process is considered as A system is said to be in thermodynamic
reversible process. equilibrium when it follows:
i) Mechanical Equilibrium
ii) Thermal Equilibrium
iii)Chemical Equilibrium
i) Mechanical Equilibrium:
If there is no unbalanced force within the
system and also between the system and
surrounding, the system is said to be in
mechanical equilibrium.
ii) Thermal Equilibrium:
Fig.1.6 quasistatic process if temperature of the system is same in all
the parts of system, the system is said to be
Let us consider the expansion of a gas from in Thermal equilibrium.
initial state 1 to final state 2 as shown in fig
1.6. In first case the weight W is release iii) Chemical Equilibrium:
from the system and the gas expands
quickly from state 1 to 2. But when weight if there is chemical reaction or transfer of
W is divided into number of parts and then matter from one part of system to another,
allow to release from the gas in second the system is said to be in Chemical
case. the gas passes through the number of equilibrium.
equilibrium state and then reach the final
state 2. So second process is very slow 1.7 ZEROTH LAW OF THERMODYNAMICS
process it is known as quasistatic process.
The P-V diagram for quasistatic process is It states that when body A is in thermal
shown in fig 1.7. equilibrium with body B is in thermal
equilibrium with body C , then body A,B
and C will be in thermal equilibrium. It is
the basic of temperature measurement.

TA TC

Fig. 1.7 TB
1.5.2 CYCLE fig. 1.8 Zeroth law of thermodynamics

A thermodynamic cycle is defined as a If TA =TB & TB = TC, then TA = TB = TC.


series of state changes such that final state
is identical with initial state. for a cycle 1.7.1 APPLICATION OF ZEROTH LAW
difference of all the properties is zero.
The main application of Zeroth law of
1.6 THERMODYNAMIC EQUILIBRIUM thermodynamics is to measure the
A system is said to be in thermodynamic temperature. First thermometric property
equilibrium when there is no change in the is measure which change with respect to
macroscopic property of the system. change of temperature.

© Copyright Reserved by Gateflix.in No part of this material should be copied or reproduced without permission
Let us consider Mercury in glass temperature resistance change .so
thermometer (generally which is used to thermometric Property in resistance
measure the temperature of human body) thermometer is resistance. So to find the
as shown in fig 1.9. temperature of any state first resistance of
The length of the Hg change with respected that state is found and then temperature is
to temperature. So the thermometric measured.
property in this type of thermometer is
length.

Fig. 1.9 Hg in glass thermometer


Fig. 1.10 Resistance Thermometer
It is more important to find the change of
length so that we can calculate the (b) Thermocouple
temperature. So first length of the Hg which
is change is found then temperature is This works on See beck Effect. When two
calculated. In next topic we explain the dissimilar metal at different temperature is
process of measurement of the joint with each other, an electro magnetive
temperature. force is generated the induced e.m.f.
depends upon the temperature difference
1.7.2 TEMPERATURE MEASUREMENT of two ends of the dissimilar metal. So
thermometric property in this
In the thermometer, Zeroth law of thermocouple is induced e.m.f.
thermometer is used to measure the
temperature of the body. Thermometric
Property: The property which changes with
change in temperature is called as
thermometric property.

1.7.2.1 THERMOMETRIC PRINCIPLE


The thermometric property which change
with the change of temperature is found Fig. 1.11 thermocouple
first and with help of this thermometric c) Constant Volume gas thermometer
property temperature of that specified When the volume in constant volume gas
state is calculated. thermometer is constant, the pressure
1.7.2.2 TYPE OF THERMOMETER changes with respect to temperature. In
this type of thermometers, pressure is
There are five types of thermometers. considered as the thermometric property.
a) Resistance thermometer d) Constant pressure gas thermometer

It works on Wheatstone bridge circuit. As When the pressure in constant Pressure


temperature hang with respect to change of gas thermometer is constant, the volume

© Copyright Reserved by Gateflix.in No part of this material should be copied or reproduced without permission
changes with respect to temperature. in Ti = a.Xi --------- (1)
this type of thermometers, volume is and T = a.X ---------(2)
considered as the thermometric property. Then,
X
e) Mercury –in- glass thermometer T  372.15 ----------- (3)
Xi
In this type of thermometers, length change
with change of the temperature so in
mercury in glass thermometer, length is 1.7.3.3 TEMPERATURE SCALES
considered as the thermometric property.
There are following temperature scales
1.7.3 METHOD FOR TEMPERATURE such as Rankin, Celsius, Kelvin and
MEASUREMENT Fahrenheit. There are two common
absolute temperature scales Rankin (R),
1.7.3.1 METHOD FOR TEMPERATURE and Kelvin scale (K). They are related as
MEASUREMENT: BEFORE 1954 follows:
9
Before 1954 temperature measurement T(R)  T(K)
method, the measurement of temperature 5
is based on two reference point freezing
point and boiling point. Let temperature at
freezing point is T1 and thermometric
property at that temperature is X1 and
Temperature at boiling point is T2 and
thermometric property at that temperature
is X2. Let thermometric Property at
temperature T is X .then,
T1 X1
 ------------- (1)
T X
and Temperature at boiling point is T2 and
thermometric property at that temperature
is X2. Let thermometric Property at
temperature T is X.then,
T2 X 2
 ------------- (2)
T X
From equation (1) and (2) Fig. 1.12 different temperature scale
T1  T2 X1  X 2

T X The relationship between the Celsius scale
T T and Fahrenheit scale can be given as
T  1 2 .X --------- (3)
X1  X 2 9
T( 0 F)  T( 0 C)  32 --------- (2)
5
1.7.3.2 METHOD FOR TEMPERATURE The relationship between the Celsius scale
MEASUREMENT: AFTER 1954 and Kelvin scale can be given as
The measurement of temperature is based T(K)  T( 0 C)  273.15 ------------- (3)
on single reference point triple point of
water. Let temperature at triple point is Ti 1.8 IDEAL GAS EQUATION AND PROCESS
and thermometric property at that
temperature is Xi. Let thermometric The ideal gas law is the equation of
Property at temperature T is X. then state of a hypothetical ideal gas. It is a good

© Copyright Reserved by Gateflix.in No part of this material should be copied or reproduced without permission
approximation to the behaviour of
many gases under many conditions, although
it has several limitations. The ideal gas law
is often introduced in its common form:
PV  nRT
Where P is the absolute pressure of the
gas, V is the volume of the gas, n is number
of moles of gas, R̅ is universal gas constant,
& T is the absolute temperature of the gas. 3) Isothermal Process:
For air R̅ = 8.314 KJ/mol.K An is other mal process is a change of
n (in moles) is equal to the mass m (in gm) a system, in which the temperature remains
divided by the molar mass M ( in gm/ constant. It is also called as hyperbolic
mole) process. For Isothermal process, T =
m Constant or PV = Constant.
n
M For the isothermal process,
By replacing n with m / M, we get: P1V1  P2 V2
m
PV  RT
M
Defining the specific gas constant R as the
ratio R̅ /M
PV  mRT
Where R is specific Gas Constant.
For air R =0.287 KJ/Kg.K
Different process:
(4) Adiabatic Process
1) Constant volume process An adiabatic process is a process that
An isochoric process, also called a constant- occurs without the transfer of heat or
volume process, or an isometric process, is matter between a system and its
a thermodynamic process during which surroundings. Such processes are usually
the volume of the system undergoing such followed or preceded by events that do
a process remains constant. involve heat transfer.
For isocoric process, V = Constant For adiabatic process, PV γ = Constant.
P1 P2 P1V1γ = P2V2γ

T1 T2 γ 1
T2  P2  γ
OR  
T1  P1 

2) Constant pressure process


An isobaric process is a Thermodynamic
process in which the pressure stays constant.
For isobaric process, P = Constant. Adiabatic Process
V1 V2 γ 1
 T V 
T1 T2 OR, 2   2 
T1  V1 

© Copyright Reserved by Gateflix.in No part of this material should be copied or reproduced without permission
5) Polytropic Process Slop of isothermal and adiabatic process
A polytropic process is a thermodynamic on P-V diagram
process that obeys the relation:  For isothermal process
p n  C PV = C
Where n is polytropic index. Taking log in both the sides
P1V1n = P2V2n logP + logV = log C
n 1 Differentiating the equation
T P  n dP dV
OR, 2   2   0
T1  P1  P V
dP dV

P V
 dP  P
 
 dV  V

Polytropic Process
n 1
T2  V2 
OR ,  
T1  V1 
 For adiabatic process
Representation of different process on PVγ = C
P-V diagram: Taking log in both the sides
PVk  C log P + γ logV = log C
If k=0, P= C, process is isobaric process. Differentiating the equation
If k = ∞, V =C, process is isocoric process. dP dV
γ 0
If k=1 PV=C, process is isothermal process. P V
If k=n PVn =C, process is polytropic process. dP dV
 γ
If k=γ PV γ =C, process is adiabatic process. P V
Different processes on P-V diagram are
Slop of adiabatic process
shown in fig. γ
Slop of isothermal process
 The slop of adiabatic process is higher
than the slop of isothermal process.

WORK AND HEAT TRANSFER

1.9 WORK TRANSFER

Work is said to be done by a system if the


Sole effect on things external to system can

© Copyright Reserved by Gateflix.in No part of this material should be copied or reproduced without permission
be reduced to rising of a Weight. The different so work done for different path is
Weight may not be raised, but the net effect different. So work transfer is path function.
external to system would be the raising of a It is inexact differential.
Weight.
Let consider the Battery and Motor. 1.9.1 WORK TRANSFER FOR CLOSED
SYSTEM IN VARIOUS REVERSIBLE
PROCESSES

A) Constant Pressure Process


In the isobaric process pressure
remains constant.
So work transfer
2

The nature of work changes when W12   p.dv  P  V2  V1 


boundary between system and 1

Surrounding changes. So work done is


boundary phenomenon. It is transient form
of energy.
Wout

Surrounding System
Win B) Constant Volume Process
In the isochoric process volume
remains constant.
→ When Work is done on the system it is So work transfer in this process is zero.
considered as negative. V= constant
→ When work is done by the system, it is 2

considered as positive. Work transfer W12   p.dv  0


1
Let Consider a piston cylinder arrangement
(i.e. closed).

C) Isothermal Process

Work done by the system In isothermal process, T= C or


dW = F. dl = P.A.ds = P.dV P.V  P1V1  P2 V2  C
2
PV P V
W12   P.dV -------- (1) P  1 1  2 2 ---- (1)
1 V V
2 2
It is valid for closed system undergoing a dv
W12   p.dv  P1V1  
reversible process. V
1 1
So work done by closed system in
reversible process is area under p v V 
 P1V1In  2 
diagram about V axis.  V1 
If Path A and Path B are two paths but V  V 
initial and final point is same. Area is W12  P1V1In  2   m.RT In  2 
 V1   V1 

© Copyright Reserved by Gateflix.in No part of this material should be copied or reproduced without permission
1.10 HEAT TRANSFER

Heat is defined as a form of energy that is


transferred across the boundary by virtue
of a temperature difference. The
temperature difference is the potential and
heat transfer is the flux. Heat flow into the
system is taken as positive and heat flow
out of system is taken as negative. The
D) Adiabatic Process: process in which that does not cross the
boundary is adiabatic process. Heat
In adiabatic Process, transfer is a boundary phenomenon. Heat
PVγ  P1V1γ  P2 V2 γ  C transfer is transient form of energy. Heat

P
P V   P V
1 1
γ
2 2
γ transfer is Path function.
For the m mass of substance and ∆T
Vγ V γ

2 2
temperature difference, Heat transfer
P1V2 γ
W12   p dv   γ Q α m.∆T
1 1
V Q  m.C.T
 V 1-  V11-γ  Where m= mass of substance,
W12  P1V1γ  2  ΔT = temperature difference
 1   C = Specific heat and its unit is KJ/Kg.K.
P V γ .V 1-  P1V1γ .V11- If m = 1Kg,
 2 2 2
1 γ ΔT = 1 0 C, then C = Q
P1V1  P2 V2 MR(T1  T2 )
W12   1.10.1 SPECIFIC HEAT
 1  1
The specific heat of a substance is defined
as the amount of heat required to raise the
unit mass of substance through a unit rise
of temperature.
For the gases, in constant volume process it
is taken as CV and in constant pressure
Adiabatic Process Process it is taken as Cp.

(E) Polytrophic Process Examples


P V  P V MR(T1  T2 )
W12  1 1 2 2  Q.1 If a gas of Volume 6000 cm3 and at a
n 1 n 1 pressure of 100 KPa is compressed
Where n = Polytrophic index in reversible Process according to
PV2 = C until volume becomes 2000
cm3, determine the final pressure
and work done ?
Solution:
Given V1 = 6000 cm3 P1 = 100 KPa, V2
= 2000 cm3
P2 =? , W12  ?
P1V12  P2 V2 2
Polytropic Process 100 ×(6000)2 = P2 ×(2000)2

© Copyright Reserved by Gateflix.in No part of this material should be copied or reproduced without permission
100  60002 Work done in A-B process = Area
P2  under AB
20002
P2  900KPa WA-B = 50 × 105 × (0.4 – 0.2) J
= 106 J = +1 MJ
For the polytrophic process,
PBVBn  PBVCn
50×0.41.3=PB0.81.3
PB = 8.25 bar = 20.3 × 105 Pa
Work done in B-C process = Area
Work Transfer under BC on P-V diagram
2
P  VB  PB VC
W1 2   p.dv ----- (i) WBC  B
n 1
 50 10  0.4    20.3 105  0.8 
1
5
In the process PV2  P1V12  P2 V22  C 
C n 1
P 2 ----- (ii) WBC  1.25 10 J  1.25MJ
6
V
Substituting the Value a/p in So the total work done
equation (1) W  WAB  WBC  1  1.25 MJ
2
C = 2.25 MJ
W12   2 dv
1
V
Q.3 The limiting value of the ratio of the
1 1
 C    pressure of gas at the steam point
 V2 V1  and at the triple point of water
 P V2 P V2  when the gas is kept at constant
  2 2  1 1  volume is found to be 1.36605. What
 V2 V1 
is the ideal gas temperature of the
W12  P1V1 P2 V2 steam point?
=100×(6000)×10-6-900×(2000)×10-6 Solution:
W12  1.2KJ P
Given :  1.36605
It means 1.2 KJ work is done on the Pt
system. P
T = 273.16   =273.16 × 1.36605
 Pt 
Q.2 Determine the total work done by a
gas system, which follows the = 373.15 K
expansion Process as shown in T = 1000C
Figure.
Q.4 A platinum wire is used as a
resistance thermometer. The wire
resistance was found to be 10 ohm
and 16 ohm at ice point and steam
point respectively, and 30 ohm at
sulphur boiling point of 444.6°C.
Find the resistance of the wire at
500°C, if the resistance varies with
Solution:
temperature by the relation.
Given PA = PB = 50 bar =50 × 105 Pa,
R = R0 (1 +α t + βt2)
VA =0.2 m3, VB = 0.4 m3, VC =0.8 m3,
Solution:
WA-C = ?

© Copyright Reserved by Gateflix.in No part of this material should be copied or reproduced without permission
Given: ti=0°C, Ri = 10 ohm, ts =100°C, ∴ P1V1 = P2V2 = C
Rs = 16 ohm m 1.5
V1  1   1.29 m3
For sulphur, tb=444.6°C, Rb=30 ohm, 1 1.16
t = 500°C, R=? Work done
10 = R0 (1+0×α+β×02)
V  P 
16 = R0 (1+100×α + β×1002) W12  P1V1In  2   P1V1In  1 
30 = R0 (1 +444.6 ×α + β ×444.62)  V1   P2 
Solving the equation, we get  0.1 
R= 10 (1 +6.45 ×10-3t+ 4.48×10-6t2)  0.11.29In    2.51MJ
 0.7 
Substituting the value of t = 5000C
R= 10 (1 +6.45 ×10-3×500-4.48×
Q.7 680 kg of fish at 5°C are to be frozen
10-6×5002)
and stored at – 12°C. The specific
R = 31.05 ohm.
heat of fish Above freezing point is
3.182 KJ/Kg.K, and below freezing
Q.5 The piston of an oil engine, of area
point is 1.717 KJ/Kg K. The freezing
0.0045 m2, moves downwards 75
point is – 2°C, and the latent heat of
mm, drawing. In 0.00028 m3 of fresh
fusion is 234.5 KJ/Kg. How much
air from the atmosphere. The
heat must be removed to cool the
pressure in the cylinder is uniform
fish, and what percent of this is
during the process at 80 KPa, while
latent heat?
the atmospheric pressure is 101.325
Solution:
KPa, the difference being due to the
Given: m=680 kg, T1=50C, T2 = -120C,
flow resistance in the induction pipe
Cp1 = 3.182 KJ/Kg.K,
and the inlet valve. Estimate the
Cp2 = 1.717 KJ /Kg.K, Tf = -20C LH =
displacement work done by the air
234.5 KJ/Kg,
finally in the cylinder.
Heat to be removed above freezing
Solution:
point
Given: A=0.0045 m2, length of stroke
= 680 × 3.182 × {5 – (-2)} kJ
= 75 mm = 0.075 m
= 15.146 MJ
Volume of piston stroke = 0.0045 ×
Heat to be removed latent heat
0.075 m3
= 680 × 234.5 kJ
= 0.0003375 m3
= 159.460 MJ
∴ V2-V1 = 0.0003375 m3
Heat to be removed below freezing
As pressure is constant P = 80 KPa
point
So work done = P. (V2-V1)
= 680 × 1.717 × {– 2 – (– 12)} kJ
= 80 ×0.0003375 KJ
= 11.676 MJ
= 0.027 KJ
∴ Total Heat = 186.2816 MJ
= 27 J
Percentage of Latent heat = (Latent
heat/total heat) × 100
Q.6 A mass of 1.5 kg of air is compressed
= (159.460/186.2816) × 100
in a quasi-static process from 0.1
= 85.6 %
MPa to 0.7 MPa for which PV =
constant. The initial density of air is
1.16 kg/m3. Find the work done by
the piston to compress the air.
Solution:
Given: m=1.5 kg, P1=0.1 MPa, P2 =0.7
MPa, = 1.16 kg/m3, W=?
For quasi-static process, PV = C

© Copyright Reserved by Gateflix.in No part of this material should be copied or reproduced without permission
2 FIRST LAW OF THERMODYNAMICS

2.1 FIRST LAW OF THERMODYNAMIC 2.2.2 ENERGY IS A PROPERTY OF


FOR A CYCLE SYSTEM

Heat and work are different forms of From the equation (III), We get
energy. Both form of energy can be (dQ)B  (dQ)C  (δw)B  (δw)C ------ (III)
Conserved. As the result of Joule’s Rearranging the equation (III)
experiment, for a closed system undergoing (δQ)B  (δW)B  (δQ)C  (δw)C
a Cycle
(ΣQ) cycle =J (ΣW) cycle
(δQ)  (δW)B  (δQ)  (δW)C
J is Joule’s equivalent. This is first law of For path B and C, the difference of heat
thermodynamics for a closed system transfer and work transfer is constant. So
undergoing a cycle. Both Heat and work are this different must be a point function and
measured in the desired unit of energy property of the system. This property is
Joule (J). So constant of proportionality ‘J’ is known as energy.
unity. Q W  dE --------- (IV)
This is known as first law of
2.2 APPLICATION OF FIRST LAW OF thermodynamic for closed system
THERMODYNAMICS undergoing a process. It is valid for the
reversible and irreversible process.
2.2.1 HEAT TRANSFER IS A PATH Total Energy E = Kinetic Energy + Potential
FUNCTION Energy + Internal Energy.
In absence of motion and gravity, kinetic &
Let consider a cycle 1-A-2-B-1 and apply potential energy is Zero. In that case E = U
first law of thermodynamics for a cycle δQ ̶ δW = dU ----- (V)
(ΣQ) cycle =J (ΣW) cycle
(dQ)A  (dQ)B  (δw)A  (δw)B ---- (I)
Let consider another cycle 1-A-2-B-1 and
apply the first law of Thermodynamics
again
(dQ)A  (dQ)C  (δw)A  (δw)C ------ (II)
Solving the equation (I) or equation (II)
(dQ)B  (dQ)C  (δw)B  (δw)C ------ (III)
Where,
Difference (δw)B  (δw)C is not Zero  U = Internal energy
because work transfer is a Path function as  Unit of internal energy = KJ
it was proved with help of chapter 1, so  Internal energy is an extensive
(dQ)B  (dQ)C is not equal to Zero. So heat property.
transfer for the different path has the  This is first Law of thermodynamics for
different value. It depends upon the path. reversible and irreversible process.
So results of this discussion are: But we know that δW=P.dV for a
 Heat is a path function. reversible process, so
 The differentiation of the heat transfer Q  P.dV  dU ---- (VI)
is inexact differential (δQ).  This is first Law of thermodynamics for
 It is boundary phenomenon. a closed System undergoing a reversible
 it is transient form of energy. Process.

© Copyright Reserved by Gateflix.in No part of this material should be copied or reproduced without permission
2.2.3 ENERGY OF AN ISOLATED SYSTEM H = P.V+ U
IS CONSTANT Enthalpy is an extensive property and its
unit is KJ.
According to I Law of thermodynamics Enthalpy of an ideal gas is given by
δQ = δw + dE H = m.Cp.T
For isolated System δQ = 0 δw = 0
dE = 0 2.2.5.1 SPECIFIC ENTHALPY
E1  E 2
 Energy of an isolated System is always It is amount of enthalpy per unit mass.
constant. Specific enthalpy is an intensive property.
 Universe is an isolated system so Its unit is KJ/Kg.
energy of the Universe remains
Constant. 2.2.6 INTERNAL ENERGY
 It is known as law of conservation of
energy. All fluids store energy. The store of energy
within any fluid can be increased or
2.2.4 PERPETUAL MOTION MACHINE decreased as the result of various
(PMM-I) IS NOT POSSIBLE processes carried out on or by the fluid.
The energy stored within the fluid which
Perpetual machine is the machine which results from the internal motion of its atom
delivers the work without taking any input. and molecules is called as Internal Energy.
There is no such machine which would The Internal energy of any ideal gas is
continuously supply mechanical work given:
without some other form of energy U = m.Cv.T
dissipating. Such a machine is called as Internal Energy is an extensive property.
PMM-I. The efficiency of PMM-1 is given by It is a point function.
W W Unit of internal energy is KJ.
nPMMI  
Q O
2.2.6.1 SPECIFIC INTERNAL ENERGY
nPMMI  
It is amount of internal energy per unit
It means efficiency of PMM-1 is Infinite.
mass. Specific internal energy is an
But according to first law of
intensive property. It is denoted by u. Its
thermodynamics
unit is KJ/Kg.
(ΣQ) cycle = (ΣW) cycle
2.3 MEYER’S FORMULA
We know that Enthalpy can be given as
H = U + PV
For an ideal gas, H = m.Cp.T, U = m.Cv.T and
PV = mRT
If it is possible it will violate of first law of Substituting the values in equation
thermodynamics but it is law of nature. So H = m.Cv.T + mRT
 PMM-1 is not possible. m.Cp.T = mT (CV+R)
 The machine having the infinite CP R
efficiency is not possible. CP C   ,Cp  V=R
CV  1
2.2.5 ENTHALPY This formula is known as Meyer’s formula.
CP R
We know that   ,Cp 
Enthalpy of a Substance is defined as CV  1

© Copyright Reserved by Gateflix.in No part of this material should be copied or reproduced without permission
R There is no heat transfer in adiabatic
And CV  Process.
 1
5) Polytrophic Process:
According to first law of thermodynamics
2.4 HEAT TRANSFER IN DIFFERENT
δQ= δW + dU
PROCESS
mr(T1  T2 )
 mCV dT 
1) Constant Volume Process: n 1
According to first law of thermodynamics mR(T1  T2 ) mR(T1  T2 )
 
δQ = P.dV + dU  1 n 1
δQ = O + dU
 1 1 
δQ = dU = mCVdT  mR(T1  T2 )   
 n  1   1
mR(T1  T2 )    n 
  
n 1   1 
  n 
Q     work done
2) Constant Pressure Process:   1 
According to first law of thermodynamics
δQ = P.dV + dU
We know that for isobaric Process
dH = P.dV + dU
δQ = dH = m.Cp.dT

2.5 FREE EXPANSION

3) Isothermal Process:
According to first law of thermodynamics
δQ = P.dV + dU
For isothermal Process
dU = 0
δQ = P.dV = δW
V 
δQ = δW = P1 V1 ln  2  The expansion of gas against vacuum is
 V1  known as free Expansion. There is no work
done in free Expansion.

2 2

 W  0 although
1
 P.dv  0
1
4) Adiabatic Process:

© Copyright Reserved by Gateflix.in No part of this material should be copied or reproduced without permission
2
Steady state energy flow
Since work transfer =  P.dv is considered [Internal Energy + Kinetic energy +
1
Potential energy + Heat] 1-1
for reversible Process but free expansion is
= [Internal Energy + Kinetic Energy +
highly irreversible Process. Vacuum does
Potential Energy + Work] 2-2
not offer any resistance, so there is no work
1 1
transfer involved in free expansion. U1  mC12  mgZ1  Q  U 2  mC22  mgZ2  w Total
2 2
2.6 FIRST LAW OF THERMODYNAMICS 1
U1  mC12  mgZ1  Q  U 2 
FOR FLOW PROCESS 2
1
mC22  mgZ2  w cv  P1V1  P2 V2
Steady Flow Process 2
1
Steady flow means the rate of flow of mass U1  P1V1  mC12  mgZ1  Q
2
and energy across the control volume are 1
constant. At steady state of a System, any  U 2  P2 V2  mC22  mgZ2  w cv
2
thermodynamic property will have a fixed
1
value at the same state and does not change H1  mC12  mgZ1  Q
with respect to time. 2
1
 H 2  mC22  mgZ2  w cv Or
Flow Work 2
C12 Q
The Flow work is the work done by Fluid of H1   gZ1  
2 m
Mass m either at inlet or at the outlet of
1 w cv
control Volume. Flow work at inlet is taken  H 2  C22  gZ2 
as (-P1V1) and at outlet of control volume is 2 m
(+P1V1).where P1, V1 is pressure and It is known as steady Flow energy Equation
volume at the inlet of control volume and (SFEE)
P2,V2 is the pressure and volume at the Where
outlet of control volume. h1,h2 = Specific enthalpy at inlet and outlet
so total work Wtotal  WCV  P2 V2  P1V1 KJ/Kg
C1,C2 = Velocity at inlet and outlet m/Sec.
2.6.1 STEADY FLOW ENERGY EQUATION w cv = Work done by System KJ
δQ = Heat transfer KJ
By Appling the energy balance equation at Z1, Z2 = Elevation above on arbitrary datum
section 1-1 and section 2-2 ,m
(Energy at Section 1-1)= (Energy at
Section 2-2) 2.6.2 APPLICATION OF S.F.E.E.

1) Nozzle

For a turbine which is well insulated, and


Z1  Z2
SFEE equation:
C2 Q C2 W
h1  1  gZ1  o  h 2  2  gZ2  o
2 m 2 m
Q W
Here o  o  0
m m

© Copyright Reserved by Gateflix.in No part of this material should be copied or reproduced without permission
Z1  Z2
C12 C2
h1   h2  2
2 2
C2  C1  2(h1  h 2 )
2 2

If C1 is very less as Compared to C2, then


C2  2(h1  h 2 ) m/s.

Wcv  m0 (h1  h 2 )KJ


But in Case of turbine h1 is greater than h2
and in Case of Compressor h2 is greater
than h1.

4) Throttling Process

When the fluid flows through a constricted


passage, like an opened valve or an orifice
it is called as throttling Process. Throttling
2) Turbine process is irreversible process.
In throttling Process,
For a turbine which is well insulated, the Q W
 0
flow Velocities are often small, so the K.E m m
can be neglected and Z1  Z2 And change of P.E. P.K.E. is very small and
SFEE equation: ignored, then
C2 Q C2 W C2 Q C2 W
h1  1  gZ1  o  h 2  2  gZ2  o h1  1  gZ1   h 2  2  gZ2 
2 m 2 m 2 m 2 m
Wcv h1  h 2
h1  h 2  o
m So Enthalpy of fluid before throttling and
W  m (h1  h 2 )KJ
0
after throttling remains Constant.

3) Compressor 2.7 FIRST LAW OF THERMODYNAMICS


FOR VARIABLE FLOW PROCESS
For a turbine which is well insulated, the
flow Velocities are often small, so the K.E
Variable Flow
can be neglected and =
SFEE equation:
If there is a change of property at the same
C12 Q C22 W state, Process is called as Variable flow or
h1   gZ1  o  h 2   gZ2  o
2 m 2 m unsteady Flow Process. In the unsteady
W flow then there is an accumulation of mass
h1  h 2  ocv
m and energy.

© Copyright Reserved by Gateflix.in No part of this material should be copied or reproduced without permission
The mass Accumulated in control volume is then removed and 105 kJ of heat
given by flow to the surroundings as the fluid
 dm  dm1 dm 2 goes to state 3. The following data
    are observed for the fluid at states 1,
 dt cv dt dt
2 and 3.
Applying the law of conservation of energy
State v (m3) t (°C)
Rate of energy accumulation = Rate of
1 0.003 20
energy inlet Rate of energy outlet
2 0.3 370
 dE  d 0 1  3 0.06 20
 cv   m1 h1  m1c1  m1gZ1  Q 
2

 dt  dt  2  For the fluid system, calculate E2
d 1  and E3, if E1 = 0
  m 2 h 2  m 2c22  m 2 Z2  W  Solution:
dt  2  From first law of thermodynamics
δQ = dE +PdV
Q1-2 = (E2 – E1) +
= (E2 – E1) + 0.1× 103 (0.3- 0.003)
Because Process 1-2 is insulated so
Q1-2 = 0 and E1 = 0
So E2 = - 29.7 KJ
Q2-3 = (E3 – E2) +
-105=(E3+29.7)+0.1× 103 (0.06- 0.3)
Unsteady flow E3 = -110.7 KJ
 dE   0 1 0 2 0 δQ  Q.2 A domestic refrigerator is loaded
  cv=  m1 h1 + m1 c1 +m1 g21 + -
 dt   2 δt  with food and the door closed.
During a certain period the machine
 0 1 0 2 0 δw  consumes 1 kWh of energy and the
 me h 2 + 2 me c2 +me g22 + δt 
  internal energy of the system drops
Neglecting the Kinetic energy and potential by 5000 kJ. Find the net heat
energy transfer for the system.
 dU  Solution:
 cv  m1 h1  me h 2  Q  W
0 0 0 0
 According to first law of
 dt 
Where thermodynamics
δQ ̶ δW = dE
m0l  Mass Flow rate at inlet Kg/Sec
δQ ̶ δW = dE
m0e  Mass Flow tare at outlet Kg/Sec δW = -1× 3600 = - 3600 KJ.
Q0  Heat transfer Rate KJ/Sec dE = -5000 KJ
δQ + 3600 = -5000
W0  Work done KJ/Sec
δQ = - 8600 KJ = -8.6 MJ
h1 , h 2  Specific Enthalpy at Inlet or Outlet.
Q.3 The properties of a certain fluid are
Examples related as follows:
u = 196 + 0.718t
Q.1 A slow chemical reaction takes place pv = 0.287 (t + 273)
in a fluid at the constant pressure of Where u is the specific internal
0.1 MPa. The fluid is surrounded by energy (kJ/kg), t is in °C, p is
a perfect heat insulator during the pressure (KN/m2), and v is specific
reaction which begins at state 1 and volume (m3/kg). For this fluid, find
ends at state 2. The insulation is Cv and Cp.

© Copyright Reserved by Gateflix.in No part of this material should be copied or reproduced without permission
Solution: dE = - 8 × 40 = - 320 KJ
 dh  Q  dE  W12   320  2932.5
we know that Cp =  
 dT P Q = 2612.5KJ
 d  u  pu   2612.5 KJ heat is supplied to the
  system.
 dT P
 d 196  0.718t  0.287  t  273   Q.5 The heat capacity at constant
  pressure of a certain system is a
 dT 
 P function of temperature only and
 0  0.718dt  0.287(dt  0)  may be expressed as
  41.87 0
 dT P Cp  2.093  J/ C
 0.718dt  0.287dt  t  100
  Where t is the temperature of the
 dT P system in °C. The system is heated
 1.005dt  while it is maintained at a pressure
 
 dT P of 1 atmosphere until its volume
but T = t + 273 so dT = dt increases from 2000 cm3 to 2400
Cp =1.005KJ/Kg.K cm3 and its temperature increases
 du  from 0°C to 100°C.
again Cv    a) Find the magnitude of heat
 dT V interaction.
 d(196  0.718t)  b) How much does the internal
 
 dT V energy of the system increase?
 0.718dt  Solution:
  If T is temperature in K, then t = T-
 dT V 273 and t + 100 = T – 173
but T = t + 273 so dT = dt 373
 41.87 
CV =0.718KJ/Kg.K Q12    2.093  dT
273 
T  173 
373
Q.4 A mass of 8 kg gas expands within a  418.7 
  2.093T  ln
T  173  273
flexible container so that the P-V

relationship is PV1.2 = constant. The
initial pressure is 1000 KPa and the 200
= 2.093(373 - 273) + ln
initial volume is 1 m3. The final 100
pressure is 5 KPa. If specific internal = 209.3 + 41.87ln2 = 238.32 J
2
energy of the gas decreases by 40
Q12   E 2 – E1    PdV
KJ/kg, find the heat transfer in
1
magnitude and direction. E2 – E1 = Q1-2 –P.(V2-V1)
Solution: =238.32-101.325(0.0024 0.0020)×1000J
For the polytrophic process = 197.79 J
P1V1n = P2V2n
1000 ×11.2 = 5 ×V21.2 Q.6 a gas undergoes a thermodynamic
V2 = 82.7 m3 cycle consisting of three processes
PV P V beginning at an initial state where
work done W12  1 1 2 2
n 1 P1 = 1 bar, V1 = 1.5 m3 and U1 = 512
1000 1  5  82.7 kJ. The processes are as follows:
 i) Process 1–2: Compression with
1.2  1
PV = constant to P2 = 2 bar, U2 =
W12  2932.5KJ
690 kJ

© Copyright Reserved by Gateflix.in No part of this material should be copied or reproduced without permission
ii) Process 2–3: W23 = 0, Q23 = –150 Q.8 A turbine operates under steady
kJ, and flow conditions, receiving steam at
iii) Process 3–1: W31 = +50 kJ. the following state: Pressure 1.2
Neglecting KE and PE changes, MPa, temperature 188°C, enthalpy
determine the heat interactions 2785 kJ/kg, velocity 33.3 m/s and
Q12 and Q31. elevation 3 m. The steam leaves the
Solution: turbine at the following state:
Process (1-2): process is isothermal Pressure 20 KPa, enthalpy 2512
process. for isothermal process, kJ/kg, velocity 100 m/s, and
Q1-2 = (U2-U1) elevation 0 m. Heat is lost to the
surroundings at the rate of 0.29
= (690 – 512) + 1 105 1.5 KJ/s. If the rate of steam flow
through the turbine is 0.42 kg/s,
= (690 – 512) + 1 102 1.5
what is the power output of the
= 178 – 103.972 turbine in kW?
= 74.03 KJ Solution:
As W2-3 is ZERO so it is constant P1 = 1.2 MPa t1 = 188°Ch1 = 2785
volume process. As W31 is positive kJ/kg C1 = 33.3 m/s Z1= 3 m P2 =
so expansion is done. 20 kPa
For the process (2-3) h2 = 2512 kJ/kg C2 = 100 m/s
Q2-3 = W2-3 + (U3-U2) dQ/dt = – 0.29 kJ/s dW/dt =?
–150 = 0 + (U3 - 690)
U3 = 540 KJ
Process (3-1)
Q3-1=W3-1+ (U1-U3) = 50+(512-540)
= 50 -28 = 22 KJ

Q.7 A blower handles 1 kg/s of air at


20°C and consumes a power of 15
kW. The inlet and outlet velocities of By SFEE
air are 100 m/s and 150 m/s C12 gZ1 Q
m(h1   ) 
respectively. Find the exit air 2000 1000 dt
temperature, assuming adiabatic C2 gZ2 W
conditions. Take Cp of air is 1.005 m(h 2  2  )
2000 1000 dt
KJ/Kg-K.
Solution: 33.3 9.8  3
2
0.42(2785   )  0.29 
From S.F.E.E. 2000 1000
c2 1002 9.8  0 W
mh1  m 1  mgZ1  Q / dt 0.42(2512   )
2 2000 1000 dt
1 W
 mh 2  mC22  mgZ2  W / dt 1169.655  1057.14 
2 dt
W
 112.515kW
1×h1 + 1× (100)2/2000 + 1×9.8×Z dt
+ 0 = 1×h2 + 1× (200)2/2000 +
1×9.8×Z + 15 Q.9 A nozzle is a device for increasing
h2 – h1 = 8.75 the velocity of a steadily flowing
Cp (T2-T1) = 8.75 stream. At the inlet to a certain
T2 = 20 + 8.7 nozzle, the enthalpy of the fluid
= 28.7 0C passing is 3000 kJ/kg and the

© Copyright Reserved by Gateflix.in No part of this material should be copied or reproduced without permission
velocity is 60 m/s. At the discharge compressor at a temperature of
end, the enthalpy is 2762 KJ/kg. The 16°C, a pressure of 100 kPa, and an
nozzle is horizontal and there is enthalpy of 391.2kJ/ kg. The gas
negligible heat loss from it. leaves the compressor at a
a) Find the velocity at exists from temperature of 245°C, a pressure of
the nozzle. 0.6 MPa, and an enthalpy of
b) If the inlet area is 0.1 m2 and the 534.5kJ/ kg. There is no heat
specific volume at inlet is 0.187 transfer to or from the gas as it
m3/Kg, find the mass flow rate. flows through the compressor.
c) If the specific volume at the a) Evaluate the external work done
nozzle exit is 0.498 m3/Kg, find per unit mass of gas assuming
the exit area of the nozzle. the gas velocities at entry and
Solution: exit to be negligible.
Velocity at exit from S.F.E.E. b) Evaluate the external work done
C2 gZ1 Q per unit mass of gas when the
m(h1  1  )  gas velocity at entry is 80 m/s
1000 1000 dt
and that at exit is 160 m/s.
C2 gZ2 W
m(h 2  2  ) Solution :
2000 1000 dt t1 = 16°C, P1 = 100 kPa, h1 = 391.2
kJ/kg,t2=245°C P2=0.6 MPa=600 kPa
h2 = 534.5 kJ/kg
For V1 and V2 negligible and Z1= Z2
So SFEE
m(h1  0)  0  m(h 2  0)  W
W  1 (h1  h 2 )  (391.2  5345)kJ / kg
= –143.3 kJ/kg
h1 = 3000 kJ/kg C1 = 60 m/s h2 = C1 = 80 m/s; C2 = 160 m/s
2762 kJ/kg Then SFEE
W Q
  0 Z1  Z2 c2
dt dt m(h1  1  0)  0 
1000
 c12   c12 
h
 1    0  h
 2  0 c22
 1000   1000  m(h 2   0)  W
1000
602 c2  802 
(3000  )  0  (2762  2 )  0 1  391.2   0
1000 1000 1000
 
C22 = 602/ 2000 +3000 - 2762
C2 = 692.532 m/ s  1602 
 1  534.5   0   W
A C 0.1 60  1000 
Mass flow rate (m)  1 1 
v1 0.187 δW= (–143.3 – 9.6) kJ/kg
= 32.1 Kg/sec = –152.9 kJ/kg
AC Q.11 Air flows steadily at the rate of 0.4
Mass flow rate (m) = 2 2
v2 kg/s through an air compressor,
A  692.532 entering at 6 m/s with a pressure
32.1  2
0.498 of 1 bar and a specific volume of
A2 = 8.02 m2 0.85 m3/kg, and leaving at 4.5 m/s
with a pressure of 6.9 bar and a
Q.10 A gas flows steadily through a rotary specific volume of 0.16 m3/kg. The
compressor. The gas enters the internal energy of the air leaving is

© Copyright Reserved by Gateflix.in No part of this material should be copied or reproduced without permission
88 KJ/kg greater than that of the air m.V2 0.4  0.16
A2    0.01422 m2
entering. Cooling water in a jacket V2 4.5
surrounding the cylinder absorbs
heat from the air at the rate of 59 W.
Calculate the power required to
drive the compressor and the inlet
and outlet cross-sectional areas.
Solution :
m1 = 0.4 kg/s,C1 = 6 m/s P1 = 1 bar =
100 KPa v1 = 0.85 m3 /kg m2 = 0.4
kg/s C2 = 4.5 m/s
P2 = 6.9 bar = 690 kPa v2 = 0.16 m3
/kg u2 = u1 + 88 kJ/kg

by SFEE
c12 Q
m(h1   0) 
1000 dt
c 2
W
 m(h 2  2  0) 
2000 dt
c12 Q
m(u1  P1v1  )
1000 dt
c 2
W
 m(u 2  P2 v 2  2 ) 
2000 dt
W
m
dt
c12  c22 Q
u1  u 2  P1v1  P2 v2  )
2000 dt
 0.4(88  85  110.4  0.0076)  0.059
 45.357  0.059
= – 45.416 kJ
AC
Mass flow rate  m   1 1
V1
m.v1 0.4  0.85
A1    0.0567m2
c1 6
AC
Mass flow rate  m   2 2
V2

© Copyright Reserved by Gateflix.in No part of this material should be copied or reproduced without permission
3 SECOND LAW OF THERMODYNAMICS, ENTROPY

The first law of thermodynamics states that A heat engine that violates the
certain energy is balanced when system Kelvin‐Planck statement of the second law
undergoes a thermodynamic process. But it cannot be built.
does not give any idea whether a particular
process is in feasible condition or not. It 3.1.2 CLAUSIUS STATEMENT OF SECOND
does not predict the direction of process. It LAW
is second law of thermodynamics which
predict whether process is in feasible or It is impossible to construct a device that
not with the concept of entropy. operates in a cycle and produces transfer of
heat from a lower‐temperature body to
3.1 THERMAL RESERVOIR higher‐temperature body without any
external work. If such a device is possible
A thermal energy reservoir is defined as a its C.O.P. will be infinite. So Refrigerator
large body of infinite heat capacity which is having COP infinite is not possible.
capable absorbing or rejecting the heat
without change of temperature.
Source: The thermal energy reservoir from
which heat can be absorbed without
change of temperature is known as Source.
Sink: The thermal energy reservoir in
which heat can be rejected without change
of temperature is known as Sink.
There are two statements of second law of
thermodynamics
A refrigerator that violates the Clausius
3.1.1 KELVIN – PLANK STATEMENT OF statement of the second law cannot be
SECOND LAW built.
It is impossible for any device that operates 3.1.3 HEAT ENGINE
on a cycle to receive heat from a single
reservoir and produce a net amount of Heat engines convert heat to work. Heat
work. In other words, no heat engine can engine is a thermodynamic system
have a thermal efficiency of 100%. The operating in a thermodynamic cycle to
machine having 100% efficiency is known which heat is transfer and it is converted in
as PMM-II. So PMM-II is not possible. to work.

Fig.: Heat Engine


3.1.4 THERMAL EFFICIENCY

© Copyright Reserved by Gateflix.in No part of this material should be copied or reproduced without permission
Thermal efficiency is the index of low temperature body and delivers heat to
performance of a heat engine. It is the high temperature body. To accomplish this
fraction of the heat input that is converted energy transfer Heat pumps receive the
to the network. external energy in the form of work or heat.
Work output Wout = Q1- Q2
Efficiency
W Q1  Q2 Q
   1  2 -------(3.1)
Q1 Q1 Q1

3.1.5 REFRIGERATOR AND HEAT PUMP

The transfer of heat from a low


temperature region to a high‐temperature
one requires special devices called
refrigerators. Refrigerators are cyclic
devices, and the working fluids used in the Fig: Heat Pump
cycles are called refrigerant.
3.1.8 CO-EFFICIENT OF PERFORMANCE

The index of performance of a heat pump is


given in the term of co-efficient of
performance (COP).It is defined as the ratio
of desired effect to given input. For the
Heat pump desired effect = Heat supplied
to high temperature body (Q1)
Work done on the refrigerator W = Q1 - Q2
Fig: Refrigerator So
Desired Effect Q1
(COP)R  
3.1.6 CO-EFFICIENT OF PERFORMANCE workinput W
Q1
The index of performance of a refrigerator (COP)HP  -------(3.3)
Q1  Q2
is given in the term of co-efficient of
performance (COP).It is defined as the ratio Subtracting the equation (3.2) from
of desired effect to given input. equation (3.3)
Q Q
For the refrigerator  COP HP   COP R  1 2  1
Desired effect = Heat rejected from low Q1  Q2
temperature body (Q2) (COP)HP  (COP)R  1 ------(3.4)
Work done on the refrigerator W = Q1-Q2
From the equation (3.1) and equation (3.3)
So
1
Desised Effect Q2 (COP)HP  ---------(3.5)
 COP R   
work input W
Refrigerators and heat pumps are
Q2
(COP)R  essentially the same devices; they differ in
Q1  Q2 their Objectives only. Refrigerator is to
maintain the refrigerated space at a low
3.1.7 HEAT PUMP temperature. On the other hand, a heat
pump absorbs heat from a low‐
Heat pump is a thermodynamic system temperature source and supplies the heat
operating in a cycle that removes heat from to a warmer medium.

© Copyright Reserved by Gateflix.in No part of this material should be copied or reproduced without permission
3.2 EQUIVALENCE OF KELVIN PLANK surrounding and as a result of expansion
AND CLAUSIUS STATEMENT the gas temperature reduces from TH to TL.

The two statements of the second law are


equivalent. In other words, any device
violates the Kelvin‐Planck statement also
violates the Clausius statement and vice
versa.

Process (3‐4):
Reversible isothermal compression: The
gas is allowed to exchange heat with a sink
at temperature TL as the gas is being
slowly compressed. The heat is transferred
from the system to the surroundings such
that the gas temperature remains constant
at TL.
Fig:-The violation of the Kelvin‐Planck
statement leads to violation of Clausius. Process (4‐1):
Reversible adiabatic compression: The gas
3.3 CARNOT CYCLE temperature is increasing from TL to TH as a
result of reversible adiabatic compression.
The efficiency of a heat‐engine cycle greatly Carnot cycle is the most efficient cycle
depends on how the individual processes operating between two specified
that make up the cycle are executed. The temperature limits. The efficiency of all
net work (or efficiency) can be maximized reversible heat engines operating between
by using reversible processes. The best the two same reservoirs are the same.
known reversible cycle is the Carnot cycle. The thermal efficiency of a heat engine
Consider a gas in a cylinder‐piston (closed (reversible or irreversible) is:
system). The Carnot cycle has four W Q
processes: th   1 L
QH QH
For the Carnot cycle, it can be shown:
Process (1‐2) Reversible isothermal
T
expansion: th  1  L -------(3.6)
The gas expands slowly, doing work on the TH
surroundings. The heat transfers in the
3.3.1 CARNOT THEOREM
Reversible process from the heat source at
TH to the gas.
It states that all heat engines operating
between same temperature limit none has
Process (2‐3) Reversible adiabatic
a higher efficiency than a reversible engine.
expansion:
The efficiency of an irreversible (real) cycle
The cylinder‐piston is now insulated
is always less than the efficiency of the
(adiabatic) and gas continues to expand
Carnot cycle operating between the same
reversibly. So, the gas is doing work on the
two reservoirs.

© Copyright Reserved by Gateflix.in No part of this material should be copied or reproduced without permission
 If ηth ≤ ηth Carnot, it may be reversible or Entropy of a system is defined as the
irreversible heat engine measure of degree of molecular disorder or
 If ηth ˃ ηth Carnot, Heat engine is not random existing in the system. If higher is
possible. the randomness higher will be entropy.
When heat is supplied to the system,
3.4 REFRIGERATION AND HEAT PUMP randomness of the system increases, so the
WORKING ON REVERSED CARNOT CYCLE entropy of the system increases. Reversed
effect can be measured when heat is
A refrigerator or heat pump that operates removed from the system.
on the reverse Carnot cycle is called a
Carnot Refrigerator, or a Carnot heat pump. 3.6.2 CLAUSIUS INEQUALITY LAW FOR
The Coefficient of performance of any REVERSIBLE CYCLE
refrigerator or heat pump (reversible or
irreversible) is given by: Consider the cycle shown below composed
QL QH of two reversible processes A and B. Apply
 COP R  ,  COP HP 
the Clausius inequality for this cycle.
QH  QL QH  QL
COP of all reversible refrigerators or heat
pumps can be given by:
T T
 COP R  L and  COP HP  H
TH  TL TH  TL
If (COP)R˂ (COP)R,Rev The refrigerator is
irreversible Refrigerator
If (COP)R= (COP)R,Rev The refrigerator is
reversible Refrigerator Apply the Clausius inequality for the cycle
If (COP)R˃ (COP)R,Rev Refrigerator is not made of two internally reversible
possible. processes:
3.5 CLAUSIUS THEOREM  Q 
  Tnet int rev  0
it states that for a cycle (reversible or 2 2
irreversible cycle) the cyclic integral of  Qnet   Q 
δQ
is equal to zero or less than Zero. It
1  T   1  Tnet 
T    
int rev
 
int rev
along path A along path B
means
 Q  Since the quantity (Qnet/T) rev is
  T   0 -------(3.8) independent of the path and must be a
property, this property is known as
 δQ  entropy S.
if∮  ˂ 0, the cycle is irreversible cycle.
 T  The entropy change during a process is
 δQ  related to the heat transfer and the
if∮   0 the cycle is reversible cycle. temperature of the system. The entropy is
 T 
given the symbol S (kJ/K), and the specific
 δQ 
if∮   0 the cycle is not possible. entropy is s (kJ/kgK).
 T  The entropy change during a reversible
process is defined as
3.6 ENTROPY
 Q 
dS   
3.6.1 PHYSICAL SIGNIFICANCE OF  T Re v
2
ENTROPY  Q 
S2  S1     --------(3.9)
1
T Re v

© Copyright Reserved by Gateflix.in No part of this material should be copied or reproduced without permission
3.6.3 CLAUSIUS INEQUALITY LAW FOR the net heat transfer is positive so the
IRREVERSIBLE CYCLE entropy change will be positive. It means
entropy of the system will increase.
Consider the cycle 1-A-2-B-1, shown below,
where process A is arbitrary that is, it can ii) Heat is rejected from the system
be either reversible or irreversible, and
process B is internally reversible. if heat is rejected from the system, for
reversible process
dQ
ds  the net heat transfer is negative so
T
the entropy change will be negative. It
means entropy of the system will decrease.

iii) No heat Transfer or adiabatic Process

A cycle composed of reversible and If there is no heat transfer between the


irreversible processes. system and surrounding the net heat
 Q  transfer will be zero. It means entropy of
  Tnet int rev  0 the system will be constant.
dQ
2
 Qnet 
2
 Q  ds 
1  T   1  Tnet   0 T
   
int rev
 
int rev dS = 0
along path A along path B S2 = S1
So reversible adiabatic process is known as
The integral along the reversible path, isentropic process.
process B, is the entropy change S1 –S2.
Therefore 3.6.5 ENTROPY CHANGE FOR
2
 Q  IRREVERSIBLE PROCESS
1  Tnet   S1  S2  0
2
Qnet  kJ 
Ssys  S2  S1  
2
Q  
S2  S1   net T K
1
T 1
Here, the inequality is to remind us that the
In general the entropy change during a
entropy change of a system during an
process is defined as
irreversible process is always greater than
Qnet
dS  -------(3.10)  δQ 
T   , called the entropy transfer. That is,
 = holds for the internally reversible  T 
process some entropy is generated or created
 >holds for the irreversible process. during an irreversible process, and this
generation is due entirely to the presence
of irreversibility. The entropy generated
3.6.4 ENTROPY CHANGE OF SYSTEM IN
during a process is called entropy
REVERSIBLE PROCESS
generation and is denoted as Sgen. We can
remove the inequality by noting the
i) Heat is added to the system
following
if heat is added to the system, for a 2
Q  kJ 
reversible process, Ssys  S2  S1   net  Sgen  
dQ 1
T K
ds 
T

© Copyright Reserved by Gateflix.in No part of this material should be copied or reproduced without permission
2
Sgen is always a positive quantity or zero.
Its value depends upon the process and Qnet  TdS          (3.11)
1
thus it is not a property. Sgen is zero for an
This is the second law of thermodynamics
internally reversible process. And its value
for a reversible process.
increases with the irreversibility. So
entropy generation is a path function.
Entropy Change of an Isolated System:- For
the isolated system the heat transfer is
zero.
δQ
So, dS  for the isolated system, δQ = 0
T

In the above figure, the heat transfer in an


internally reversible process is shown as
the area under the process curve plotted on
the T-S diagram.

3.7 COMBINED FIRST LAW AND SECOND


LAW OF THERMODYNAMICS

According to first law of thermodynamics


So δQ = P.dV + dU -----------------(3.12)
dS≥0 for the reversible process
The total entropy change of an isolated According to first law of thermodynamics
system during a process always increases δQ = T.dS ---------------(3.13)
or, in the limiting case of a reversible for the reversible process
process, remains constant. Universe is an From the equation (3.12) and (3.13)
isolated system so entropy of universe T.dS = P.dV + dU ----------- (3.14)
during a process always increases or, in the it is valid for reversible as well as
limiting case of a reversible process, irreversible process.
remains constant. Equation (3.14) can be written as
The increase in entropy principle can be T.dS = dH – VdP ------(3.15)
summarized as follows:
0 Irreversible processes 3.8 ENTROPY CHANGE FOR AN IDEAL

Sgen  Stotal   0 Re versible processes GAS
 0 Im possible processes
 From the Equation (3.14) entropy change is
Heat Transfer as the Area under a T-S given:
Curve: P.dV dU
For the reversible process, the equation for dS  
T T
dS implies that
For an ideal gas,
Qnet
dS  P m.R
T dU  m.CV dT and 
T V
Qnet  TdS
then
The heat transfer in a reversible process is m.R.dV m.CVdT
the differential area under the process dS  
V T
curve plotted on the T-S diagram.

© Copyright Reserved by Gateflix.in No part of this material should be copied or reproduced without permission
Integrating the equation from state 1 to From the Equation (3.14)
state 2 T.dS = P.dV + dU
2 2 2
m.R.dV m.C dT For the constant volume process, dV = 0
1dS  1 V  1 TV So T.dS = dU = m.CV dT
 dT  T
S2  S1  m.R In
V2 T
 m.Cv .In 2        (3.16)         (3.20)
V1 T1  dS  v m.Cv
From the Equation (3.1)
From the equation (3.15) T.dS = dH - V.dP
dH VdP For the constant pressure process, dP = 0
dS   So
T T
T.dS = dH = m.CP dT
For an ideal gas,dH = m.CP dT and
V m.R  dT  T
         (3.21)
T P  dS p m.Cp
m.R.dP m.CP dT From the equation 3.20 and 3.21 it is
Then dS   cleared that slop of isobaric process is
P T
2 2 2 lesser than slop of isocoric process on T – S
m.R.dP m.C dT
1dS  1 P  1 TP diagram.

P2 T Examples
S2  S1  m.R In  m.Cp .In 2        (3.17)
P1 T1 Q.1 An ideal gas cycle is represented by
a rectangle on a P-V diagram. If P1
Solving the equation (3.16) and (3.17), We and P2 are the lower and higher
get pressures and V1 and V2, the smaller
V2 P and larger volume, respectively.
S2  S1  m.Cp In  m.Cv .In 2        (3.18)
V1 P1 Then
a) Calculate the work done per cycle.
3.9 ENTROPY CHANGE FOR FINITE BODY b) Indicate which parts of the cycle
involve heat flow in the gas
For the finite body entropy change may be c) Show that
written as  1

2 2
δQ
2
m.c.dT P2 V1

1dS  1 T  1 T P2  P1 V2  V1
 If heat capacities are constant.
S2  S1  m.C.In 2      (3.19)
1

3.10 SLOP OF ISOBARIC PROCESS AND


ISOCHORIC PROCESS ON T-S DIAGRAM

Solution:
a) w=area of the cycle
 (P2  P1 )(V2  V1 )
b) Process a-b and b-c
Heat absorbed by 1 Mole of gas in
one cycle
Isobaric and isocoric process

© Copyright Reserved by Gateflix.in No part of this material should be copied or reproduced without permission
Q  Qab  Qbc  CV  Tb  Ta   CP (TC  Tb )
P1
Now, Ta  Tb and P2 V1  RTb
P2
V PV
Tc  Tb 2 ;Tb  2 1
V1 R
 P  V 
Q  CV Tb 1  1   CP TP  2  1 
 P2   V1 
P2 V1   P2  P1   V2  V1  
 CP    CP   
R   P2   V1   (a) Maximum efficiencies of the
Ans….c) heat engine cycle
T 313
n max  1  2  1   1  0.358  0.642

W

 P2  P1  V2  V1  T1 873
Q P2 V1   P2  P1   V2  V1   = 64.2 %
CP    CV   W
R   P2   V1   Again 1 = 0.642
R  P2  P1  V2  V1  Q1
 W1  0.642  2000  1284KJ
CV V1  P2  P1   CP P2  V2  V1 
CP  CV Maximum cop of the refrigerator
 cycle
V1 P2
CV  CP T3
V2  V1 P2  P1 (cop)max 
T2  T3
γ 1
 253
V1 γP2   4.22
 313  253
V2  V1 P2  P1
Q
Also cop  4  4.22
Q.2 A reversible heat engine operates W2
between two reservoirs at Since w1  w 2  w  360KJ
temperatures of 600℃ and 40℃.  w 2  w1  w  1284  360  924KJ
The engine drives a reversible Q4 = 4.22 × 924 = 3899 KJ
refrigerator which operates
between reservoirs at temperatures 
   Q2  Q1  W1  2000 –1284  716 KJ
at 40℃ and -20℃. The heat transfer Heat rejection to t w  400C reservoir
to the heat engine is 2000 KJ and the  Q2  Q3  716  4823  5539 KJ
net work output of the combined
b) Efficiency of the actual heat
engine refrigerator plant is 360 KJ.
engine Cycle
(a)Evaluate the heat transfer to the
N = 0.4 ×ηmax = 0.4 × 0.642
refrigerant and the net heat transfer
to the reservoir at 400C (b) W1  0.4  0.642  2000  513.6 KJ
Reconsider (i) given that the  W2  513.6 – 360  153.6 KJ
efficiency of the heat engine and the Cop of the actual refrigerator as do.
cop of the refrigerator are each 40% COP = 0.4 × 4.22 = 1.69
of the maximum possible values. Therefore
Q4  153.6 1.69  259.6 KJ
Q3  259.6  153.6  413.2KJ

© Copyright Reserved by Gateflix.in No part of this material should be copied or reproduced without permission
Q2  Q1  W1   2000  –  513.6  1486.4 KJ Since  S  0
Heat rejected the 400C
reservoir 98 103  W
=Q2  Q3  413.2  1486.4  1899.6 KJ 630  0
100
W
Q.3 A System has a heat capacity at 980   630  0
100
constant volume CV = AT2 Where
W
A=0.042J/K3. The system is  350
originally at 200 K , and a thermal 100
reservoir at 100 K is available. What W (max) = 35000 J = 35 KJ
is the maximum amount of work
that can be recovered as the system Q.4 A body of constant heat capacity CP
is cooled down to the temp of the and initial temperature T1 is placed
reservoir? in contact with a heat reservoir at
Solution temperature T2 and comes to
Heat removed from the system thermal equilibrium with it. If
T2  T1 , Calculate the entropy
change of the universe and show
that it is always positive.
Solution:
T2
dT T
S body  mCP   mCP ln 2
T1
T T1

S reservoir  mCP
 T2  T1 
T2
 T  T 
T2 T2 100K S universe=mCP  In 2  1  1  
Q  CV dT   0.042T 2dT  T1  T2  
T1 T1  200K T T 1
Putting 1- 1 =n, 2 =
 0.042[T / 3](100K)
3
(200K) T2 T1 1-n
0.042 T
 J / K 3  (1003  2003 )K3 Since 1 >1 , n is positive
3 T2
 98 103 J Suniverse n 2 x3 n 4
100K
dT
100K
dT  ln1  ln 1  n   n    
 S System   CV   0.042T 2 mCP 2 3 4
200K
T 200K T (ds) > 0 It means entropy of
universe is always positive.
0.042

2 Q.5 A hypothetical device is supplied
J / K3  1002  2002  K 2  630J / K with 2 kg/s of air at 4 bar, 300 K.
Two separate streams of air leave
Q1  W 98X103  W the device, as shown in figure below.
 S res= 
Each stream is at ambient pressures
Tres 100
 S Working flued in HE = 0 of 1bar, and the mass flow rate is the
same for both streams. One of the
 S =  S system =  S ros exits streams is said to be at 330 K
98X103  W while the other is at 270 K. The
 630  ambient temperature is 300 K.
100

© Copyright Reserved by Gateflix.in No part of this material should be copied or reproduced without permission
Determine whether such a device is Where C = Specific heat of fluid.
possible. Solution:
Solution: Heat given by part-1
The entropy generation rate for the = heat taken by part-2
control Volume mc  T1  T   mc  T  T2 
T T 
T 1 2 
 2 
Change of entropy
ds  (dS)1  (dS)2
T T
dT dT
 mc   mc 
T1
T T2
T
T T
 mc ln    mcln  
 T1   T2 
2
S gen  m
 e Se  m
 i Si  T2   T 
 mc ln    mc ln  
m
 2S2  m
 3S3  m
 1S1  T1.T2   T1.T2 
m  3S3   m
 2S2  m  3  S1
 2 m T T
But T  1 2
 2 S2  S1   m
m  3 S3  S1  2
Substituting the value of T in the
T2 P Expression
Now S2  S1  CP In  R In 2
T1 P1   T1  T2  
330 1   2  
 1.005 In  0.287In  0.494KJ / kg.K dS  2mc In   

300 4
 T1.T2 
T3 P3
S3  S1  CP In  RIn  
T1 P1
270 1
 1.005 In  0.287In
300 4
 0.294 KJ / Kg.K
sgen  1 0.484  1 0.292
 0.786KJ / K
= 0.786 K
Since

Sgen >0, the device is possible.
Such device actually exist and
called Vortex tube.
Q.7 A heat pump operates between two
Q.6 A mass of m kg of fluid at temperate identical bodies. In the beginning ,
me T1 is mixed with an equal mass both the bodies are at same temp T1
of game fluid at temperature𝑇2 . but operation of heat pump cools
Prove that the resultant change of down one of the body to
entropy of universe is temperatureT2. Show that for the
 T1  T2  operation of heat pump the
 
2 
 2 MEln  minimum work input needed by the
T1.T2 heat pump for unit mass if given

© Copyright Reserved by Gateflix.in No part of this material should be copied or reproduced without permission
 T2 
Wmin   1  T2  2T1  . Where C is Solution:
 T2  Q = C dT
specific heat of bodies.
Solution:
Let the final temperature attained
by body to which heat is rejected is
Tf
W = 𝑄𝐵 − 𝑄𝐴
Heat transfer from body A,
QA  C(T1  T2 )
Heat absorbed by body B,
QB  C  Tf  T1  300
Q   0.04T  dT
2
Work input =
QB  QA  C  Tf  T2  2T1 
600
300
 0.04T3 
Now entropy change of body A  
T   3  600
 dSA  C In  2  0.04
 T1   3003  6003 
Entropy change of body B 3
= -2520 kJ
T 
 dSB  C In  f  Now the change of entropy of the
 T1  body
T2 T T
For a reversible cycle entropy dQ 2 CdT 2 0.04T 2dT
change of universe = 0.  ds body     
T T1 T T
 dSuniverse  0
T1 T1

 dSA   dSB  0 0.04


3002  6002   5400J / k
substituting the values, we get C 2 
T  T  Entropy change of sink
ln ln  2   C ln  f   0 Q QW
 T1   T1   dS2  2 
T0 T0
 T .T 
C ln  2 2 f   0 For maximum work considering
 T1   dS Universe = 0
T1  T2 .Tf
 dS1   dS2  0
T2 QW
Tf  1 or 5400  0
T2 T0
Substituting the value of Tf in 2520  W
equation (i) or 5400  0
300
 T12  so W = 900 kJ
Wmin  C   T2  2T1 
 T2 

Q.8 A finite thermal system having heat


capacity C=0.04T2J/K initially at 6oo
K. Estimate the maximum work
obtained from the thermal system if
surrounding temperature is 300 K.

© Copyright Reserved by Gateflix.in No part of this material should be copied or reproduced without permission
4 AVAILABLE ENERGY & THERMODYNAMIC RELATIONS

The second law of thermodynamics tells us The maximum work done obtained in the
that it is not possible to convert all the heat cycle will be the work done in the
absorbed by the system in to the work. reversible cycle.
Only some fraction of heat can be So for the reversible cycle,
converted on the work. This work will be W T
th  1 2
more valuable as compare to the heat. So Q1 T1
Work is high grade energy and heat is the
 T 
low grade energy. W  Q1.th  Q1. 1  2 
Example: High grade energy:  T1 
But the ambient temperature is T0. The sink
1) Mechanical work temperature will be T0.
2) electrical energy  T 
3) water power W max  Q1. 1  0   Q1  T0.  dS
4) wind power  T1 
5) kinetic energy of a jet Where dS is the entropy change of the
6) tidal power. system. The Carnot cycle and the available
energy are shown in figure. The area 1-2-3-
Example: Low grade energy: 4 represents the available energy. Suppose
1) Heat or thermal energy a finite body is used as a source. Let a large
2) heat derived from nuclear fission or number of differential Carnot engines be
fusion. used with the given body as the source.
3) Heat derived from combustion of fossil
fuels.
4) Solar energy.
4.1 AVAILABLE ENERGY
The maximum amount of work that can be
obtained by a system in a cycle is called as
Available Energy (AE).
Suppose a certain quantity of energy Q1 as
heat can be received from a reservoir, at
temperature T1.The sink temperature is T2.
It rejects the heat to the surrounding at
temperature of T2. The maximum possible
work done by the system in the cycle is
known as Available Energy. The maximum fig.: Available and unavailable Energy
possible work can be achieved when the
cycle is reversible cycle. Suppose a finite body is used as a source.
Let a large number of differential Carnot
engines be used with the given body as the
source.
 T 
dW  dQ  dQ 1  0 
 T
If the initial and final temperatures of the
source are T1 and T2 respectively, the total
Fig: heat engine

© Copyright Reserved by Gateflix.in No part of this material should be copied or reproduced without permission
work done or the available energy is given T T 
by   AE 1   AE 2  Q1.  0  0 
T2 T2  T2 T1 
 T0  dQ
w   dQ   dQ 1    Q  To   T T 
T1  T T1
ET  Q1.T0  1 2 
 T1.T2 
or w  Q  T0 | S |
= T0. (dS) – T0. (dS!)
So Decrease in available energy or increase in
 T 
Available Energy  AE   Wmax  Q1. 1  0  unavailable energy
 T1   T -T 
=Q1.T0  1 2   T0 .  dS – T0 .  dS1  ----- (4.3)
= Q1 –T0. (dS) ------(4.1)  T1.T2 
From equation 3 it clears that when same
4.2 UNAVAILABLE ENERGY amount of heat is supplied at the lower
temperature its value is low, because it
The shaded area 4-3-B-A represents the produces the less amount of work as
energy, which is discarded to the ambient compare to work done produced by the
atmosphere and this quantity of energy, heat when it is supplied by higher
cannot be converted into work and is temperature.
called Unavailable energy. So according to first law of
T 
UAE  T0.  dS  Q1  o  . ----- (4.2) thermodynamics heat at higher and lower
 T1  temperature has the same significance. But
according to second law of
4.3 LOSS IN AVAILABLE ENERGY thermodynamics heat at the higher
temperature is more valuable as compare
Suppose a certain quantity of energy Q is to heat at lower temperature.
transferred from a body at constant So,
temperature T1 to another body at  First law of thermodynamic is called as
constant temperature T2 (T2<T1). quantitative law.
Initial available energy, with the body at T1,  Second law of thermodynamic is called
Available Energy as qualitative law.
 T 
 AE 1  Wmax  Q1. 1  0  = Q1 –T0. (dS) 4.4 AVAILABILITY FUNCTION
 T1 
Available energy, with the body at T2, The availability of a given system is defined
Available Energy as the maximum useful work that can be
 T  obtained in a process in which the system
 AE 2  Wmax  Q1. 1  0   Q1 – T0 .  dS!  comes to equilibrium with the
 T2 
surroundings or attains the dead state. It is
given by maximum work done –
atmospheric work.

A) AVAILABILITY FUNCTION FOR NON-


FLOW PROCESS

Let P0 be the ambient pressure, V1 and V0


be the initial and final volumes of the
system respectively. If in a process, the
fig.: loss of available energy
system comes into equilibrium with the
Decrease in available Energy

© Copyright Reserved by Gateflix.in No part of this material should be copied or reproduced without permission
surroundings, the work done in pushing The maximum power that can be obtained
back the ambient atmosphere is P0 (V0-V1). in a steady flow process while the control
Availability  Wuseful  Wmax – P0  V0  V1  . volume exchanges energy as heat with the
ambient at T0:
Consider a system which interacts with the
The steady stet steady flow equation is
ambient at T0. Then from the first law of
given by
thermodynamics
1 1
δQ ̶ δW = Du H1  mC12  mgZ1  Q  H 2  mC22  mgZ2  w cv
2 2
Max work can be obtained when process is
The subscript 1 and 2 refer to the entrance
reversible process.
and exit respectively. At the exit let the
Let S1 and S2 be the entropies of the
system be in equilibrium with the
systems at the entrance and exit of the
environment at pressure Po and
device then (dS)system= S2–S1. and (dS)
temperature To. Let symbols without
surr = - (Q / To)
subscripts refer to the entrance condition
From the entropy principle
of the system and changes in KE and PE are
S2 – S1  –  Q / To   0. negligible. Then useful work is given by
For the reversible process, W = (H2-H1) + Q
S2 – S1  –  Q / To   0 The greater the value of Q larger will be the
useful work. Thus W will be maximum
Therefore Q  To S2  S1 
when Q is a maximum.
Wmax   U1  U0   T0 S1  S0  Let S1 and S2 be the entropies of the
  U1  T0S1  –  U0  T0S0  systems at the entrance and exit of the
device then (dS)system = S2 – S1. and (dS)surr
So Availability = Wuseful = (U1-T0 S1) – (U0-T0 = - (Q / To)
S0)- P0(V0-V1) From the entropy principle
= (U1+ P0V1-T0 S1) – (U0+P0V0-T0 S0) ( S2 –S1 ) – (Q /To) ≥ 0
= ɸ1- ɸ0 For the reversible process,
Availability Function (=U+P0V-T0S---- ( S2 –S1 ) – (Q /To) = 0
(4.4) Therefore Q = To (S2- S1).
Where ɸ = U+P0V-T0S is called the The useful work Wuseful = W = (H2-H1) + To
availability function for the non flow (So-S)
process. Thus, the availability on any given = (H1-ToS1) – (H2 – ToS1)
state: ɸ 1- ɸ 0 So the maximum possible work done
If a system undergoes a change of state Wmax = ψ1 - ψ2
from the initial state 1 (where the Where ψ is known as the availability
availability is (ɸ 1- ɸ 0) to the final state 2 function for steady flow.
(where the availability is (ɸ 2- ɸ 0), the ψ = H - ToS--------- (4.6)
change in the availability or the change in
maximum useful work associated with the 4.5 IRREVERSIBILITY
process, is ɸ 1- ɸ2.
So the work done by the system when The actual work done by a system is always
system undergoes from state 1 to state 2 is less than the idealized reversible work, and
given by the difference between the ideal work and
W12  f1  f 2   U1  P0 V1  T0S1    U2  P0 V2  T0S2  actual work is called the irreversibility of
-------- (4.5) the process.
So irreversibility can be given as
B) AVAILABILITY FUNCTION FOR FLOW I = Wmax – Wact
PROCESS This is also some time referred to as
degradation or dissipation. For a non flow
process between the equilibrium states,

© Copyright Reserved by Gateflix.in No part of this material should be copied or reproduced without permission
when the system exchanges heat only with ii) dH = Tds + VdP
the environment Gibbs function (G) and the Helmholtz
I   U1  U2  – To S1  S2 system –  U1  U2    Qsurr function (F) can be given as
= To (S2-S1) – Q dG = dH – TdS
= To (dS) system + To (dS)surr Gibbs function
= To [(dS)system + (dS)surr ] Hence I> 0. dF = dU –TdS
Similarly for the steady flow process Helmholtz function
I = Wmax – Wact Or these functions can be written as
= To (S2-S1) – Q iii) dF=-PdV - SdT − Helmholtz function
= To (dS) system + To (dS)surr iv) dG = VdP - SdT - Gibbs function
= To[ (dS)system + (dS)surr ]
Thus same expression for irreversibility U, H, F and G are thermodynamic
applies to both flow and non-flow process. properties and exact differential ,so
The quantity To[ (dS)system + (dS)surr ] Appling the Exact differential rule equation,
represents irreversibility. we get Maxwell equations:
I = To(dS)universe = To[ (dS)system + (dS)surr ]  T   P 
     ------------ (4.11)
-------- (4.7)  V s  S  v
 T   V 
THERMODYNAMIC RELATIONS      ------------ (4.12)
 P s  S P
4.6 EXACT DIFFERENTIAL EQUATIONS  P   S 
    ----------- (4.13)
 T V  V T
Theorem No. 1: If a variable Z is a function  V   S 
of the two variable x and y i.e.      ------------ (4.14)
 T P  P T
If Z = f (x , y )
dZ = Mdx + Ndy These four equations are known as
M and N are the function of x and y. Maxwell’s equations.
The differential is exact differential if
4.8 T – DS EQUATIONS
 M   N 
    ---------- (4.8)
 y  x  x  y Let Entropy is function of temperature and
volume S = f (T , V)
Theorem No. 2: if a variable f is the Differentiating the equation
function of the x, y, z and a relation exists  S   S 
i.e. If f = f (x, y,z ) dS    dT    dV
 T V  V T
 x   y   z 
      1 ----------- (4.9) multiplying the equation with temperature T
 y f  z f  x f  S   S 
Tds  T   dT  T   dV
 T V  V T
Theorem No. 3: If a variable z is the We know that
function of x, y i.e.
 S   S   P 
If x = f (y, z ) ------- (4.10) T    CV and    
 T V  V T  T V
4.7 MAXWELL’S EQUATIONS substituting the values in TdS equation, we
get
A pure substance in a single phase has only  p 
Tds  Cv dT  T   dv ------------(4.15)
two independent variables.  T v
According to combined first Law & Second This is known as first T.ds Equation.
Law of thermodynamics
i) dU = Tds – PdV

© Copyright Reserved by Gateflix.in No part of this material should be copied or reproduced without permission
Let the entropy is the function of   V  
temperature T and Pressure P Tds  CP dT  T    dP
  T P 
S = f (T , P)
Differentiating the equation   V  
d  CP dT  T    dP  Vdp
 S   S    T P 
ds    dT    dP
 T P  P T   v  
multiplying the equation with temperature dH  Cp dT   V  T    dP ------- (4.19)
  T p 
 S   S 
Tds  T   dT  T   dP For an ideal gas
 T P  P T
PV = mRT
We know that,
 V  mRT
 S   S   V  T   V
T    CP and        T P P
 T P  P T  T P
Substituting the Value in equation 4.19, we
substituting the values in TdS equation, we get
get
dH  CPdT   V  V dP
 v 
TdS  Cp dT  T   dp --------(4.16) dH = Cv/dT---- (4.20)
 T p
This is known as Second T -dS Equation 4.11 DIFFERENCE IN HEAT CAPACITIES

4.9 ENERGY EQUATION Equating the first and second T- dS


Equation from equation 4.15 and 4.16
From combined first Law & second Law of  V   P 
thermodynamics TdS=CP dT  T   dP  CV dT  T   dV
 T P  T V
dU = TdS – P.Dv
 P   V 
Substituting the first TdS equation in the Or  CP  CV  dT  T   dV  T   dP
internal energy equation,  T V  T P
 P  Or
dU  CV dT  T   dV  P.dV
 T V T
 P   V 
 dV T   dP
or  T V  T P
dT  
  p   CP  CV CP  CV
dU  Cv dT  T    p  dv    (4.17)
  T v  Or
For an ideal gas  T   T 
dT    dV    dP
PV = mRT  V P  P V
 P  mRT  P   V 
or T    P T  T 
 T V V  T V  T   T P   T 
   and  
substituting the value in equation 4.17, we CP  CV  V P CP  CV  P V
get ------------ (4.21)
dU  CV dT   P  P dV    (4.18) From the equation 4.21
dU =CV.dT  P   V 
CP  CV  T    
 T V  T P
4.10 ENTHALPY EQUATION
 P   T   V 
But        1
From combined first and second Law of  T V  V P  P T
2
thermodynamics,  V   P 
dH = TdS + VdP CP  CV  T     ------ (4.22)
 T T  V T

© Copyright Reserved by Gateflix.in No part of this material should be copied or reproduced without permission
It indicates the following facts A gas is made to undergo continuous
 V 
2 throttling process by a valve as shown in
(a) Since   is always positive and fig. Let P1, T1 be the arbitrarily chosen
 T  pressure and temperature before throttling
 P  and P2, T2 are the pressure and
  is negative so CP  CV is always
 V T temperature after the throttling. These are
positive. plotted on T-P diagram.
(b) at absolute temperature (T=OK);
CP  CV
(C ) for an ideal gas
PV  mRT
 V  MR V
   
 T P P T
 P  MRT
  
 V T V2
V 2  MRT  Then the initial temperature and pressure
 C P  C V  T X X 
T2  V2  of the gas are set to new values, a family of
CP  C V  R ------- (4.23) isenthalpic is obtained for the gas as shown
in fig. the locus of the points where μ is
zero is known as Inversion curve. The
4.11.1 VOLUME EXPANSIBILITY  β  region inside the inversion curve when μ is
negative is called as heating region. The
1  V  region inside the inversion curve when μ is
        (4.24) positive is called as cooling region.
V  T p

4.11.2 ISOTHERMAL COMPRESSIBILITY

1  U 
K   T      (4.25)
V  P 
substituting the values from equation 4.24
and 4.25 in equation 4.22,
2
 1  V  
TV   
 V  T P 
CP  CV  The enthalpy change of a gas is given by
1  V 
     V  
V  P T  
dh  Cp dT  T    V  dP
TVB2   T p
 

CP  CV        4.26  dh = 0
K
 T  1   V  

4.12 JOULE KELVIN COEFFICIENT    T    V     (4.28)
 P h Cp    T p 

The slop of an isenthalpic Curve (throttling For an ideal gas,
process) on T-P diagram is known as Joule  V 
Kelvin Co-efficient (μ). T  V 0
 T p
 T 
  ------------ (4.27) So μ=0
 P h So there are some important points:

© Copyright Reserved by Gateflix.in No part of this material should be copied or reproduced without permission
 For an ideal gas Joule Kelvin Co-  717.4KJ
efficient is Zero. For m mass flow rate of gas
 In the cooling region the value of Joule Heat given by gas=heat gain by
Kelvin Co efficient is positive. water
 In the heating region the value of Joule mg cpg  T2  T1   mw  LH 
Kelvin Co efficient is negative.
mg X1.08 1300  320  1X1662.5
 At the inversion curve, value of Joule
Kelvin Co efficient is zero. mg  1.57kg
AE1  1.57  739.16  1161.1KJ
Examples
Loss of Availability = Agas - Aw
Q.1 In a steam generator, water is
= (1161.1-717.4)
evaporated at 26.00 c while the = 443.67 KJ
KJ
combustion gas (CP  1.08 .k) is
kg Q.2 Air expands through a turbine from
cooled from 13000 C to 3200 c . The 500 Kpa, 5200C to 100 KPa ,
surrounding are at 30.00 c . 3000C . During expansion 10 KJ/kg
Determine the loss of available of heat is lost to surrounding which
energy due to the above heat is at 98 KPa , 200C. neglecting K.E.
transfer per kg of water evaporated and P.E. changes, determine per kg
(Latent heat of vaporization at of air (a) the decrease in availability
(b) maximum work (c) the
water at 2600 c  1662.5KJ / kg ).
Irreversibility. For air, take
Solution: cp  1.005KJ / kg.k, h  cpT, where
Availability decrease of gas
AE1  H1  T0S1  (H2  T0S2 ) Cp is constant.
 H1  H2  T0 (S1  S2 ) Solution:
  T  The change of entropy of the air is
 mg cp  T1  T2   T0 mg cp ln  2  
T  P 
  T1   S2  S1  mcp ln  2   mR ln  2 
 T2   T1   P1 
 mg cp  T1  T2    T0 ln   For unit mass of air
 T1  513 1
Given S2  S1  11.005ln ( )  1 0.287 ln  
793 5
T1  1573k, T2  320  273
 0.4619  0.3267
 593kT0  30  273  303k  0.1352KJ / kg k
  593  
AE1  mg X1.08 1573  593  303ln  
Change in availability
  1573  1  2  H1  T0S1   H2  T0S2    H1  H2   T0 (S1  S2 )
 739.16mg KJ  mCP  T1  T2   T0 (S2  S1 )
Availability increase of water  11.005  520 – 300  293  0.1352  260.7 KJ / kg
AE2  Q1  T0ds Maximum work done
 T1ds  T0ds w max  1  2  260.7KJ / kg
  T1  T0  ds But from S.F.E.E
Q  h1  w  h 2
 LH 
  T1  T0  m   [Neglecting K.E. and P.E.]
 T1  w actual  Q  (h1  h 2 )
 303   10  1X1.005(520  300)
 11662.5 1 
 533 

© Copyright Reserved by Gateflix.in No part of this material should be copied or reproduced without permission
 Q  (h1  h 2 ) Let mass of air in the vessel is m kg
211.1 KJ/kg PV
Mass of air (m)  1 1
So, RT1
Irreversibility I  w max  w actual 1400 1

 260.7  211.1 0.287  998
 49.6KJ / kg  10.89kg
Volume of gas after reaching the
Q.3 0.2 kg of air at 3000 c is heated dead state
reversible at constant pressure to mRT0
V0 
2066 k. Find the available and P0
unavailable energies of the heat 10.89  287  298

added. Take TO  300 c and 100
cp  1.0017 KJ/kg.k. V0  9.31m3
Solution: Availability at initial state
Let the 0.2 kg of air is heated at cons AE1  1  2
leant pressure. So entropy change  U1  U0  T0 S1  S0   P0 (V1  V0 )
T  P 
S2  S1  mCP ln  2   mRln  2   mCV  T1  T2   T0
 T1   P1    V1   P1  
 2066   mCP ln    mCV ln     P0  V1  V0 
 0.2 1.0047ln     V0   P0  
 573   m0.718X  448  298  298
 0.2577KJ / K
Increase in available energy of air  1  1400
1.005ln 931  0.718ln 100
AE   H2  H1   T0 S2  S1 
+ 100(1-9.33)
 mCP  T2  T1   303  0.2577
 9.31150  103.52  831
 0.2X1.0047  2066  573  78.084
 1250.14  78.08 Q.5 A heat source at 6270 c transfer heat
 1172.2KJ
at rate of 3000 KJ/min. To a system
Heat Input  mCP (T2  T1 )
maintained at 2870 c . A heat link is
 0.2 1.0047(2066  573) available at 270 c . Assuming these
 1250.24 temperatures to remain constant
Unavailable Energy=Heat Supplied Find:
Available energy i) Change in entropy of source
 1250.24 1172  78.08KJ ii) Entropy production accompanying
heat transfer
Q.4 A pressure vessel has a volume of iii) The original available energy
1m3 and contains air at 1.4mPa , iv) The Available energy after heat
1750 c . The air is coaled to 250 c transfer
by heat transfer to surrounding at Solution:
250 c . Calculate the availability in Temperature of source
the initial and final states and the T1  627 c  627  273  900K
0

irreversibility of this process. Take Temperature of a system


P0  100KPa . T2  2870 c  287  273  560K
Solution: Sink temperature

© Copyright Reserved by Gateflix.in No part of this material should be copied or reproduced without permission
T0  270 c  27  273  300K (ii) Entropy change of mixture
i) Change of entropy of the source Ds=Entropy change of 4 kg of
Q 3000 water+Entropy change of b kg at
ds   1   0.056KJ / Ksec water
T1 900X60
T  T 
ii) Entropy production accompanying  m1cp ln  f   m2cp ln  f 
heat transfer heat transfer by  T1   T2 
source
=Heat absorbed by System  349   349 
 4  4.18ln    6  4.18ln  
T1ds  T2 .ds'  313   373 
T .ds  1.82 1.66  0.15KJ / K
ds'  1
T2 (iii)Unavailable energy with respect
3000 KJ to water at 400 c
  0.089 UAE  T0  ds  system
560X60 Ksec
=29891.82)=542.36KJ
iii) The original available energy
 T   300  Q.7 Using Maxwell’s relations, show that
 Q1 1  0   3000 1   for a pure substance
 T1   900 
TdS  CPdT  TVβdV
 2000KJ / min
Where β is co-efficient of thermal
iv) Available energy after heat expansion, K is co-efficient of
transfer compressibility and CP , CV are
 T   300  specific heat at constant pressure
 Q1 1  0   3000 1   and constant volume respectively.
 T2   560  Solution:
 1392.8KJ / min Let entropy of the pure substance
S  f (P,T)
Q.6 4 kg of water at 400 c mixed with 6
 T   S 
kg of water at 1000 c in steady flow TdS  T   dβ  T   dT _ (1)
process.  P T  T P
Calculate: But
(i) Temperature a resulting mixture  T  T
   Slop of Isobaric process 
(ii) Change in entropy  S P CP
(iii) Unavailable energy with respect
 S 
to energy receiving water at So T    CP
100 c .  T P
Solution: And from Maxwell’s equation
(i) The final temperature at the  S   V 
    
mixture is Tf , then  P T  T P
Heat transfer by 4 kg at water = Substituting the values in equation(1)
Heat transfer by 6 kg at water  V 
m1c  Tf  T1   m2c  T2  Tf  TdS  CP dT  T   dP
 T P
4  4.18  Tf  40   6  9.18 100  Tf  Coefficient of thermal expansion
Tf  40  1.5 100  Tf  B 
1  V 

Tf  1.5Tf  150  10 V  T P
Tf  T60 c
TdS=CPdT-TVβdP

© Copyright Reserved by Gateflix.in No part of this material should be copied or reproduced without permission
And entropy of pure substance again If U  f  T, V 
S  f (T, V)
 U   U 
 S   S  dU    dT    dV
dS    dT    dV  T V  V T
 T V  V T
 U   U   P 
 S   S     CV ,    T   P
TdS  T   dT    dV     2   T v  T T  T  v
 T V  V T For isothermal process
 T  T  dU T  0
   Slop of isocoric process 
 S V CV
 U   P 
 S     T   P
T    CV  V T  T V
 T V But from the relation
And from Maxwell’s Relation RT q
 S   P  P 
    V V2
 T V  T V  P  R
Substituting the values in equation(2)    0
 T V V
 P 
Tds  CV dT  T   dV    (B)  P  TR
 T V T  
 T V V
And
 P  RT q
1  V  1  V  T   P  P  2
K   andβ     T V V V
V  P T V  T P
 U  q
B  P   V  So    2
   .   V T V
K  V T  T P
q
From cyclic property  dU T  2 dV
 P   V   T  V
  .     1 Integrating the equation
 V T  T P  P V 2 2
q

 P   V   P 
 .   
1 dU  1 V2 dV
 V T  T P  T V
1 1
B  P  U 2  U1  q   
So    V1 V2 
K  T V
Substituting the values in equation(3)  dh T   U2  U1    P2V2  P1V1  h 2  h1 
B 1 1
TdS  CV dT  T dV
K
 P2 V2  P1V1   q   
 V1 V2 
Q.8 Derive the expression for (dh)T for a  h2 – h1T   P2V2 – P1V1   q 
substance that obeys the equation of 1 1
  
state given by  V1 V2 
RT q
P 
V V2
Solution:
We know that internal energy of
pure substance
  P  
dU  CV dT  T    P  dV
  T V 

© Copyright Reserved by Gateflix.in No part of this material should be copied or reproduced without permission
5 PROPERTIES OF PURE SUBSTANCE & GAS MIXTURE

5.1 PURE SUBSTANCE  Liquid molecular spacing is comparable


to solids but their molecules can float
A substance that has a fixed chemical about in groups.
composition throughout is called pure  Gas molecules have weakest molecular
substance. Examples-Water, helium carbon bond strength. Molecules in the gas
dioxide, nitrogen Etc. phases are far apart, they have no
It does not have to be a single chemical ordered structure. The molecules move
element just as long as it is homogeneous randomly and collide with each other.
throughout, like air. A mixture of phases of
two or more substance is can still a pure 5.2 P- V DIAGRAM OF PURE SUBSTANCE
substance if it is homogeneous, like ice and
water or water and steam. Let the ice be heated slowly so that its
temperature is always uniform.fig show the
state change of a pure substance (other
than water which contract due to
conversion from ice to water).line passing
through all the saturated solid states is
called the saturated solid line.
The line passing all the saturated liquid
points is called as the saturated liquid line.
The line passing all the saturated Vapour
points is called as the saturated Vapour
line.
Where the two lines (saturated liquid line
and saturated vapour line) meets with each
other is called as critical point.
The triple point on P-V diagram is a line
where all the phases, i.e. solid liquid and
gas phases exist in equilibrium.

(Not a pure substance because the


composition of liquid air is different)

5.1.1 PHASE OF PURE SUBSTANCES

There are three principle phases – solid,


liquid and gas, but a substance can have
several other phases within the principle
phase. Nevertheless, thermodynamics deals
with the primary phases only.
In general Fig.: P-V diagram of pure substance
 Solids have strongest molecular bonds.
Their molecules do not move relative to At the pressure below than triple point
each other. substance can exist in liquid form. The

© Copyright Reserved by Gateflix.in No part of this material should be copied or reproduced without permission
region below the triple point line is solid According to Gibbs phase rule,
vapour mixture region. 2+C=F+P
For Water Where
Critical pressure (Pc) = 221.2 bar C = Number of chemically independent
Critical Temperature (Tc) = 374.150C components
F = Number of degrees of freedom of
5.3 TRIPLE POINT intensive properties
P = Number of phases presented
Triple point is state at which solid, liquid For a pure substance existing in a phase C =
and gas phases exist in equilibrium. On a P- 1, P = 1 so,
T diagram, this condition is a point. But on F = C + 2 – P = 1+2-1 = 2
P-V Diagram it is a line. The curve which It means there are two properties required
shows the phase change of solid into liquid to b3 known to fix up the state of the
or liquid into solid is called as fusion curve. system at equilibrium.
The curve which shows the phase change of For a single component and two phase
liquid into vapour or vapour into liquid is system (i.e. critical point),
called as vaporization curve. C =1, P = 2
F = F = C + 2 – P = 1+2-2 = 1
It means only one property is required to
fix up the state of the system at
equilibrium.
if all three exists in equilibrium (i.e. triple
point) then C = 1, P = 3
F = C + 2 – P = 1+2-3 = 0
This state is unique for a substance. It
means at triple point all the intensive
properties are fixed.

5.5 PHASE CHANGE OF PURE SUBSTANCE

Fig.: Triple point on P- T diagram Thermodynamics deals only with liquid to


gases to generate power. So it is important
The curve which shows the phase change of to understand the conversion of liquid into
solid into vapour directly or vapour into vapour or vice versa.
solid is called as sublimation curve. The Consider water at room temperature
slop of vaporization and sublimation curve (20°C) and normal atmospheric pressure as
is positive. The slop of fusion curve for shown in fig.
most of the substance is positive but for
water is negative.
For water,
Triple point temperature (Tt) = 273.16 K
Triple point pressure (Pt) = 0.6113 KPa

5.4 GIBBS PHASE RULE

Gibbs phase rule describes the number of


degrees of freedom, or the number of Fig. formation of steam
variables that must be fixed to specify the The water is in liquid phase and it is called
composition of a phase. compressed liquid or sub cooled liquid.

© Copyright Reserved by Gateflix.in No part of this material should be copied or reproduced without permission
Now heat is supplied to water, its given pressure process of phase change is
temperature will increase. Due to the shown T-S diagram.
increase in temperature, the specific
volume v, specific enthalpy h and specific
entropy s will increase. As a consequence,
the piston will move slightly upward
therefore maintaining constant pressure.
Now if continue heat is added to the water,
the temperature will increase further until
100°C. At this point, any additional addition
of heat will not increase the temperature of
water; it will utilise to vaporize some
water. This specific point where water
starts to vaporize is called saturated
liquid point. And the condition of the Fig.:T-S diagram representing phase
liquid is called as saturated liquid. change for water at constant pressure
If we continue to add heat to water, more Let us consider the different pressures and
and more vapour will be created, while the repeat process of phase change. On the
temperature and the pressure remain different pressure we get the different
constant (T=100°C and P=1 atm). These saturated liquid point and saturated
conditions will remain the same until the vapour point. The locus of all saturated
last drop of liquid is vaporized. At this liquid points is called as saturated liquid
point, the entire cylinder is filled with curve. And the locus of all the saturated
vapour at 100°C. This state is called vapour points at different pressure is called
saturated vapour. as saturated vapour curve.
The state between saturated liquid and Where the saturated liquid and vapour
saturated vapour where two phases exist is curve meet with each other is called as
called saturated liquid-vapour mixture or Critical Point. The same process is shown
wet vapour. on T-s and P-s diagram.
After the saturated vapour phase, any
addition of heat will increase the
temperature of the vapour; this state is
called superheated vapour.
This concept can be applied to pure
substance other than water.

5.6 T-S DIAGRAM OF PURE SUBSTANCE

During the phase change of a pure


substance as discussed in the previous
topic first addition of heat is utilized to
increase the temperature end entropy of
the system. And then further addition of
heat is to change the phase of liquid in
which only entropy change temperature
remains constant. But further addition of
heat is to superheat the steam in which
temperature and entropy both change. At a Fig. T-s diagram of pure substance

© Copyright Reserved by Gateflix.in No part of this material should be copied or reproduced without permission
1-2 represents the sensible heat of water at
1 atmospheric pressure. At 1000C saturation
point is attained.

Latent Heating

The addition of heat which is utilized to


phase change is called as latent heating. In
process 2-4, heat is added to water that is
utilised to convert the phase of liquid in to
vapour and it is called as latent heating. the
latent heat decreases as pressure increases
and at the critical point latent heat of
vaporization is zero.
Fig. P-v diagram of pure substance
Critical Point
5.7. PROPERTIES OF PURE SUBSTANCE
It is point at which liquid directly converts
Saturation Temperature into vapour phase. At the critical point,
latent heat is zero. For water at the critical
The temperature at which water starts point,
boiling depends on the pressure. In other Pcr = 220.8 bar
words, water starts boiling at 100 ºC but Tcr = 374.140C
only at 1 atm. At different pressures, water
boils at different temperatures. Degree of Sub cooling

Saturation Pressure If the actual temperature of liquid is less


than the saturated temperature of liquid at
At a given pressure, the temperature at given pressure the liquid is called as sub
which a pure substance changes phase is cooled liquid. The difference of saturated
called the saturation temperature (Tsat). temperature and actual temperature of
Likewise, at a given temperature, the liquid is called as degree of sub cooling. Let
pressure at which a pure substance the saturated temperature is Tsat. and actual
changes phase is called the saturation temperature is Tsub.,then
pressure (Psat). Degree of sub cooling = Tsat -Tsub

Sensible Heating Degree of Sub cooling

If the actual temperature of vapour is


greater than the saturated temperature of
vapour at given pressure the vapour is
called as superheated vapour. The
difference of actual temperature of vapour
and saturated temperature is called as
degree of superheating. Let the saturated
The addition of heat which is utilised to temperature is Tsat. and actual temperature
increase the temperature is called as is Tsup., then
sensible heating. In T- S diagram, Process Degree of superheating =Tsup - Tsat

© Copyright Reserved by Gateflix.in No part of this material should be copied or reproduced without permission
Wet Vapour compression refrigeration system. In most
of the thermodynamic problem, these
Wet vapour is mixture of liquid and vapour important phases can be existed. So it is
phase. in wet vapour region liquid and important to determine the parameters in
vapour phase exists in equilibrium. different region of pure substance.
Dryness Fraction or quality of mixture (x)
1) Specific Volume, enthalpy and
If vapour is made wet by liquid droplets in entropy of saturated liquid
suspension, it is important to know the
degree of wetness. Any mass of wet In the T-S and P-h diagram it is shown by
consists of dry saturated liquid and dry point 1. Enthalpy of this point is the heat
saturated vapour. supplied to increase the temperature from
The ratio of mass of dry saturated vapour 00 C to saturated temperature at given
to total mass of water vapour is known as constant pressure. these all parameters can
dryness fraction (x). It is also known as be found Corresponding to saturated temp.
quality of steam. at given pressure with help of stem table.
Let the mass of dry saturated vapour is mv Consider specific enthalpy, entropy and
and the mass of dry saturated liquid in wet volume of saturated liquid at a given
pressure and temperature are hf, sf and vf.
vapour is m l , then dryness fraction
specific enthalpy, entropy and volume of
mv saturated vapour at a given pressure and
x
ml  m v temperature are hg, sg and vg.
Then latent heat of vaporization
hfg = hg - hf
specific volume Vfg = Vg - vf , Specific
entropy Sfg = Sg - Sf
From steam table at given pressure
Enthalpy of saturated liquid
h1 = hf
Entropy of saturated liquid
s1 = sf
Important points Volume of the saturated liquid
 At saturated liquid line mass of vapour v1 = vf
is zero so value of x is zero along
saturated liquid line.
 At saturated vapour line mass of liquid
is zero, so dryness fraction is 1. So
dryness fraction varies from 0 to 1.
 All dryness fraction lines converge at
original point.

5.8 SPECIFIC VOLUME, ENTHALPY AND Fig. T- S diagram


ENTROPY OF DIFFERENT PHASES

The specific enthalpy entropy and specific


volume are very important parameters to
define the performance of any power
producing system such as steam power
plant working on Rankin cycle and power
absorbing system such as vapour Fig. P- h diagram

© Copyright Reserved by Gateflix.in No part of this material should be copied or reproduced without permission
T 
2) Specific Volume, enthalpy & entropy S5  Sg  cp ln  5 
of sub cooled liquid  T4 
Volume of superheated vapour
In the T-S and P-h diagram it is shown by T 
point 2. v3= vg.  5 
Enthalpy of subcooled liquid  T4 
h 2  h f  cp  T1  T1  , PROPERTIES OF GAS MIXTURE
Entropy of subcooled liquid 5.9 EQUATION OF STATE
T 
S5  Sf  cp ln  1  The ideal gas equation of state is P.v = RT
 T2  can be established for the real gas with two
Volume of subcooled liquid can be found at important assumptions of the ideal gas. The
particular temperature at the given ideal gas assumptions are:
pressure from the steam table. a) There is no attraction between the
molecules of the gas.
3) Specific Volume, enthalpy and
b) The volume occupied by the molecules
entropy of saturated vapour
is negligible.
In the T-S and P-h diagram it is shown by When the pressure is very low and
point 4. temperature very large, the intermolecular
From steam table at given pressure attraction and volume of molecules has no
Enthalpy of saturated vapour more significance. As the pressure of gas
h4  hg , increases, the intermolecular force forces
Entropy of saturated vapour of attraction and repulsion increases and
s4  sg also volume of the molecules becomes
appreciable compare to total volume of the
Volume of saturated vapour gas.
v4 = vg
5.9.1 VANDER WAALS EQUATION
4) Specific Volume, enthalpy and
entropy of wet vapour Vander Waals, by applying the law of
mechanism of individual molecule
In the T-S and P-h diagram it is shown by introduced two correction terms in ideal
point 3. gas equation. This is valid for the real gases.
Enthalpy of wet vapour  
h 3  h f  x.h fg ,  P  2  (v  b)  RT
 v 
Entropy of wet vapour Where a and b are Vander Waals constant.
LH a is introduced to account the existence of
s3  sf  xsfg  sf  x
T mutual attraction between molecules. The
Volume of wet vapour term a/v2 is called as force due to cohesion.
v3= x.vg The term b is introduced to account the
5) Specific Volume, enthalpy and volume of molecules is known as co –
entropy of superheated vapour volume.

In the T-S and P-h diagram it is shown by 5.9.2 PROPERTIES OF CRITICAL POINT
point 5.
Enthalpy of superheated vapour According to Vander Waals equation,
h 5  h g  cp  T5  T4   a 
Entropy of superheated vapour  P+ 2   v-b  =RT or
 v 

© Copyright Reserved by Gateflix.in No part of this material should be copied or reproduced without permission
RT a 5.9.3 COMPRESSIBILITY FACTOR
P  2 -------- (1)
(v  b) v The ideal gas equation can be given as
Pv = RT
If the gas is compressible (real gas), then
pv
Z
RT
Z > 1 or Z < 1

5.9.4 COMPRESSIBILITY FACTOR AT


Slop of P-v diagram at critical point is zero, CRITICAL POINT
so differentiating the equation with respect At the critical point the compressibility
to v and putting equal to zero. factor is given by
 dp  RT 2a PV
0    3 Zc  c c
 dv c  v  b 
2
v RTc
RT 2a Substituting the value of P, V and T at
 -------------- (II)
 vc  b 
2
v3c critical point, we get
a
critical point is not only zero slop curve but X3b
2 3
at this point slop change so second Zc  27b 
derivative of P with respect to v is also 8a 8
RX
zero. Differentiating the equation II and 27b R
putting equal to Zero, 3
Zc   .375
 d2p  2RT 6a 8
 2  0  4
 vc  b  vc
3
 dv c 5.10 PROPERTIES OF GAS MIXTURE
RT 3a 5.10.1 DALTON’S LAW OF PARTIAL
 ----------------- (III)
 vc  b 
3
vc4 PRESSURES
Now dividing the equation II by equation Let consider a homogeneous mixture of
III, we get inert ideal gas at pressure P, Volume V and
2
 vc  b   vc temperature T. Let us suppose there are n1
3 numbers of molecules of gas A, n2 number
Vc = 3b of molecules of gas B and n3 number of
Substituting the value of vc in equation III, molecules of gas C.
we get
2a  2b 
2
8a
RT  3

27b 27b
8a
Tc 
27bR
Substituting the value of Tc and vc in
equation I, we get
8a
RX
P 27bR  a
2b 9b 2
a
pc 
27b 2
fig. gas mixture

© Copyright Reserved by Gateflix.in No part of this material should be copied or reproduced without permission
Then the equation of state for mixture of In the same way internal energy, enthalpy
the gas is and specific heats are given as
PV = (n1+n2+n3) ṜT
Ṝ = universal gas constant = 8.314 KJ/ kg Specific Internal Energy of the mixture
mol K
Partial pressure of gas A, B and C can be m1u1  m2 u 2  m3u 3
Ue 
written as m1  m2  m3
n R.T n R.T n R.T
P1 = 1 ,P2 = 2 ,P3 = 3
V V V Specific enthalpy of the mixture
According to Dalton law of partial pressure
P=P1  P2  P3 m1h1  m2 h 2  m3h 3
He 
n1 R.T n 2 R.T n 3 R.T m1  m2  m3
P= + +
V V V
Specific heat at constant pressure of the
n R.T
or P= K mixture
V
where, m1Cp1  m2Cp 2  m3Cp3
n k  n  n1  n2  n3 Cp 
m1  m2  m3

5.10.2 MOLE FRACTION Specific heat at constant volume of the


mixture
The ratio of number of mole of a given gas
to the total number of moles in the given m1Cv1  m2Cv2  m3Cv3
mixture of ideal gas is known as mole Cve 
m1  m2  m3
fraction (x).
n n n
x1 = 1 , x2 = 2 and , x3 = 3 Examples
n n n
Q.1 Determine the specific enthalpy of
Equivalent Gas constant (Re) steam at 2 MPa and with a
temperature of 2500C.
Ideal gas equation for the gas mixture in Solution:
the term of masses can be written as From the steam table
For gas A in the gas mixture At 2 MPa , Tsat  212.40 C
P1V = m1R1T
It must be the condition of super
For gas B in the gas mixture
heating because the temperature of
P2V = m2R2T
steam is higher than saturated
For gas C in the gas mixture
steam.
P3V = m3R3T
So Degree of superheating
Using Dalton law of partial pressure,
Tsup -Tsat =250-212.4=37.6K
PV = P1V+ P2V+ P3V
or PV = m1R1T + m2R2T + m3R3T Specific enthalpy of steam at 2MPa
PV = (m1R1 + m2R2 + m3R3)T-----------(I) is given
Ideal gas equation for a gas mixture, h g  2797KJ / kg
PV = (m1 + m2 + m3).Re.T------------(II) So enthalpy
Comparing the equation I and II, we get h  h g  cp  Tsup  Tsat 
m R  m 2 R 2  m3 R 3
Re  1 1 = 2797.2 + 2.09 × 37.5
m1  m2  m3
= 2875.9 KJ/kg

© Copyright Reserved by Gateflix.in No part of this material should be copied or reproduced without permission
Q.2 Steam 0.95 dry at a pressure of 0.7 Enthalpy gained by water =
MPa is supplied to a heater through Enthalpy lost by steam
a pipe of 25mm internal diameter. m  376.8  79.8  81.9(2658.8  376.8)
The velocity in the pipe is 12 m/sec. 81.9  2282
water enters the heater at 190C , the  
m
steam is blow into it and the mixture 297
  629.28 kg / hr.
m
of water and condensate leaves the
heater at 900C. Calculate:
a) mass of steam entering the heater Q.3 1.5 kg of steam originally at
b) mass of water entering the heater pressure of 1 MPa and temperature
Properties of steam are: 2250C is expended until the
pressure becomes 0.28 MPa. The
dryness fraction at the steam is 0.9.
Determine the change of internal
energy.
Solution:
At 1 MPa and 2250C, from steam
Solution: table
Specific volume of dry (0.95) at pr.
h1 =2886KJ/kg,v1 =0.2198m3 /kg
0.7 MPa is
V1  xvg  0.95  0.273 1X106
So u1 =h1 -p1v1 =2886- X0.2198
= 0.259 m3 /kg 103
a) Steam volume passing/sec = 2886 – 219.8
π = 2566.2 KJ/kg
 d 2l At 0.28 MPa and dryness fraction 0.9
4
h 2  h f 2  xh fg2
π
   25 103  12
2
= 551.4 + 0.9 X 2170.1
4
= 551.4 + 1953.09
Steam volume passing /
= 2504.49 KJ/kg
π
h r    0.025 12  3600
2
Volume
4 v2  xvg2  0.9  0.646
mass of steam entering / hr
π = 0.581 m3 / kg
  0.025 12  3600
2
Internal energy u 2  h 2  p2 v2
 4
0.259   2504.49 – (0.28 103  0.5814)
= 81.9 kg/hr = 2504.49 – 162.8
= 2341.69 KJ/kg
b) Specific enthalpy of steam Hence change of internal energy
entering heater is
U2  U1  m  u 2  u1 
h 2  h f  xh fg  697.1  0.95  2064.9
 1.5   2341.69 – 2566.2 
= 697.1 + 1961.7
= 2658.8 KJ / kg = - 489.77 KJ
From the steam table at 900C
h f  376.8KJ / kg Q.4 The dryness fraction of steam at a
pressure of 2.2 MPa is measured
And at 190C
using throttling calorimeter. After
h f  79.8KJ / kg
throttling, the pressure in the
So for the heater applying energy calorimeter is 0.13 MPa and the
balance equation temperature is 1120C. Determine

© Copyright Reserved by Gateflix.in No part of this material should be copied or reproduced without permission
the dryness fraction of steam at 2.2 Determine the dryness fraction of
MPa. steam after throttling.
Solution: - Solution:
From the steam table For the throttling process
h1  h 2
h f 1  x1h fg1  h f 2  x 2 h fg2
830.1+0.7×1957.7=928.8+X2+2250.8
1771.7
Property of Superheated steam x2 
2250.8
Specific x 2  0.787
Pr Sat.temp enthalpy kJ/kg
 The steam becomes drier in this
MPa (0C) hg At 1500C
case.
0.1 99.6 2675 2777
0.5 111.4 2693 2773
Q.6 a) Determine the volume occupied
Enthalpy of this stem after throttling by 1 kg of steam at a pr. Of 0.85
can be found with help of linear MPa and having a dryness
interpolation fraction of 0.97 .
b) This volume is expanded
adiabatically to a pressure of
0.17 MPa, law of expansion is
PV1.3 = C. Determine
i) Final dryness fraction of the
steam
ii) Change of internal energy of
system during the expansion.
h 2 at the temp. of 1120C can be Solution:
found by linear interpolation Properties of system from steam
Tsup  Tsat table
h 2  h g   h150  h g  At 0.85 MPa , vg = 0.2268 m3/kg
Tsup  Tsat
a) So
=2685.2+(2774.6-2685.8)
v1 =x1vg1 =0.97X0.2268=0.22m3 /kg
(112  106.68)
(150  106.68)
b) (i) Final dryness fraction
= 2685.2 + 10.9 = 2696.1 KJ/kg
P1V11.3  P2 V21.3
Now for the wet steam before steam
1
throttling  0.85 1.13
h1 =h f1 +x1h fg1 V2  0.22  
 0.17 
= 931 + x1 X 1870 KJ/kg V2  0.22  4.15
For the throttling process
h1  h 2 = 0.913 m3/kg
At 0.17 MPa ,
931+(x1×1870) =2696.1 vg2 = 1.031 m3/kg
1765.1 The steam is wet, so dryness
x1 
1870 fraction is
x1  0.94 u 0.913
x2  2   0.886
u g2 1.031
Q.5 Steams at 1.4 MPa and dryness
fraction 0.7 is throttled to 0.11 MPa. (ii) For the adiabatic expansion
δQ = 0

© Copyright Reserved by Gateflix.in No part of this material should be copied or reproduced without permission
So change of internal energy (c).Equivalent gas constant of the
U2  U1  W mixture gas constant of gas 1
R 8.314
U 2  U1  
 P1V1  P2 V2  R1    0.19 KJ / Kg.K
M1 44
n 1


 0.85  0.22  0.17  0.913 103 R1 
R 8.314
  0.30 KJ / Kg.K
1.13  1 M2 28
 246 KJ / kg m1R1  m2 R 2 44  0.19  28  0.30
Re    0.22 KJ / Kg.K
m1  m2 44  28
There is a loss of internal
energy.
(d) The Equivalent specific heat at
Q.7 A mixture of ideal gases consisting constant pressure
R 0.19
of 3 kg of nitrogen and 5 kg of Cv1  1   0.66 KJ / Kg.K
carbon monoxide at a pressure of γ  1 1.286  1
300 KPa and temperature of 200C . CP1  γCV1  1.286  0.66
Find (a) the mole fraction of = 0.85 KJ/Kg.K
constituent (b) Equivalent R 0.30
molecular weight of the mixture CV2  2   0.75 KJ / Kg.K
γ  1 1.4  1
(c) Equivalent gas constant of the
mixture, (d) the specific heat at CP2  γCV 2  1.4  0.75
constant pressure and volume of the = 1.05 KJ/Kg.K
mixture. Take γ  1.286for CO2 and The equivalent specific heat of
γ  1.4forN2 . mixture at constant pressure
m1Cp1  m2Cp2
Solution: Cpe 
let the gas CO2 as the gas 1 and gas m1  m2
44  0.85  28 1.05
N 2 as the gas 2 is mixture.   0.928 KJ / Kg.K
44  28
The mole fraction for CO2
The equivalent specific heat of
n m mixture at constant volume
x1  1 and n1  1
n M1 m C  m2CV2
Cve  1 V1
m1 5 m1  m2
M1 44  0.66  28  0.75
x1   44  0.52 
m1 m 2 5 3 44  28
 
M1 M 2 44 28 = 0.695 KJ/Kg.K
Similarly the mole fraction for N 2
Q.8 from an experimental determination
m2 3 the specific heat ratio for acetylene
M2 C2 H2 is found to 1.26. Find the two
x1   28  0.48
m1 m 2 5 3 specific heats. If heat is supplied to
 
M1 M 2 44 28 increase the temperature from 300 C
(b) Equivalent molar weight of the to 600 C at constant pressure then
mixture find the increase in the enthalpy and
Me = x1M1 + x2M2 entropy of the system.
= 0.52×44 +0.48 × 28 = 36.32 Kg/Kg Solution:
mole Gas constant of acetylene
R 8.3143
 C2 H2  (R)   kJ / kgK
M 26

© Copyright Reserved by Gateflix.in No part of this material should be copied or reproduced without permission
2
= 0.3198 kJ/kgK
As adiabatic index (γ) = 1.26, then  W12   pdV  p  V2  V1   27.266kJ
1
γ
We know that cp  R
γ 1 (b) Heat transferred
1.26 KJ Q12  u 2  u1  W12
  0.3198  1.55 K
1.26  1 Kg  mcv  T2  T1   W12  95.476kJ
R
And c v 
γ 1 (c) Entropy change of the gas
0.3196 V p
  1.23KJ / kgK S2  S1  mcp In 2  mcv In 2
1.26  1 V1 p1
Change in enthalpy is given by V kJ
h 2  h1  cp  T2  T1   mcp In 2  0.29543 .K
V1 kg
KJ
 1.55  30  4.65
Kg
Change in enthalpy is given by
 T2   P2 
S2  S1  cp ln    R ln  
 T1   P1 
 T2 
 cp ln  0
 T1 
 T2 
 Cp ln  
 T1 
 333 
 1.55  ln  
 303 
= 0.146 KJ/Kg.K

Q.9 One kg of air in a closed system,


initially at 50C and occupying 0.3 m3
volume, undergoes a constant
pressure heating process to 1000C.
There is no work other than Pdv
work. Find (a) the work done during
the process, (b) the heat transferred,
and (c) the entropy change of the
gas.
Solutions:
T1 = 278 K
V1 = 0.3 m3
M = 1 kg
P1 = 265.95 kPa
T2 = 1000C = 373 K
P2 = 265.95 kPa
mRT2
 V2   0.40252m3
P2
(a) Work during the process

© Copyright Reserved by Gateflix.in No part of this material should be copied or reproduced without permission
6 REFRIGERATION

6.1 INTRODUCTION 6.1.3 REFRIGERANT


In generally, refrigeration is defined as any
process of heat removal. Refrigeration is In the refrigeration process, the substance
defined as branch of Science that deals with employed as the heat absorber or coaling
the process of reducing and maintaining agent is called the refrigerant. When the
temperature of a space or material below absorbed heat causes an increase in
the temperature of the surrounding. temperature of refrigerant, cooling process
We see in nature that the heat is said to be sensible, whereas when the
spontaneously flows of heat from a high absorbed heat causes a change in physical
temperature body to a low temperature state of refrigerant, cooling effect is said to
body. The reverse process to complete the be latent. Ice has been used successfully for
thermodynamic cycle, in which heat Q will many years as a refrigerant. Not to many
flow back from low temperature body to years ago ice was the only cooling agent
high temperature body, is not possible. So available for use in domestic and small
the logical conduction is that there must be commercial refrigerator.
a process in which some work is done.
6.1.4 UNIT OF REFRIGERATION
6.1.1 NEED OF THERMAL INSULATION
It is given by Tonnes of Refrigeration (TR).
Since heat well always flow from a region 1TR is defined as refrigeration effect
of high temperature to a region of lower produced by uniform melting of 1000 kg of
temperature. There is always a continuous ice from and at 00C in 24 hours.
flow of heat into the refrigerated region 1 TR = 1000×334 KJ refrigeration effect in
1000  334
from the warmer surroundings. To limit 24 hours   3.86 KJ / Sec.
the heat flow into refrigerated region, it is 24  3600
usually necessary to isolate the region from  For practical problem it is taken as 3.5
its surrounding with a good insulation. KJ/Sec or 210 KJ/Minute.

6.1.2 REFRIGERATION LOAD 6.1.5 CO EFFICIENT OF PERFORMANCE


The rate at which heat must be removed
from refrigerated space or material in The co efficient of performance (COP) of
order to produce and maintain the desired refrigerator is ratio of heat extracted in the
temperature condition is called as refrigerator to the work done on the
refrigeration load or cooling load. refrigerant working for the refrigeration
The total refrigeration load is the sum of cycle.
heat gains from different sources:- Heat extracting from refrigerated space Q
COP  
(1) Heat transmitted by conduction Work Done W
through the insulated walls.
(2) Heat that must be removed from warm 6.2 AIR STANDARD REFRIGERATION
air that enters the space through CYCLE
openings and closing doors.
(3) Heat that must be removed from Air cycle refrigeration systems belong to
refrigerated product to reduce the the general class of gas cycle refrigeration
temperature of products to the storage systems, in which a gas is used as the
temperature. working fluid. The gas does not undergo

© Copyright Reserved by Gateflix.in No part of this material should be copied or reproduced without permission
any phase change during the cycle; low temperature TL during isothermal
consequently, all the internal heat transfer expansion.
processes are sensible heat transfer
processes. Process 1-2: Isentropic compression of the
 Gas cycle refrigeration systems find working fluid with the aid of external work.
applications in air craft cabin cooling and The temperature of the fluid rises from TL
 It also in the liquefaction of various gases. to TH.

6.2.1 AIR STANDARD CYCLE ANALYSIS Process 2-3: Isothermal compression of


the working fluid during which heat is
Air cycle refrigeration system analysis is rejected at constant high temperature TH.
considerably simplified if one makes the
following assumptions: Process 3-4: Isentropic expansion of the
i. The working fluid is a fixed mass of air working fluid. The temperature of the
that behaves as an ideal gas working fluid falls from TH to TL.
ii. All the processes within the cycle are
reversible, i.e., the cycle is internally
reversible.
iv. The specific heat of air remains
constant throughout the cycle.
First of all we study the ideal refrigeration
cycle i.e. Reversed Carnot Cycle.

6.2.2 A REFRIGERATOR WORKING ON


REVERSED CARNOT CYCLE:

Reversing the Carnot cycle does reverse the Fig. P-V and T-S Diagram of Reversed
directions of heat and work interactions. A Carnot Cycle
refrigerator or heat pump that operates on
the reversed Carnot cycle is called a Carnot  The reversed Carnot cycle is the most
refrigerator or a Carnot heat pump. efficient refrigeration cycle operating
Reversed Carnot cycle is shown in Fig. It between two specified temperature
consists of the following processes. levels. It sets the highest theoretical
COP. The coefficient of performance for
Carnot refrigerators and heat pumps
are:

COP of Refrigerator

Heat absorbed
COPR  
Work supplied
Heat absorbed
Heat rejected  Heat absorbed
Fig. a refrigerator working on reversed QL TL (dS)
 
Carnot Cycle Q H QL TH(dS) TL (dS)
So,
Process 4-1: Absorption of heat by the QL TL
working fluid from refrigerator at constant COPR   -----(7.1)
QH  QL TH  TL

© Copyright Reserved by Gateflix.in No part of this material should be copied or reproduced without permission
COP of Heat Pump There are four important processes in
reversed Bray ton Cycle:
Heat Rejected
COPHP  Process 1-2: Reversible adiabatic
Work supplied compression in a compressor
Heat Rejected Process 2-3: Reversible isobaric heat

Heat rejected  Heat absorbed rejection in a heat exchanger
QH TH (dS) Process 3-4: Reversible adiabatic
  expansion in a turbine
Q H QL TH(dS) TL (dS)
Process 4-1: Reversible isobaric heat
QH TH absorption in a heat exchanger
So COPHP   --(7.2)
QH  QL TH  TL All the Processes are on P-V and T-S
From the equation 7.1 and 7.2 diagram.
COPHP  COPR  1 ------------- (7.3)
 Notice that turbine is used for
expansion process between higher and
lower temperature limit. Work done by
the turbine helps supply some of the
work required by the compressor from
the external source.
 Practically, the reversed Carnot cycle
cannot be used for refrigeration Fig. Reversed Bray ton cycle on T-S diagram
purpose as the isentropic process
requires very high speed operation, Process 1-2: Gas at low pressure is
whereas the isothermal process compressed isentropic ally from state 1 to
requires very low speed operation. state 2.
Applying steady flow energy equation and
6.3 REFRIGERATOR WORKING ON neglecting changes in kinetic and potential
REVERSED BRAY TON CYCLE energy,
W1-2 = m (h2-h1) = m. Cp. (T2-T1)
This is an important cycle frequently But process 1-2 is reversible adiabatic
employed in gas cycle refrigeration Process or isentropic process. So,
systems. This may be thought of as a γ 1
modification of reversed Carnot cycle, as T2  P2  γ γ 1

the two isothermal processes of Carnot      rp  γ    (7.4)


T1  P1 
cycle are replaced by two isobaric heat
Where rp = (P2/P1) = pressure ratio
transfer processes. This cycle is also called
as Joule or Bell-Coleman cycle. Figure
shows the schematic of a closed reverse Process 2-3: Hot and high pressure gas
Bray ton cycle and also the cycle on T-s flows through a heat exchanger and rejects
diagram. heat sensibly and isobaric to a heat sink.
The enthalpy and temperature of the gas
drop during the process due to heat
exchange, no work transfer takes place and
the entropy of the gas decreases.
Q2-3 = m (h2-h3) = m. Cp. (T2-T3)

Process 3-4: High pressure gas from the


heat exchanger flows through a turbine
Fig. Reversed Bray ton Refrigeration System undergoes isentropic expansion and

© Copyright Reserved by Gateflix.in No part of this material should be copied or reproduced without permission
delivers net work output. The temperature 1
So, COP 
 rp    1  1
of the gas drops during the process from T3
to T4. From steady flow energy equation:
W3-4 = m (h3-h4) = m. Cp. (T3-T4) From the above expression for COP, the
But process 3-4 is reversible adiabatic following observations can be made:
Process or isentropic process. So,
γ 1
T3  P3  γ γ 1
     rp  γ ------------- (7.5)
T4  P4 
Where rp = (P3/P4) = pressure ratio
T3 T2
So, 
T4 T1

Process 4-1: Cold and low pressure gas


from turbine flows through the low Fig. Comparison between Reversed Bray
temperature heat exchanger and extracts ton Cycle and Carnot Cycle.
heat sensibly and isobaric ally from a heat
source, providing a useful refrigeration  For fixed heat rejection temperature
(T ) and fixed refrigeration temperature
effect. The enthalpy and temperature of the 3
gas rise during the process due to heat (T1), the COP of reverse Bray ton cycle
exchange, no work transfer takes place and is always lower than the COP of reverse
the entropy of the gas increases. The Carnot cycle as shown in fig.
refrigerating effect by the refrigerant can  COP of Bray ton cycle approaches COP
be given as: of Carnot cycle as T1 approaches T4

6.3.1 REFRIGERATING EFFECT (thin cycle), however, the specific


refrigeration effect [cp(T1-T4)] also
Q4-1 = m (h1-h4) = m. Cp. (T1-T4)
reduces simultaneously.
COP of reversed Bray ton cycle can be
 COP of reverse Bray ton cycle decreases
given:
as the pressure ratio rp increases.
COP   Refrigerating Effect  /  NetWork Done 
Q41 6.3.2 ACTUAL REVERSE BRAY TON CYCLE

W12  W34
The actual reverse Bray ton cycle differs
m.cp. (T1 T4 ) from the ideal cycle due to:

m.cp. (T2 T1 )  m.cp. (T3T4 )  Non-isentropic compression and
(T1 T4 ) expansion processes.
  Pressure drops in cold & hot heat exchangers.
(T2 T3 )  (T1 T4 )
Solving the equation
 T 
T1 1  4 
  T1 
 T   T 
T2 1  3   T1 1  4 
 T2   T1 
T1 1 1
   γ 1
T2  T1 T2
 1  rp  γ  1
T1 Fig. Actual Reversed Bray ton Cycle

© Copyright Reserved by Gateflix.in No part of this material should be copied or reproduced without permission
Figure shows the ideal and actual cycles on operating cycle on T-S diagram. This is an
T-s diagram. Due to this irreversibility, the open system. As shown in the T-S diagram,
compressor work input increases and the outside low pressure and low
turbine work output reduces. temperature air is compressed due to ram
If ηcompressor ,isen and ηturbine ,isen are the effect to ram pressure in the process 1-2.
isentropic efficiencies of compressor and During this process its temperature
turbine, respectively, these are defined as: increases from T1 to T2. This air is
h h T T compressed in the main compressor in
Copm.  2' 1  2' 1 process 2-3, and is cooled to temperature
h 2  h1 T2  T1
T4 in the air cooler. Its pressure is reduced
h  h4 T T to cabin pressure in the turbine, as a result
Turbine  3  3 4'
h 3  h 4 T3  T4
'
its temperature drops from T4 to T5. The
Thus the net work input increases due to cold air at stateT5 is supplied to the cabin. It
increase in compressor work input and picks up heat as it flows through the cabin
reduction in turbine work output. The providing useful cooling effect. The power
refrigeration effect also reduces due to the output of the turbine is used to drive the
irreversibility’s. As a result, the COP of fan, which maintains the required air flow
actual reverse Bray ton cycles will be over the air cooler.
considerably lower than the ideal cycles.  This simple system is good for ground
Design of efficient compressors and cooling.
turbines plays a major role in improving
the COP of the system.
In practice, reverse Bray ton cycles can be
open or closed.
6.3.3 AIRCRAFT COOLING SYSTEMS
An aircraft, cooling systems are required to
keep the cabin temperatures at a
comfortable level. Even though the outside fig: Simple air craft refrigeration system
temperatures are very low at high
altitudes, still cooling of cabin is required
T2' γ 1 2
due to:  1 M
 Large internal heat generation due to T1 2
occupants, equipment etc. Where M is the Mach number, which is the
 Heat generation due to skin friction ratio of velocity of the aircraft (V) to the
caused by the fast moving aircraft sonic velocity C
 At high altitudes, the outside pressure V  V 
will be sub-atmospheric. When air at Mach number: M    
C   RT1 
this low pressure is compressed and
supplied to the cabin at pressures close 6.4VAPOUR COMPRESSION REFRIGERATION
to atmospheric, the temperature CYCLE
increases significantly.
Vapour compression cycle is an improved
 Solar radiation
type of air refrigeration cycle in which a
suitable working substance, termed as
6.3.4SIMPLE AIRCRAFT REFRIGERATION
refrigerant, is used. The refrigerant used,
CYCLE
does not leave the system, but is circulated
throughout the system alternately
Figure shows the schematic of a simple
condensing and evaporating.
aircraft refrigeration system and the

© Copyright Reserved by Gateflix.in No part of this material should be copied or reproduced without permission
In evaporating, the refrigerant absorbs its heat to the surrounding condensing
latent heat from the solution which is used medium which is normally air or water.
for circulating it around the cold chamber.
In condensing, it gives out its latent heat to Receiver
the circulating water of the cooler. The condensed liquid refrigerant from the
Vapour compression refrigeration system condenser is stored in a vessel known as
is now-a-days used for all purpose receiver from where it is supplied to the
refrigeration. It is used for all industrial evaporator through the expansion valve or
purposes from a small domestic refrigerant control valve.
refrigerator to a big air conditioning plant.
Expansion Valve
6.4.1 SIMPLE VAPOUR COMPRESSION It is also called throttle valve or refrigerant
REFRIGERATION SYSTEM control valve. The function of the expansion
valve is to allow the liquid refrigerant
It consists of the following essential parts: under high pressure and temperature to
pass at a controlled rate after reducing its
pressure and temperature. Some of the
liquid refrigerant evaporates as it passes
through the expansion valve, but the
greater portion is vaporized in the
evaporator at the low pressure &
temperature
Evaporator
An evaporator consists of coils of pipe in
which the liquid-vapour. refrigerant at low
pressure and temperature is evaporated
and changed into vapour refrigerant. In
evaporating, the liquid vapour refrigerant
absorbs its latent heat of vaporization from
the medium which is to be cooled.
Fig. Simple vapour compression
refrigeration system 6.4.2 THEORETICAL VAPOUR COMPRESSION
CYCLE
Compressor
A vapour compression cycle with dry
The low pressure and temperature vapour
saturated vapour after compression is
refrigerant from evaporator is drawn into
shown on T-S and P-h diagrams as shown
the compressor through the suction valve
in Figures. At point 1, let the temperature,
A, where it is compressed to a high
pressure and entropy of the vapour
pressure and temperature. This high
refrigerant is T1, p1 and S1 respectively.
pressure & temperature vapour refrigerant
The four processes of the cycle are as
is discharged into the condenser through
follows:
the discharge valve B.

Condenser
The condenser consists of coils of pipe in
which the high pressure and temperature
vapour refrigerant is cooled and
condensed. The refrigerant, while passing
through the condenser, gives up its latent

© Copyright Reserved by Gateflix.in No part of this material should be copied or reproduced without permission
Some of the liquid refrigerant evaporates
as it passes through the expansion valve,
but the greater portion is vaporized in the
evaporator. We know that during the
throttling process, no heat is absorbed or
rejected by the liquid refrigerant.

Fig: VCR cycle on T-s and P-h diagram Process 4-1-Vaporizing Process

Process 1-2-Compression Process The liquid-vapour mixture of the


refrigerant at pressure P4=P1 and
The vapour refrigerant at low pressure P1 temperature T4=T1 is evaporated and
and temperature T1 is compressed changed into vapour refrigerant at constant
isentropic ally to dry saturated vapour as pressure and temperature, as shown by the
shown by the vertical line 1-2 on the T-s horizontal line 4-1 on T-s and p-h
diagram and by the curve 1-2 on p-h diagrams. During evaporation, the liquid-
diagram. The pressure and temperature vapour refrigerant absorbs its latent heat
rise from P1 to P2 and T1 to T2 respectively. of vaporization. The heat which is absorbed
The work done during isentropic by the refrigerant is called refrigerating
compression per kg of refrigerant is given by effect and it is briefly written as RE.
w = h2 – h1 The refrigerating effect or the heat
Where h1 = Enthalpy of vapour refrigerant absorbed or extracted by the refrigerant
at temperature T1, i.e. at suction of the during evaporation per kg of refrigerant is
compressor, and h2=Enthalpy of the given by RE = h1 – h4 = h1 – hf3
vapour refrigerant at temperature T2. i.e. at Where hf3 = Sensible heat at temperature
discharge of compressor. T3, i.e. enthalpy of liquid refrigerant
Process 2-3- Condensing Process leaving the condenser.

The high pressure and temperature vapour CO Efficient of Performance


refrigerant from the compressor is passed
through the condenser where it is Coefficient of performance, C.O.P. =
completely condensed at constant pressure (Refrigerating effect)/ (Work done)
P2 and temperature T2 as shown by the h  h 4 h1  h f 3
horizontal line 2-3 on T-s and p-h O.P  1 
h 2  h1 h 2  h1
diagrams. The vapour refrigerant is
changed into liquid refrigerant. The
6.5 EFFECT OF PARAMETERS ON COP OF
refrigerant, while passing through the
VAPOUR COMPRESSION REFRIGERATION
condenser, gives its latent heat to the
CYCLE
surrounding.
1. Effect of Suction Pressure
Process 3-4 - Expansion Process
The suction or evaporator pressure
decreases due to the frictional resistance of
The liquid refrigerant at pressure P3 = P2
flow of the refrigerant. Let us consider a
and temperature T3 = T2, is expanded by
theoretical vapour compression cycle 1-2-
throttling process (Isenthalpic Process)
3-4 when the suction pressure decreases
through the expansion valve to a low
from Ps to Ps’ as shown on p-h diagram in
pressure P4 = P1 and Temperature T4 = T1
Figure.
as shown by the curve 3-4 on T-s diagram
It may be noted that the decrease in suction
and by the vertical line 3-4 on P-h diagram.
pressure:

© Copyright Reserved by Gateflix.in No part of this material should be copied or reproduced without permission
a) Decreases the refrigerating effect from 6.5.1 EFFECT OF LIQUID SUB COOLING
(h1 – h4) to (h11 – h4).
b) Increases the work required for It is possible to reduce the temperature of
compression from (h2–h1) to (h21– h11). the liquid refrigerant to within a few
degrees of the temperature of the water
entering the condenser. In some condenser
designs it is achieved by installing a sub-
cooler between the condenser and the
expansion valve. The effect of sub-cooling
of the liquid from t3 = tk to t31 is shown in
Figure. It will be seen that sub-cooling
reduces flashing of the liquid during
fig. Effect of suction pressure expansion and increases the refrigerating
Since the C.O.P, of the system is the ratio of effect. But there is no effect on the work
refrigerating effect to the work done, done by the compressor. And the COP of
therefore with the decrease in suction the cycle is ratio of refrigeration effect to
pressure, the net effect is to decrease the work done by the compressor. So the COP
C.O.P. of the refrigerating system for the of the cycle increases in sub-cooling of the
same refrigerant flow. Hence with the liquid.
decrease in suction pressure
 The refrigerating capacity of the system
decreases.
 The refrigeration cost increases.

2. Effect of Discharge Pressure

In actual practice, the discharge or


condenser pressure increases due to
frictional resistance of flow of the
refrigerant. Let us consider a theoretical
vapour compression cycle l-2-3-4 when the
discharge pressure increases from PD to PD fig. Effect of suction pressure
1 as shown on p-h diagram in Figure

resulting in increased compressor work 6.6VAPOURABSORPTION REFRIGERATION


from (h2 – h1) to (h21 – h1) and decreasing SYSTEMS
refrigeration effect from (h1 – h4) to (h1 –
h41). So COP is ratio of refrigeration effect The Refrigeration effect by Vapour
to work done. So COP of the cycle Compression Refrigeration System is an
decreases. efficient method. But the input energy
given in the VCR system is work i.e. high
grade energy and therefore very expensive.
So In Vapour Absorption Refrigeration
Systems, the mechanical work is replaced
by the heat. Hence these systems are also
called as heat operated or thermal energy
driven systems. Since these systems run on
low-grade thermal energy, they are
preferred when low-grade energy such as
fig. Effect of suction pressure waste heat or solar energy is available.

© Copyright Reserved by Gateflix.in No part of this material should be copied or reproduced without permission
Since conventional absorption systems use heat exchanger. The weak high
natural refrigerants such as water or temperature ammonia solution from the
ammonia they are environment friendly. generator is passed to the heat exchanger
through the pressure reducing valve. The
6.6.1 WORKING OF SIMPLE VAPOUR pressure of the liquid is reduced to the
ABSORPTION REFRIGERATION CYCLE absorber pressure by the throttle valve.
Ammonia vapour is produced in the
The systematic diagram of VAR system is generator at high pressure by a heating
shown. In this, compressor is replaced by generator. The water vapour carried with
absorber system (i.e. Absorber, Pump, Heat ammonia is removed in the rectifier and
Exchanger Heat generator and rectifier). only the dehydrated ammonia gas enters
Working of the system is almost same as into the condenser. High pressure NH3
the working of VCR system excepting the vapour is condensed in the condenser. The
functionality of compressor. The working cooled NH3 solution is passed through a
can be explained with help of sketch throttle valve and the pressure and
diagram. temperature of the refrigerant are reduced
The low temperature refrigerant enters the below the temperature to be maintained in
evaporator and absorbs the required heat the evaporator. The refrigerant absorbs the
from the evaporator and leaves the heat from the medium and converts in to
evaporator as saturated vapour. Slightly the vapour phase and again it goes to
low pressure NH3 vapour is absorbed by generator.
the weak solution of NH3 which is sprayed
in the absorber as shown in fig. 6.6.2 COP OF IDEAL VAR CYCLE

The working of VAR system is shown in fig.


Let us consider the Heat QE is absorbed by
the refrigerant in absorber at temperature
of TE. In the heating generator, QG amount of
heat is added at temperature TG to the
refrigerant. The QC amount heat is rejected
in condenser at temperature of TC. Pump
work is WP.
The COP of system is given as
Refrigeration Effect
 COP VAR 
Given input
QE Q
  E
Fig. Vapour Absorption Refrigeration QG  WP QG
System Considering a reversible refrigeration
cycle,
Weak NH3 solution (i.e. know as aqua– QC = QG + QE -----------(1)
ammonia) enters in to absorber and for a reversible cycle Clausius Inequality
becomes strong solution after absorbing Law,
NH3 vapour. Then it is pumped to the δQ Q Q Q
generator through the heat exchanger. The ∮ T  0, TC  TE  TG
C E G
pump increases the pressure of the strong Substituting the value from equation 1
solution. QG  QE QE QG
The strong NH3 solution absorbs heat form  
TC TE TG
high temperature weak NH3 solution in the

© Copyright Reserved by Gateflix.in No part of this material should be copied or reproduced without permission
Q E Q E QG QG  If the operating temperatures are above
Or,    o
TC TE TG TC 0 C, then pure water can also be used as
QE (TC  TE ) QG (TG  TC ) secondary refrigerant, The commonly
Or,  used secondary refrigerants are:
TC TG
 The solutions of water and ethylene
Q (T  TC ) TC
Or, E  G  glycol,
QG TG (TC  TE )  The solutions of propylene glycol or
 COP VAR  Carnot  COP Carnot calcium chloride.

6.7 REFRIGERANT 6.7.2 PROPERTIES OF REFRIGERANT

Refrigerant is the fluid used for heat There should be the following properties of
transfer in a refrigerating system that the refrigerants:
absorbs heat during evaporation from the a) Latent heat of vaporization:
region of low temperature, and releases it should be as large as possible so that the
heat during condensation at a region of required mass flow rate per unit cooling
higher temperature. capacity will be small.
Due to several environmental issues such
b) Isentropic index of compression:
as ozone layer depletion and global
It should be as small as possible so that the
warming and their relation to the various
temperature rise during compression will
refrigerants used, the selection of suitable
be small.
refrigerant has become one of the most
important issues in recent times. c) Vapour specific heat:
For Example, Air used in an air cycle It should be large so that the degree of
refrigeration system can also be considered superheating will be small.
as a refrigerant.
d) Thermal conductivity:
6.7.1 CLASSIFICATION OF REFRIGERANT Thermal conductivity in both liquid as well
as vapour phase should be high for higher
There are two types of refrigerants
heat transfer coefficients.
1. Primary Refrigerant
2. Secondary Refrigerant e) Viscosity:
1. Primary Refrigerant Viscosity should be small in both liquid and
vapour phases for smaller frictional
Primary refrigerants are those fluids, which pressure drops.
are used directly as working fluids, for
example in vapour compression and f) Ozone Depletion Potential (ODP):
vapour absorption refrigeration systems. According to the Montreal protocol, the
ODP of refrigerants should be zero, i.e., they
2. Secondary Refrigerant should be non-ozone depleting substances.
Secondary refrigerants are those liquids, Refrigerants having non-zero ODP have
which are used for transporting thermal either already been phased-out (e.g. R11, R
energy from one location to other. 12).
Secondary refrigerants are also known g) Global Warming Potential (GWP):
under the name brines or antifreezes. Refrigerants should have as low GWP value
Generally, the freezing point of brine will as possible to minimize the problem of
be lower than the freezing point of its global warming.
constituents. So these are used as
Secondary refrigerants. h) Toxicity

© Copyright Reserved by Gateflix.in No part of this material should be copied or reproduced without permission
The refrigerants used in a refrigeration R -500: Mixture of R 12 and R 152a
system should be non-toxic. R -502: Mixture of R 22 and R 115
R-503: Mixture of R 13 and R 23
6.8 DESIGNATION OF REFRIGERANTS
(4) Hydrocarbons:
Propane (C3H8) : R 290
There are following types of refrigerants.
The designation of these refrigerants can n-butane (C4H10) : R 600
be done by following manner: Iso-butane (C4H10) : R 600 a

1) Halocarbon compounds: Examples


These refrigerants are derivatives of Q.1 In reversed Bray ton cycle, the
alkenes (CnH2n+2) such as methane (CH4), temperature at the end of heat
ethane (C2H6). These refrigerants are absorption and heat rejection are
designated by R-ABC 00C and 300C. The pressure ratio is 4
Where: and the pressure in the cooler is 4
A+1 indicates the number of Carbon (C) bar, determine the temperature at
atoms all the states and volume flow rates
B-1 indicates number of Hydrogen (H) at inlet of compressor and at exit of
atoms, and Z indicates number of Fluorine turbine for 1 TR cooling capacity.
(F) atoms Solution:
T1 = 273 K
Example: R- 22 T3 = 303 K
A = 0 ⇒ No. of Carbon atoms = 0+1 = 1 Temperature at all points
γ 1
T2
B = 2 ⇒ No. of Hydrogen atoms = 2-1 = 1  r γ
C = 2 ⇒ No. of Fluorine atoms = 2 T1
0.4
The balance = 4 – no. of (H+F) atoms
T2  273  4 1.4  405.67K
= 4-1-2 = 1, So No. of Chlorine atoms = 1
γ 1
T3
 r γ
The chemical formula of R 22 = CHClF2 T4
The commonly used refrigerant 303
T4  0.4
 203.9K
i.e. R-134a = C2H2F4
 4 1.4

2) Inorganic refrigerants:
These are designated by number 7 followed
by the molecular weight of the refrigerant.
Examples:
(a) Ammonia: Molecular weight is 17,
the designation is R -717
(b) Carbon dioxide: Molecular weight is 44, Volume flow rate of air for 1 TR or
the designation is R -744 3.5KJ/sec
(c) Water: Molecular weight is 18,
Refrigeration effect  ma cp  T1  T4 
the designation is R -718
3.53.5  3.5  mp X1.005(273  203.9)
3) Azeotropic Refrigerant: mp  0.05kg / sec
Azeotropic mixtures are designated by 500 At the inlet of compressor density,
series, these are the mixture of two or more
P1 305
halocarbon compound. P1    1.27kg / sec.
Some Azeotropic mixtures: RT1 287X273

© Copyright Reserved by Gateflix.in No part of this material should be copied or reproduced without permission
30
1.005  T2  T31 
Volume flow rate

V = P1 X ma = 1.27 X 0.05 60
= 0.064 m3/sec. = 0.5 × 1.005 (303 – 207.92)
= 47.78 KW
Q.2 The compressed air from main c) Refrigeration effect
compressor of an air craft cooling RE  ma cp  Tcabin  T31 
system is bled off at 4.5 bar and
2000C. It is then passed through the = 0.5 × 1.005 (298 – 207.92)
heat exchange in which the ram air =45.27 KJ/s =45.27/3.5 = 12.9 TR
is forced through for cooling purpose Q.3 A dense air refrigeration machine
by fan driver by cooling turbine. The operating on Bell-Coleman cycle
condition of the inlet of cooling operates between 3.4 bar and 17 bar.
turbine is 4 bar an 300C. The air The temperature of air after the
expands in cooling turbine is 4 bar cooler is 150C and after the
an 300C. The air expands in cooling refrigerator is 60C. The refrigeration
turbine up to 0.7 bar. The isentropic capacity is 6 Tonnes, calculate:
efficiency of cooling turbine is 80% 1) temperature after the
and flow rate through cooling turbine compression and expansion
is 30 kg/min. Find 2) air circulation per minute
a) Actual exit temperature from 3) work of compressor and
cooling turbine expander
b) The power delivered to ram air 4) Theoretical COP.
blow fan Solution:
c) Tons of refrigeration, if T3  150 C  288K
temperature of cabin cockpit T1  60 C  279K
area is 250C.
RE = 6 tonnes
Solution:-
(I) temp. After comp. & expansion
1) Exit temp. Of turbine γ 1
γ1 γ 0.4

T2  P2 
γ 0.4
T2  P2   17  1.4
 0.7  1.4
   
   T3  300    184.15K T1  P1 
T3  P3   4   3.4 
T2=279×1.5=445.69 k
V 1 0.4
T4  P4  V  3.4  1.4
     .62
T3  P3   17 
T4 = 288× 0.62 =180.2 K

T3 = 184.15 K
Efficiency of the turbine
T  T1
0.8  2 3
T2  T3
T31    T2  T3  0.8  T2 (II) Air circulation per minute
Refrigeration effect= ma cp  T1  T4 
T31  303  0.8(303  184.15)
6 ×210 = ma ×1.005(279-180.2)
T31  207.92k Ma=12.7 kg/min.
b) Power delivered to fan = work (III) Work of compressor:-
output from turbine

© Copyright Reserved by Gateflix.in No part of this material should be copied or reproduced without permission
n
w comp  ma R  T2  T1 
n 1
1.4 12.7
   287(445.29  279)
1.4  1 60
= 35.44 KW
Work of expander h1  h g at -50C
n
w turbine  ma R  T3  T4  h1  185.4kJ / kg
n 1
1.4 12.7 Enthalpy of point 4
   287(288  180.2) h 4  h3  h f 3  69.5kJ
1.4  1 60
= 22.89 KW Refrigerant effect
(IV) Cop of cycle m   h1  h 4   m
 185.4  69.5
Heatabsorbed 6X3.5 m  115.9kJ / sec
 Cop   
10  3.5  115.9  m 
workdone 35.44  22.89
COP = 1.67 Mass flow rate
35
m   0.3kg / sec.
Q.4 A simple saturation cycle using R-12 115.9
is desired taking a load of 10 Tons. In process(1-2)i.e.isentropic process
The refrigerating and ambient s1  s 2
temperature are 00C and 300C. A
minimum temperature difference of  Tsap 
sg1  sg2  cp ln  
50C is required in evaporator and  Tsat 
condenser for heat transfer. Find  Tsap 
a) Mass flow rate through system 0.6991  0.6839  cp ln  
b)Power required in kw. The  308 
properties of R-12 are following:- Tsup  312.9 k
Sat. Sat. Specific volume m3/kg h2 = hg2 + cp Tsap − Tsat
Temp. Pr.
0C bar Sat (10-3) Sat. vapour = 201.5 + 0.95 (312.9-308)
-50C 2.61 0.71 0.0650 = 206.25 KJ/Kg
00 C 3.08 0.72 0.0554 (II) Power required by compressor
300C 7.45 0.77 0.0235 wm   h 2  h1  
350C 8.47 0.79 0.0206 0.3  (206.25 185.4)  6.3KW

Enthalpy kJ/kg Entropy kJ/kg. k Q.5 An ideal vapour compression


Sat. Sat. Sat. Sat. refrigerator using R-12 operates
Liquid vapour Liquid vapour between temperature limits of -100C
31.4 185.4 0.1251 0.6991 and 400C. The refrigerator leaves
36.1 187.5 0.1420 0.6966 the condenser dry saturated. The
64.4 199.6 0.2399 0.6854 rate of flow of refrigerant through
69.5 201.5 0.2559 0.6839 the unit is 150 Kg/hr.
Specific heat of vapour R-12 is 0.95 Calculate the refrigerating effect per
KJ/KgK kg of refrigerant , COP If
Solution:- 1) refrigerant leaves the condenser
Temp. Of refrigerant in evaporator as dry saturated
is -50C and in the condenser is 350C. 2) refrigerant is sub cooled to 200C
At low pressure before throttling.

© Copyright Reserved by Gateflix.in No part of this material should be copied or reproduced without permission
Assume that enthalpy of refrigerant h13  50.59  h 4
before throttling is approximately Refrigerating effect = h1 – h4
equal to enthalpy of refrigerant at = 183.1– 50.59
under cooled temperature of 200C = 132.51 KJ/kg
and it as hf = 50.59 kJ/kg. h1  h 4 132.51
 Cop     5.06
h 2  h1 209.6  183.19

Q.6 A two cylinder R-12 compressor has


bore and stroke equal to 5.65 cm
and 5 cm. Its rpm is 1450 rpm.If
liquid refrigerant is throttled by
throttle valve at 40 C. Determine
0

the mass of refrigerant circulated


per minute and theoretical capacity
when suction temperature is –100C.
Assume γ = 1.13 , properties of R-12
are:
Temp Pr.(bar) Volume Enthalpy
vapour liquid kJ/kg
m3/kg
-100C 2.1928 0.07702 190.72
Solution:
(1) if the refrigerant leaves dry 400C 9.5944 0.01863 239.29
saturated h1 = 183.19 kJ/kg
s1  7019kJ / kg.k Enthalpy Entropy Entropy vapour
vapour liquid kJ/kgk
s 2  s1 kJ/kg kJ/kgk
 Tsap  347.96 0.9656 1.5632
s1  sg  cp ln   368.67 1.1324 1.5456
 Tsat 
At 400C Sg = 0.6825 Solution:
Cp = 0.609 kJ/kgk Theoretical piston displaced volume
π
 Tsap  VP  d 2 LNk
s1  0.6825  0.609ln   4
 313  π
   5.65X102   5 1450  2 102
2

 Tsap  4
0.7019  0.6825  0.609ln  
 313  = 0.335 m3/min.
Tsap  323.1K Vg = 0.07702 m3/kg at -100C
So refrigerant mass circulated/min
h 2  h g  cp  Tsap  Tsat  at -100C suction temperature
= 203.20+ 0.609 (323.1-313) = 0.335/0.07702 = 4.6 kg/min
h 2  209.37kJ / kg.k h1 = 347.96 kJ/kg
h4 = 239.29 kJ/kg
h3  h f 3  h 4  74.59kJ / kg Refrigerating effect
(i) Refrigerating effect = h1 – h4 = RE  m  h1  h 4 
183.1 – 74.59
= 108.6 KJ/kg = 4.6 X (347.96 – 239.29)
h h 108.6 = 500.8 kJ/min
 Cop   1 4   4.15
h 2  h1 209.6  183.1 Q.7 A R-12 refrigeration machine has
(iii) off refrigerant is sub cooled saturated suction temperature of

© Copyright Reserved by Gateflix.in No part of this material should be copied or reproduced without permission
50C and saturated discharge Q.8 Assume a simple saturated cycle
temperature of 40 C. Determine
0 with suction saturation temp. -
i) COP 100C and condensing saturation
ii) Theoretical power. Per ton of temperature of 300C. If clearance
refrigeration when the compression volume 15% of stroke volume. How
is dry properties are: will the volumetric efficiency vary
Temp. Pr. bar Volume hf for different refrigerant namely
m3/kg (1) R-12 (2) R-22 (3)NH3?
50C 3.6375 0.048 204.68
Solution:
400C 9.5909 0.0184 239.03
Volumetric efficiency can be given
Enthalpy Entropy Entropy
v 
vapour liquid vapour n vol.  1  C  C  suction 
kJ/kg kJ/kgk kJ/kgk  vdischarge 
347.96 0.9656 1.5632
368.67 1.1324 1.5456 1) For R-12, from p-h chart
Solution: m3
vsuction  0.07813 , vdischarge  0.025m3 / kg
Process 1-2 is isentropic process. kg
S1  S2  0.07813 
So n vol.  1  0.05  0.05 
 0.025 
= 0.894 = 89.4%
For R-22, from p-h chart

 Tsap 
1.5569  Sg2  cp ln  
 Tsat 
 Tsap 
1.5569  1.5569  0.762ln  
 313  vsuction  0.07m3 / kgvdischarge  0.0225m3 / kg
Tsup  317.5k  0.07 
So n vol.  1  0.05  0.05 
h 2  h g2  cp  Tsap  Tsat   0.0225 
= 1.05 – 0.0156 = 0.894 = 89.4%
= 368.87 + 0.762 (317.5 – 313)
= 372.3 kJ/kg (3) For NH3 , from p-h chart,
h 3  h f at 313K =239.03 kJ/kg = h4
vsuction  0.4184m3 / kg, vdischarge
  h1  h 4 
Refrigerating effect  m
 0.135m3 / kg
210
m   h1  h 4   0.4184 
60 So n vol.  1  0.05  0.05 
m  0.03kg / sec  0.135 
= 0.895 = 89.5%
Theoretical power per ton of
refrigeration
Q.9 Catalogue data from R-12
Pm   h 2  h1 
compressor shows that compressor
 0.03  372.3  354.885  0.53KW / TR delivers 15 TR at saturation suction
h  h 4 354.885  239.03 temperature of -50C and saturated
COP  1   6.65 condensing temperature of 400C
h 2  h1 372.3  354.885
and actual suction temperature is

© Copyright Reserved by Gateflix.in No part of this material should be copied or reproduced without permission
150C. Liquid leaving the condenser
is saturated. Calculate the mass flow
rate of this compressor.
Solution:

from the p-h diagram


h g1  353KJ / kg, vg  0.66m3 ,
h1  363KJ / kg, v1  0.072m3 / kgP
The refrigeration effect
m   h1  h 4 
15  210  m(363
  239)
15  210
m   25.40
124
m  25.40kg / min.
And volume flow rate
V = 25.4 X 0.072
V = 1.828 m3/min.

Q.10 A geothermal wall at 1300C supplies


heat at a rate of a 100500 kJ/hr to
an absorption refrigeration system.
The environment is at 300C and the
refrigerated space is maintained at -
220C. Determine the maximum
possible heat removed from
refrigerated space.
Solution:
TC  303kTE  251kTG  403k
QG  100500kJ / hr
For a reversible absorption cycle
Q TE T  TC
COP  E  X G
QG TC  TE TG
251 403  303
 
303  251 403
100500
QE  QG X1.19  X1.19
3600
QE  32.22KW

© Copyright Reserved by Gateflix.in No part of this material should be copied or reproduced without permission
7 PSYCHOMETRY

Psychrometry is the branch of science Fig. Water vapour in air mixture


which deals with study of the properties of
the moist air i.e. mixtures of air and water 7.1.3 HUMIDITY RATIO
vapour. Atmospheric air is a mixture of
many gases plus water vapour and a The humidity ratio or specific humidity (W)
number of pollutants. is the mass of water per kg of dry air in a
The amount of water vapour and pollutants given mixture of dry air and water vapour.
vary from place to place. A mixture of If mv is the mass of water vapour and ma is
various gases that constitute air and water the mass of dry air. Then humidity ratio
vapour is known as moist air. m
Constituent Molecular weight Mol fraction W= v
ma
Oxygen 32 0.2095
Nitrogen 28.016 0.7809 We know that
Carbon dioxide 44.01 0.0003 PV V  mV R VTforthewatervapour
7.1 PROPERTIES OF MOIST AIR And Pa V  ma R aV T for the dry air
Substituting the values
7.1.1 MOIST AIR PV V
The moist air can be thought of as a R T P  Ra
W V  V
mixture of dry air and moisture. Based on Pa V Pa  R V
the above composition the molecular RaT
weight of dry air is found to be 28.966 and P  8.314 / 28 PV
the gas constant R is 0.287035 KJ/Kg.K.  V  0.622
Pa  8.314 /18 Pb  PV
If the partial pressure of dry air and water
vapour is Pa and PV, then according to PV
W  0.622    (7.2)
Dalton’s law of partial pressure Pb  PV
Pb  Pa  Pv ------- (7.1) If air is saturated at temperature T, then
Where Pb is barometer reading of Specific humidity of saturated air
Ps
atmospheric pressure. In general, Ws  0.622    (7.3)
atmospheric pressure is taken as 760 mm Pb  Ps
of Hg.
7.1.4 DEGREE OF SATURATION
7.1.2 SATURATED AIR
The degree of saturation is the ratio of the
At a given temperature and pressure, the humidity ratio (W) of the mixture to the
dry air can only hold a certain maximum humidity ratio of a saturated mixture (WS)
amount of moisture. When the moisture at the same temperature and pressure. it is
content in the moist air is maximum, the air denoted by μ.
is known as saturated air. Degree of Saturation
0.622PV
W P P P (P  P )
μ  b V  V b s
Ws 0.622Ps Ps .  Pb  PV 
Pb  Ps
P (P  P )
μ  V b s    (7.4)
Ps .  Pb  PV 

© Copyright Reserved by Gateflix.in No part of this material should be copied or reproduced without permission
7.1.5 RELATIVE HUMIDITY moisture presents in the air begins to
condense is known as dew-point
Relative humidity is defined as the ratio of temperature (DPT) of air.
the mass of water vapour present in a given
volume of air at temperature T to the mass 7.1.8 WET BULB TEMPERATURE (WBT)
of water vapour when the same volume of
air is saturated at the same temperature T It is temperature of air measured by
and pressure. Relative humidity is normally thermometer when wet cloth is covered
expressed as a percentage. When Φ is 100 over the thermometer.
percent, the air is saturated.
7.1.9 DENSITY OF DRY AIR
Relative humidity
mass of water vapour in a given volume The density of air or air density is the mass
of air at Temperature T of air per unit volume Air density decreases

mass of water vapour when same volume of air is with increasing altitude. It also changes
saturated at Temperature T with variation in temperature or humidity.
At sea level and at 15 °C, air has a density of
mV approximately 1.225 kg/m3.density of dry
 air can be calculated using the ideal gas
mS
law, expressed as a function
Assuming the water vapour is an ideal gas,
of temperature and pressure
PV V  mV R VT P
And for the saturated air condition ρa  a      7.6 
R a TD
Ps V  ms R VT
Where ρ  Air density
So from the equation () and Equation (), we
Pa = Absolute pressure
get
TD = Dry Bulb temperature
m V PV
 Ra = Specific gas constant for dry air
mS PS
m P 7.1.10 ENTHALPY OF THE AIR
So   V  V --------- (7.5)
mS PS The enthalpy of moist air is the sum of the
for the dry air mV and PV are zero, so enthalpy of the dry air and the enthalpy of
Φ =0 the water vapour. Enthalpy values are
for the dry air mV andPV are equal to always based on some reference value. The
enthalpy of the mixture is given by
ms andPs , so
h  h a  Wh V  Cp Ta  W h g  1.88  t W  t dp 
Φ = 100%
So relative humidity varies from 0% to ------ (7.7)
100%.
7.2 PSYCHROMETRIC CHART
7.1.6 DRY BULB TEMPERATURE
A Psychrometric chart graphically
DBT is the temperature of the air as represents the thermodynamic properties
measured by a standard thermometer of moist air. Standard Psychrometric charts
when it is unaffected by thermometer. are bounded by the dry-bulb temperature
line (abscissa) and the vapour pressure or
7.1.7 DEW-POINT TEMPERATURE (DPT) humidity ratio (ordinate). The Left Hand
Side of the Psychrometric chart is bounded
If moist air is cooled at constant pressure, by the saturation line. Figure shows the
then the temperature at which the schematic of a Psychrometric chart.

© Copyright Reserved by Gateflix.in No part of this material should be copied or reproduced without permission
 Psychrometric charts are readily Dry bulb temperature is temperature of an
available for standard barometric air sample, as determined by an ordinary
pressure of 101.325 KPa at sea level thermometer. It is typically plotted as
o the abscissa (horizontal axis) of the graph.
and for normal temperatures (0-50 C).
 The Relative humidity at the saturation The SI unit for temperature is degrees
curve is 100 %. Celsius. These lines are drawn straight,
 At the saturation curve, Dry bulb parallel to each other. Each line represents
Temperature, Wet bulb Temperature a constant temperature.
and Dew point temperature are same.

7.3.2 WET-BULB TEMPERATURE (WBT)


LINE
Wet-bulb temperature is temperature of
an air sample after the air has passed over
a large surface of liquid water in an
insulated channel. When the air sample is
saturated with water, the WBT will read
the same as the DBT. These lines are
oblique lines that differ slightly from the
enthalpy lines. They are identically straight
but are not exactly parallel to each other.
These intersect the saturation curve at DBT
fig. Psychrometric Chart
point.
The parameters of Psychrometric chart are
relative humidity, dry bulb temperature,
wet bulb temperature, dew point
temperature, specific humidity, specific
volume, specific enthalpy and vapour
pressure.
The Psychrometric chart allows all the
parameters of some moist air to be fig. Wet Bulb Temperature line
determined from any three independent
parameters, one of which must be the 7.3.3 DEW POINT TEMPERATURE (DPT)
pressure. LINE
7.3 SIGNIFICANCE OF DIFFERENT LINES Dew point temperature (DPT) is the
ON PSYCHROMETRY CHART temperature at which a moist air at the
same pressure would reach water vapour
7.3.1 DRY-BULB TEMPERATURE LINES saturation. At this point further removal of

© Copyright Reserved by Gateflix.in No part of this material should be copied or reproduced without permission
heat would result in water vapour typically plotted as the ordinate (vertical
condensing into liquid water fog. From the axis) of the graph. For a given DBT there
state point follow the horizontal line of will be a particular humidity ratio for
constant humidity ratio to the intercept of which the air sample is at 100% relative
100% RH, also known as the saturation humidity. These are the horizontal lines on
curve. The dew point temperature is equal the chart.
to the fully saturated dry bulb or wet bulb
temperatures.

fig. Dew Point temperature line


Fig. Humidity ratio lines
7.3.4 Relative humidity (RH) Line 7.3.6 SPECIFIC ENTHALPY LINE
Relative humidity (RH) is the ratio of the
mole fraction of water vapor to the mole These are oblique lines drawn diagonally
fraction of saturated moist air at the same downward from left to right across the
temperature and pressure. RH is chart that are parallel to each other. In the
dimensionless, and is usually expressed as approximation of ideal gases, lines of
a percentage. Lines of constant RH reflect constant enthalpy are parallel to lines of
the physics of air and water: they are constant WBT.
determined via experimental measurement.
Relative humidity: These hyperbolic lines
are shown in intervals of 10%. The
saturation curve is at 100% RH, while dry
air is at 0% RH.

Fig. Specific Enthalpy Lines


fig. relative humidity lines
7.3.7 SPECIFIC VOLUME LINE
7.3.5 HUMIDITY RATIO LINE
Specific volume is the volume of the
mixture containing one unit of mass of dry
Humidity ratio is the proportion of mass of
air. The SI units are cubic meters per
water vapour per unit mass of dry air at the
kilogram of dry air. These lines are a family
given conditions. It is also known as the
of equally spaced straight inclined lines
moisture content or mixing ratio. It is
that are nearly parallel.

© Copyright Reserved by Gateflix.in No part of this material should be copied or reproduced without permission
Fig. Sensible cooling Process

7.4.2 SENSIBLE HEATING PROCESS

During this process, the moisture content


of air remains constant and its temperature
increases as it flows over a heating coil. The
fig. Specific Volume lines
heat transfer rate during this process is
7.4 DIFFERENT PROCESS ON given by
PSYCHROMETRIC CHART
7.4.1 SENSIBLE COOLING PROCESS
During this process, the moisture content
of air remains constant but its temperature
decreases as it flows over a cooling coil. For
moisture content to remain constant the
surface of the cooling coil should be dry
and its surface temperature should be
greater than the dew point temperature of
air. If the cooling coil is 100% effective,
then the exit temperature of air will be
equal to the coil temperature. However, in
practice, the exit air temperature will be
higher than the cooling coil temperature.
The heat transfer during the cooling
process is given as
QC  ma  h O  h A   ma Cpm  TO  TA  ----(7.8)

fig. Sensible Heating Process


Qh  ma  h b  h o   ma Cpm  Tb  TO     (7.9)
Where Cpm is the humid specific heat. ma is
the mass flow rate of dry air (kg/s).

7.4.2.1 BY PASS FACTOR

In Fig., the temperature Td3 is the effective


surface temperature of the heating coil, and
is known as apparatus dew-point (ADP)
temperature. Td1 and Td2 are the
temperature of air at inlet and outlet of the
coil. In an ideal situation, when all the air
comes in perfect contact with the heating
coil surface, then the exit temperature of
air will be same as ADP of the coil.

© Copyright Reserved by Gateflix.in No part of this material should be copied or reproduced without permission
7.4.3 COOLING AND DEHUMIDIFICATION
PROCESS

When moist air is cooled below its dew-


point by bringing it in contact with a cold
surface as shown in Fig., some of the water
But in actual case the exit temperature of vapour in the air condenses and leaves the
air will always be lesser than the apparatus air stream as liquid, as a result both the
dew-point temperature. So By -pass factor temperature and humidity ratio of air
(BPF) is defined as the ratio of heat loss to decreases as shown. The heat and mass
maximum possible heat transfer. transfer rates can be expressed in terms of
Heat Loss T T the initial and final conditions by applying
BPF   d3 d2
Maximum Possible heat transfer Td3  Td1 the conservation of mass and conservation
--------- (7.10) of energy equations.
In the same way, Bypass factor for cooling
coil can be given as
Heat Loss T T
BPF   d2 d3
Maximum Possible heat transfer Td1  Td3
--------- (7.11)

7.4.2.2 EFFICIENCY OF COIL Fig. Cooling

The efficiency of heating Coil is defined as


the ratio of actual heat transfer to the cold
air to maximum possible heat transfer to
the cold air by heating coil.
Efficiency
Actual Heat transfer

Maximum Possible heat transfer
T T Fig. dehumidification
 d2 d1 -------- (7.12) By applying mass balance for the water
Td3  Td1 ma.Wo = ma.WC + mw
In the same way efficiency for cooling coil By applying energy balance
can be given as ma.ho = Qt +ma.hC + mw.hw
The efficiency of cooling Coil is defined as From the above two equations, the load on
the ratio of actual heat transfer to the cold the cooling coil, Qt is given by:
air to maximum possible heat transfer to
the cold air by heating coil. Qt=ma(ho.-hC)+ma.(Wo-WC).hw ------- (7.15)
Efficiency Where Wo and Wc are the humidity ration
Actual Heat transfer before and after the cooling process.mw is
 mass of water vapour separating from air.
Maximum Possible heat transfer It can be observed that the cooling and de-
T T
 d1 d2    (7.13) humidification process involves both latent
Td1  Td3 and sensible heat transfer processes, hence,
By comparing the equation (7.11) and the total, latent and sensible heat transfer
equation (7.13) of Bypass factor and rates (Qt, Ql and Qs) can be written as
efficiency of the heating and cooling coil, Qt = QL + QS
we get QL = ma.(Wo- WC).hw
  1  BPF -------- (7.14) QS = ma (ho.- hC) = ma.Cpm. (To.- TC)

© Copyright Reserved by Gateflix.in No part of this material should be copied or reproduced without permission
7.4.3.1 SENSIBLE HEAT FACTOR From energy balance:
Qh = ma (hD-ho) - mw hw --------- (7.16)
By separating the total heat transfer rate Where Qh is the heat supplied through the
from the cooling coil into sensible and heating coil. mw and hw is the mass and the
latent heat transfer rates, a useful enthalpy of steam. Since this process also
parameter called Sensible Heat Factor involves simultaneous heat and mass
(SHF) is defined. transfer, we can define a sensible heat
So SHF is defined as the ratio of sensible to factor for the process in a way similar to
total heat transfer rate. that of a cooling and dehumidification
Sensible Heat Transfer process.
SHF 
Total Heat Transfer
Sensible Heat(SH) 7.4.5 COOLING & HUMIDIFICATION
=
Sensible Heat SH   Latent heat(LH) PROCESS

During this process, the air temperature


7.4.4 HEATING AND HUMIDIFICATION
drops and its humidity increases. This
PROCESS
process is shown in Fig. This can be
achieved by spraying cool water in the air
During winter it is essential to heat and
stream. The temperature of water should
humidifies the room air for comfort. This is
be lower than the dry-bulb temperature of
normally done by first sensible heating the
air but higher than its dew point
air and then adding water vapour to the air
temperature to avoid condensation (TDPT <
stream through steam nozzles as shown in
Tw < TO).
the fig. By this process, dry bulb
It can be seen that during this process there
temperature and humidity ratio increase.
is sensible heat transfer from air to water
Appling Mass balance of water vapour for
and latent heat transfer from water to air.
the control volume
Hence, the total heat transfer depends upon
mw = ma (WD-Wo)
the water temperature.
Where ma is the mass flow rate of dry air.

Fig. Cooling Process


Fig. Heating Process

Fig. Humidification process


 If the temperature of the water sprayed
is equal to the wet‐bulb temperature of
Fig. Humidification Process
air, then the net transfer rate will be

© Copyright Reserved by Gateflix.in No part of this material should be copied or reproduced without permission
zero as the sensible heat transfer from upon the state of the individual streams,
air to water will be equal to latent heat the mixing process can take place with or
transfer from water to air. The process without condensation of moisture. Fig
is called as adiabatic dehumidification shows an adiabatic mixing of two moist air
process. streams. As shown in the figure, when two
 If the water temperature is greater than air streams at state points 1 and 2 mix, the
WBT, then there will be a net heat resulting mixture condition 3 can be
transfer from water to air. obtained from mass and energy balance.
 If the water temperature is less than
WBT, then the net heat transfer will be
from air to water.
7.4.6 CHEMICAL DE‐HUMIDIFICATION
PROCESS
This process can be achieved by using a
hygroscopic material, which absorbs or
adsorbs the water vapour from the
moisture.

fig. Mixing of two air streams


From the mass balance of dry air and water
vapour:
ma1  ma 2  ma3
From energy balance:
ma1h1  ma 2 h 2  ma3h 3   ma1  ma 2  h 3
ma1h1  ma 2 h 2
h3     (7.17)
 ma1  ma 2 
From the above equation, it can be
observed that the final enthalpy of mixture
Fig. Chemical De‐Humidification Process is weighted average of inlet enthalpies.
If this process is thermally isolated, then
the enthalpy of air remains constant and Example:
the process is called as adiabatic Q.1 Atmospheric air at 760mm Hg
dehumidification process. As a result the pressure and dry bulb and wet bulb
temperature of air increases as its moisture temperature150 C and 120 C enter
content decreases as shown in Fig. In the heating coil whose temperature
general, the absorption of water by the is 410 C. The By pas factor of heating
hygroscopic material is an exothermic coil is 0.5. Determine the dry bulb
reaction, as a result heat is released during temperature, wet bulb temperature,
this process, which is transferred to air and relative humidity of air leaving the
the enthalpy of air increases. heating coil and also the sensible
heat supplied per kg of dry air.
7.4.7 MIXING OF AIR STREAMS Solve the problem with help of
psychrometric chart.
Mixing of air streams at different states is Solution:
commonly encountered in many processes, By pass factor of heating coil is given
including in air conditioning. Depending as

© Copyright Reserved by Gateflix.in No part of this material should be copied or reproduced without permission
Td3  Td2 pressure of vapour, relative humidity
BPF 
Td3  Td1 and dew point temperature. Assume
41  Td2 standard barometric pressure is 760
0.5  mm of Hg.
41  15 Solution:
Td2  41  0.5  41  15  280C Specific humidity or humidity ratio
is given by
PV
W  0.622
Pb  PV
PV
0.016  0.622
760  PV

0.016  760
Pv 
0.622 1.02575
= 19.05 mm of Hg.
Relative humidity
P
RH  V1
PS1
Thus the actual condition at which Where Ps1 is saturated pressure
air comes out of the heating coil is corresponding to the saturation
280C.With the help of the temperature equal to dry bulb
psychrometric chart relative temperature of 27.50 C. From the
humidity at this point is 28.5% Wet table it can be read as 27.535 mm of
bulb temperature at this point is Hg.
160C. So,
From psychrometric chart, P 19.05
Enthalpy at point 1 h1 = 31.7 KJ/Kg RH  V1   0.694
PS1 27.535
Enthalpy at point 2 h2 = 44.9 KJ/Kg
So the sensible heat supplied to the = 69.4 %
air Q =h2- h1 Dew point temperature is the
= 44.9-31.7 saturation temperature
= 13.2 KJ/Kg of the dry air. corresponding to the vapour
pressure of the 19.05 mm of Hg as
Q.2 The humidity ratio of an read from table. So, Tdp1 = 21.340 C
atmospheric air at 27.5 C is 0.016
0

kg/ kg of dry air. determine partial Q.3 Air enters a chamber at 5oC dry bulb
temperature (DBT) And 2.5o C wet

© Copyright Reserved by Gateflix.in No part of this material should be copied or reproduced without permission
bulb temperature at the rate of ms
w 3  w 2
100m3 / min and the pressure of 1 mq
bar. While passing through the 35
chamber the air absorbs 50 KW heat  0.0035   0.0081
3600  2.1
and pick’s up 35 kg/hr of saturated Enthalpy of living air :
steam at 110oC. Show the process on
ms
a psychrometric chart and find the h3  h 2  hv 
dry and wet bulb temp of the leaving mq
air at 110oC, enthalpy of saturated hv =2691.3 kj/kg at 110 0C
steam is 2691.3 kj/kg. 35
h 3  37.76   2691.3
Solution: 3600  2.104
For process 1-2 this is sensible =50.19ks/kg
heating process At state (3):
Heat absorbed = 50 kw t3 = 310C, tw2 =18.3 0C
 m(h 2  h1 )
i.e at state 1-2: Q.4 In an air conditioned space, 50 kg
dry air/sec of fresh air at 45 0C DBT
and 30% RH is introduced. The
room air at 25 0C DBT and 50% RH
is recalculated at 450 kg dry air/sec.
The mixed air flows over a cooling
coil which has apparatus dew point
of 12 0C and by pass factor of 0.15.
Determine the conditions at outlet
of cooling coil, RSH, RLH, the cooling
load of coil and the condensate rate
.The saturation pressure of water of
required temperature are
Temperature (0C) Pws (bar)
12 0.14016
25 0.03166
W1=0.0035(from psychometric chart) 45 0.09584
Specific volume = 0.792m3/kg Enthalpy at temperature, h=1.005 T
h1 =14 kj/kg +W (2500+1.885) kg/kg dry air.
Volume flow rate =100 m3/min
=1.67 m3/s Solution:
1.67 Temperature of air after mixing
Mass flow rate  Tx = 0.9 t2 +0.1t1
0.792
= 2.104 kg/s = 0.9×25 +0.4× 45
Now heat transferred:- Specific humidity:
a) At outlet condition:
Q  m(h 2  h1 )  2.104 m(h 2 14)
Pv1
50  2.104 m(h 2  14) W1  0.622 
 Po  Pv1
h2= 37.76 kj/kg
Pv1 = 0.3×0.09584
Now at state 2, from psychometric
chart = 0.02875 bar
0.02875
W2  0.0035, h 2  37.76 kj / kg W1  0.622 
After humidification by steam 1.0132  0.02875
injection = 0.01816 kg vapour/kg dry air

© Copyright Reserved by Gateflix.in No part of this material should be copied or reproduced without permission
b) In the Room =37.06 KJ/kg dry air.
Pv2
w 2  0.622 
Pv  Pv2 Condition at outlet
Pv2 = 0.5 ×0.03166 Ts=14.25 0C
= 0.01583 bar hs =37.06 KJ/kg dry air
Ws=0.009 kg vapour/kg dry air
0.622  0.01583
W2  Room sensible heat
1.0132  0.01583 RSH = Ma Lp (T2-TS)
W2 = 0.00987 kg vapour /kg dry air =500×1.0216 (25-14.25
Specific humidity at A = 5491.1 KW
0.622  0.014016 Room latent heat
WA 
0.0132  0.014016 RLH = Ma (w2-ws)+2500
= 0.008725 kg vapor /kg dry air =500 (0.00987-0.009) ×2500
Specific humidity at inlet to cooling = 1087.5 kw
coil Load on cooling coil = ma (hx-hs)
Wx = 0.9 W2+0.1 W1 =500(54.43-37.06)
8685
= 0.9×0.009870 + 0.1×0.01816 =8685 kw = 3.5
=0.0107 kg vapour /kg dry air = 2481.43 tones
Condensate rate = (wx-ws) Ma
Enthalpy at X = (0.0107-0.009) ×500
hx=1.00×27+0.00107(25001.88+27) =0.85 kg vapour /sec
= 54.43 kj/kg dry air =3060 kg vapour/hr
Enthalpy at 2 Q.5 Calculate all the psychometric
H2=1.005×25 properties of air at 1 bar, 300C DBT
+0.00987(25001.88×25) and 250C WBT. The following
= 50.3 KJ/kg dry air properties of water may be
B.P.F. of cooling coil assumed :-
ts  ta
BPF 
(tx  ta)
ts  ta ts  12
0.15  
(tx  ta) 27  12
Ts  14.250C
ws-wa ws-0.008725
0.15= =
(wx-wa) 0.0107-0.008725 Solution:
Ws = 0.009 kg vapour/kg dry air The following expression may be
used, if necessary
Pv  Pv 
 P  Pw  DBT  WBT   1.8
2824  1.325(1.8  DBT  32)
DBT=250C, WBT=150C
P   Pv  
 P  Pw  DBT  WBT   1.8
2854  1.325(1.8tDBT  32)

 0.017307 
1  0.03166  25  15  1.8
Enthalpy at supply point 2854  1.325(1.8  25  32)
hs=1.005×14.25+0.009 17.43
(2500+1.88×14.25) v  0.017307 
2751.975

© Copyright Reserved by Gateflix.in No part of this material should be copied or reproduced without permission
Pv = 0.01097 bar

Pyschrometric properties:-

1. specific humidity
pv
W  0.622 
p  pv
0.01097
 0.622 
1  0.01097
W= 0.006899 kg vapor/kg of dry air

2. Relative humidity (ɸ)


Pv
 100
P  PV
0.01097
 100
0.03160
ɸ = 34.65%

3. Degree of saturation (µ)


P  PS
µ  ( )
P  PV
(1  0.03166)
 0.3465
(1  0.00109)
 0.96834 
  0.3465  
 0.9890 
µ = 0.339 or 33.9%

4) Enthalpy of moist air (h)


h = Cpa + W [2500 + 1.88]
where Cpa =1.005 kj/kg.k
h  1.005  25  0.006899  2500  1.88  25
h = 25.125+0.006899(2547)
h  42.696KJ / Kg of dry air .

© Copyright Reserved by Gateflix.in No part of this material should be copied or reproduced without permission
8 POWER PLANT ENGINEERING

Power plant engineering is the branch of area covered by 2 – 3 process on T-S


thermal science which deals with study of diagram is equal to the area covered by 4 –
chemical energy of fuel is converted into 1 process.
thermal energy and then utilises into the
mechanical work done in the efficient 8.2 ERICSON CYCLE
manner. When the thermal energy is Ericson cycle has two reversible isothermal
converted into the mechanical energy it is heat additions & heat rejection process and
impossible to convert whole amount of two reversible isobaric processes. It differs
heat into work. So efficiency play an from cannot cycle in that two isentropic
important role in power plant to check the processes of Carnot cycle are replaced by
performance of the power plant. The main two constant pressure process. P-V and T-s
important power cycles are gas power diagram of Ericson cycle is shown.
cycle and steam power cycle. Gas power
cycle works on Brayton cycle and steam
power cycle works on Rankine cycle.

8.1 STIRLING CYCLE

Stirling cycle has two reversible isothermal


heat additions & heat rejection process and
two reversible isocoric processes. It differs
from cannot cycle in that two isentropic
processes of Carnot cycle are replaced by
two constant volume process.

The efficiency of Stirling cycle is given as


T
η  1- 2
T1
Regenerative Ericson cycle has same
efficiency as that at cannot cycle. And both
Stirling and Ericson cycle has same
efficiency it temperature limits are same.
Gas turbine cycle with inter cooling,
reheating & regeneration closely resemble
The efficiency of Stirling cycle is given as the Ericson cycle. Both cycles are difficult in
T T practice because heat transfer at constant
η  1  3  1  min temperature requires either infinite long
T1 Tmax
surface area or infinite long time.
If cycle is regenerative cycle then it has
same efficiency as that of cannot cycle i.e. 8.3 GAS POWER PLANT

© Copyright Reserved by Gateflix.in No part of this material should be copied or reproduced without permission
In gas power cycle, the gas is a working The P-V and T-S diagram of Brayton cycle
fluid. It does not go under the phase change has been shown.
during the cycle. Gas power cycle works on
Brayton cycle. There are two type of cycle:
Open Cycle
In this cycle, the working fluid renew after
completion of the cycle.
Closed cycle
In closed cycle, chemical composition of
working fluid remains same as heat is
supplied through the heat transfer and not
by direct combustion.
Advantages of closed cycle over open
cycle
1) Use of high Pr. Throughout cycle The efficiency of Brayton cycle is given by
reduces size of plant Q C T  T 
η  1 R  1 P 4 1
2) Close cycle has higher thermal QS CP  T3  T2 
efficiency.
3) Elimination of possible of turbine blades T 
T1  4  1
T
 1  1 
erosion
------ (8.1)
4) Closed cycle can use any working fluid  T3 
or gas of higher value of 𝛾 to increases T2   1
the power output and efficiency.  T2 
process 1-2 and process 3-4 are isentropic
8.3.1 BRAYTON CYCLE process, so
γ 1
Brayton cycle consists of four processes: T2  P2  γ γ 1
T
Process 1–2: Isentropic Compression     r γ  3
T1  P1  T4
Process: In this process the heat transfer is
zero. Where r = compression ratio
dQ = 0 T3 T4
 ------ (8.2)
Process 2–3 : Constant pressure Heat T2 T1
addition Process: Heat addition in this substituting the values in equation 8.1, we
process is given by get
QS = m. CP T3 − T2 T
Process 3–4 : Isentropic Expansion Process: η  1 1
T2
In this process the heat transfer is zero.
dQ = 0 1
η  1
γ 1
Process 4–1 : Constant pressure Heat
r
rejection Process: The heat is reject in γ
constant pressure process in heat  Thus efficiency of Bray ton cycle
exchanger. Heat rejection in this process is depends upon compression ration and
given by nature of gas.
QR = m. CP T4 − T1

© Copyright Reserved by Gateflix.in No part of this material should be copied or reproduced without permission
8.3.2 ENERGY BALANCE EQUATION ON  TT 
COMBUSTION CHAMBER w net  CP T3  3 1  T2  T1 
 T2 
for optimum condition differentiating the
equation with respect to T2
dw net  TT 
 0  Cp 0  3 21  1  0
dt  T2 
Heat released due to combustion of fuel is
converted into enthalpy increases of gas in T2  T1T3
combustion chamber. T1T3
T4   T1.T3
Heat input = Enthalpy change of gas T1T3
m  g c pg T3  T2 
 f XCVXηc.c  m
T4  T1.T3
 f XCVXηc.c   m
m  f  cpg T3  T2 
 a m
Compression ratio for optimum work done
γ γ
8.3.3 ANALYSIS OF GAS TURBINE CYCLE P  T  γ 1  T  2 γ 1
r  2  2   3 
P1  T1   T1 
The upper limit of temperature is fixed due γ
to metallurgical condition of turbine  T  2 γ 1
blades. Though by increasing the Pressure r  3 
 T1 
 
 1  w net  CP T3  T1T3  T1T3  T1 
Ratio, net efficiency 1  γ 1 
can be
 Rp  γ  w net max  CP  T3  T1 
2
 
increased but net work will decrease. So
there is a optimum limit of the compression 8.3.4 REGENERATION IN GAS TURBINE
ratio at which optimum efficiency and work
output can be obtained. Efficiency of gas can be increased by
utilizing the energy of exhaust gases from
the turbine exit in heating air leaving the
comp. In heat exchanger, and this process
is called as regeneration.

Net work done in gas power plant is given


by
w net  w T  w C
 CP  T3  T4    T2  T1 
 CP  T3  T4    T2  T1  ----- (8.3)
form the process 1-2 and process 3-4
T2 T3 TT 8.3.5 EFFECTIVENESS OF HEART
  T4  1 3
T1 T4 T2 TRANSFER
substituting the value in the equation 8.3
It is ratio of actual temperature rise of air
to maximum possible rise of the

© Copyright Reserved by Gateflix.in No part of this material should be copied or reproduced without permission
temperature. It is also known as degree of 8.3.7 BRAY TON CYCLE WITH INTER
regeneration. COOLING
m a Cpa  Ta  T2 
 The efficiency of Brayton cycle can be
 m a  m f  Cpa  T4  T2  improved by use of stages compression
If the mass of air is higher as compare to with inter cooling.
mass of the fuel, so the effectiveness of heat
exchanger is 8.3.7.1 EFFECT OF INTER COOLING
Actualtemp.Rise T  T2
  a
max possibleRise T4  T2
m
1) Decrease in compressor work
2) No change in turbine work
8.3.6 EFFECT OF REGENERATION 3) Network increases

1) There is no change in turbine work.


2) There is no change in compressor work.
3) Net work done remains same
4) Heat Supplied is decreased so the
efficiency of the plant increase
Efficiency of ideal regeneration cycle
Q
η  1 R  1 b 1
T  T 
QS  T3  Ta 
for the perfect regeneration,
Ta  T4 , Tb  T2
Optimum Pressure Ratio for minimum
substituting the values in equation, we get
compressor work
T  T 
T1  b  1 T1  2  1 w C  w C1  w C2
T T
  1  1   1  1   CP  T2  T1   CP  T1  T3 
 T   T 
T3 1  a  T3 1  4  For perfect inter cooling T1  T3
 T3   T3 
 CP  T2  T1   CP  T4  T1 
 γ 1 
T1   r  γ  1 T T 
η  1  CP T1  2  4  2 
T3 1  r γγ1 
     T1 T1 
γ 1 γ-1 γ-1
T
ηreg  1  1  r  γ T2 P  γ T P  γ
T3 = i  , 4 = 2 
T1  P1  T1  Pi 
So as compression ration increases the
efficiency of simple Brayton cycle increases  P  η  P  η 
w C  CP T1  i    2   2
but the efficiency of regenerative Brayton  P1   Pi  
cycle will decreases.
Differentiating the equation with respect to
Pi, we get
pi  p1p2
substituting the value of pi in compressor
work for perfect inter cooling
 γ 1

  P  γ
w C1  CP T1  2
 1
 P  
 1  

© Copyright Reserved by Gateflix.in No part of this material should be copied or reproduced without permission
T  8.4 STEAM POWER PLANT
w C2  CP  T4  T3   CP T3  4  1
 T3  In steam power plant, the working
 γ 1
 substance recalculates in two phases liquid
  P2 γ
w C2  CP T1   1
 P 
and vapour. The components of simple

 1   vapour plant are shown in figure.

8.3.8 BRAY TON CYCLE WITH


REHEATING

The efficiency of Brayton cycle can be


increased by reheating the gas and passing
it through the number of the turbines.

Simple vapour plant works on Rankine


cycle. Heat is transferred to water in the
boiler from external source. On P-h and T-S
diagram, Rankine cycle can be shown.

Effect of reheating
1) Turbine work increases because const
Pressure Line diverge
2) No change in compressor work
3) Network done increases

Conditions for perfect reheating


1) T3  T5
2) Pi  P1P2
3) w T1  w T2
4) w T  2w T1
Rankine cycle consists of four processes:
8.3.9 ISENTROPIC EFFICIENCY OF Process1-2:IsentropicCompressionProcess:
COMPRESSOR AND TURBINE In this process woter is compressed
isentropically in feed pump. Pump work
Due to irreversibility turbine produce less can be given:
work as compare to ideal work and Wp = h2 – h1
compressor requires more work as Process 2 – 3 : Constant pressure Heat
compare to ideal work required for addition Process in the boiler: The heat is
compressor. added to water in the boiler. It is given as
efficiency of the compressor is given by QS  h 3 – h 2
Isentropic work h 2  h1 T2  T1
ηC   1  Process 3 – 4 : Isentropic Expansion
actual work h 2  h1 T21  T1 Process in the turbine: In this process the
efficiency of the turbine is given by heat transfer is zero and work done by the
actual work h h T T turbine is given as
ηT   3 14  3 41
Isentropic work h 3  h 4 T3  T4 WT  h 3 – h 4

© Copyright Reserved by Gateflix.in No part of this material should be copied or reproduced without permission
Process 4 – 1 : Constant pressure Heat the steam from turbine and water just
rejection Process in the condenser: The before entering the boiler. Regenerative
heat is rejected at constant pressure cycle can be shown on T-S diagram.
process in condenser. Heat rejection in this
process is given by
QR  h 4 – h1
The efficiency of Rankine cycle is given by
Q h4 – h1
η  1 R  1
QS h3 – h2

8.4.1 REHEAT RANKINE CYCLE


In reheating cycle, the expansion of steam
from the initial state at turbine is carried
out in two or more no. of re heater. The
network out increases and hence steam
rate decreases.
P  m  w net
Turbine work in regeneration is given by
w T   h1  h 2   1  x1  h 2  h3 
Energy balance equation on OFWH
xh 2  1  x  h 5  h 6

8.4.3EFFECT OF PARAMETERS ON
1) Work done by the turbine PERFORMANCE OF STEAM POWER PLANT
w T   h3  h 4    h5  h 6 
1) Effect of Boiler pressure If the boiler
2) Work done by the turbine pressure is increased, the effect on the
w p  h 2  h1  vdp performance can be checked with help of T-
3) Heat supplied in the boiler S diagram
QS   h 3  h 2    h 5  h 4 
wT  wp
η
Qs
 The main advantage of reheat factor is
to increase dryness fraction at the
turbine exit.
1) w p increases
8.4.2 REGENERATION IN RANKINE CYCLE
2) w T  same
In order to increase the mean temperature
3) QS Decreases
of the heat addition so that heat can be
transferred at higher temperature. So the 4) η increases
efficiency of the cycle can be increased. In 5) Dryness fraction at outlet of turbine
this cycle, it is possible by transferring the decreases
heat from vapour as it flows through the 2) Effect of Superheating
turbine to the liquid flowing around the If steam is superheated in the boiler, the
turbine. With the help of open feed water effect on the performance can be checked
heater (OFWH) heat is transferred between with help of T-S diagram

© Copyright Reserved by Gateflix.in No part of this material should be copied or reproduced without permission
Taking log in both sides
lg p  γlgρ  lg C
differentiating the equation, we get
dp dρ
γ 0
p ρ
 dp  γp
  
1) w psame  dρ S ρ
2) w T increases γρRT
C2 
3) w net increases ρ
4) QS increases So the velocity of sound in a medium is
given by
5) η iincreses
C  γRT
6) Dryness at the outlet of turbine
increases Where R is the specific Gas constant.
8.5.5 MACH NUMBER
8.5 NOZZLE AND DIFFUSER
Mach number is defined as the ratio of
8.5.1 NOZZLE actual velocity to the velocity of the sound
or sonic velocity.
It is a device of varying cross section used V
M
to convert the Pressure Energy of working C
fluid into kinematic energy. It is mainly #M>1, then V>C flow is supersonic flow.
used to produce the jet of steam or gas of # M = 1 , then V = C flow is Sonic flow.
high velocity to produce throat for Jet # M < 1 then V < C flow is Subsonic flow.
propulsion to derive steam turbine or gas # M>5 then V>5C flow is hypersonic flow.
turbine. 8.5.6 STAGNATION POINT AND
STAGNATION PROPERTIES
8.5.2 DIFFUSER
Stagnation point is state in the fluid flow
It is a device of varying cross section where velocity of flow is become Zero and
causing rise in Pressure Energy atthe cost kinetic energy converts into pressure
of kinematic energy. These are used in energy.
centrifugal and axial compressor. The Pressure, Density, temperature etc. of
 The smallest section of nozzle is known stagnation point are known as stagnation
as throat. Properties.
8.5.3 SPEED OF SOUND

It may be defined as velocity of Pressure


Wave in fluid medium.
 dp 
C  
 dp s
for the ideal gas at stagnation point,
8.5.4 VELOCITY OF SOUND IN IDEAL GAS
For an ideal gas, flow is isentropic V2
h0  h 
Pvγ  C 2
P V2
C CP TO  CP T 
ργ 2

© Copyright Reserved by Gateflix.in No part of this material should be copied or reproduced without permission
V2 dρ dA dV
TO  T  0  
2CP ρ A V
γR dA dρ dP
We know that CP   
γ 1 A ρ PV 2
V 2  γ  1 dA dV  dρ V 
 1 X
T0  T 
2  γR  A V  ρ dV 
Substituting the value of dV from equation
T0 V2
 1  γ  1 (8.7), we get
T 2γRT
dA dP  V 2 
T0
 1
 γ  1 . V2  1
A ρV 2  C2 
T 2 C2
dA dV
T0
 1
 γ  1 M2   M 2  1
A V
T 2
γ 1) When M < 1, flow is Subsonic flow
P0  T0  γ 1
   dA 
P T a) Converging passage   Ve 
 A 
8.5.7 FLOW THROUGH NOZZLE & DIFFUSER
Assumption for flow
1) flow is the isentropic flow.
2) Stagnation enthalpy is constant h 0  c

h0  h 
V2
     8.4  A  V  P  M   ρ  
2
 dA 
differentiating the equation b) Diverging passage   Ve 
dh  VdV ------ (8.5)  A 
From combined first and second law of
thermodynamic
TdS  dh  VdP
differentiating the equation, we get
dP
dh  VdP  -------- (8.6) A  V  P  M   ρ  
ρ
dP
or  Vd 2) When M > 1 flow is supersonic flow
ρ
 dA 
dρ a)   Ve 
 ρV  A 
dV
dP
dV   -------- (8.7)
ρV

Continuity equation for the mass flow


m  ρAV
taking login both the sides A  V  P  M   ρ  

logm  logρ  logA  logV
 dA 
differentiating the equation b) Diverging Passage    Ve 
 A 

© Copyright Reserved by Gateflix.in No part of this material should be copied or reproduced without permission
A compressor is a device in which work is
done on the gas to raise the pressure of the
gas. The work done by the compressor is
negative in the compression process
because the work is done on the system to
raise the pressure of the gas.

A  V  P  M   ρ   8.6.1 WORK DONE IN COMPRESSOR

Conclusion Process of compression in compressor is


a) For subsonic flow and compressible shown in the fig.
fluid convergent passage acts as a
nozzle and divergent passage acts as a 1) Work done in adiabatic process
diffuser.  γ 1

γ  P2  γ
b) For supersonic flow and compressible W P1V1    1

γ 1  P1  
fluid convergent passage acts as  
diffuser & divergent passage acts as a
nozzle.
c) For an incompressible fluid converging
passage always aits as nozzle and
divergent as a diffuser.
ρ dA dV
  0
ρ A V
dA dV 2) Work done in polytropic process
  n 1

A V n  P2  n
W P1V1    1

8.5.8 MASS FLOW RATE PER UNIT AREA n 1  P1  
 
mass flow rate per unit area is given by 8.6.2 ACTUAL DIAGRAM OF COMPRESSOR

m  2 n 1

 ρ1 2CP T1  r  n   r  n 
A2   The actual work done will be more due to
For maximum flow rate resistance offered by suction value and
discharge value. The shaded portion
d m  
 0 represents the extra work to counter value
dR  A 2  resistance. The actual diagram with the
n clearance volume is shown in fig.
P  2  n 1
r  2  
P1  n  1 
m
will be max when Area is minimum and
A
it occurs at throat. So the critical condition
occurs at throat. So Critical pressure is Fig.: Actual P-V diagram of compressor
given by
n
Pc  2  n 1 8.6.3 CONDITION FOR MINIMUM WORK
  REQUIRED IN MULTISTAGE
P1  n  1 
COMPRESSOR WITH PERFECT INTER
COOLING
8.6 COMPRESSOR For perfect inter cooling

© Copyright Reserved by Gateflix.in No part of this material should be copied or reproduced without permission
T1  T3 The velocity triangle is shown in the fig.
From the outlet of nozzle, steam flow at
P2  P1P3
absolute velocity V1. But due to blade
For multi stage compressor velocity u, steam strikes at the blade with
P2 P3 P4 Pn 1 relative velocity Vr1.
   C
P1 P2 P3 Pn Following are the parameters of the turbine.
PN 1 α=Angle at which steam enters or guide
 CN blade angle
P1
θ= Vane angle at inlet
ϕ= Vane angle at outlet
β= Angle at exit of moving blade

8.7 STEAM TURBINE

Steam turbine is a prime mover which is


used to convert the potential energy into fig.: velocity triangle
the kinetic energy and then utilize it into V1 &V2 = Absolute velocity at entrance and
the mechanical work. Main use of steam
exit
turbine is to produce the mechanical work
Vw1 &Vw 2 =Velocity of whirl at entry and at
in steam power plant.
exit or tan agent of absolute velocity
8.7.1 TYPES OF TURBINE Vr1 &Vr2  Relative velocity at entry and
exital blade
There are two types of turbines: Vf 1 &Vf 2  axial component of absolute
1) Impulse Turbine: In impulse turbine velocity
pressure drops in nozzles but remains u = blade velocity
same in moving blade.
2) Reaction Turbine: In reaction turbine 8.7.4 PERFORMANCE PARAMETERS OF
pressure drops in fixed blade and it also TURBINE
changes during the expansion of the
steam in moving blades. 1) Velocity Ratio
It is the ratio of blade velocity to the flow
8.7.2 COMPOUNDING IN TURBINE velocity at the inlet of the blade. It is given
by ρ.
In this type of compounding two shafts are u
used to drive separate generator at ρ
V1
different RPM. There are two type of
compound turbine:
2)Blade or Diagram efficiency
It is ratio of net work done by turbine to
1) Pressure compounding- Rateua turbine
kinetic energy at the inlet of the blade.
2) Velocity compounding - Curtis turbine
2u  Vw1  Vw 2 
ηD 
8.7.3 VELOCITY TRIANGLE OF TURBINE V12
If m a T2θ  ,Vr1  VR 2 thenVw1  V1 cos 

© Copyright Reserved by Gateflix.in No part of this material should be copied or reproduced without permission
Vw2  Vr 2 cos   u  Vr 2 cos   u u  Vw1  Vw 2 
ηstage 
 V1 cos  u   u  V1 cos α  2u  h1  h 2 
2u Vw1  Vw2  2u  2V1 cos cos α  u 
ηD  2
 Examples
V 1 V12
Q.1 Determine the minimum no. of stage
4ucos  u2 required in an air comp. which takes
ηD   4 2  4ρ cos α  4ρ2
V1 V1 air at 1 bar 270C and delivers 180
ηD  4ρ  cos α  ρ  --- (8.8) bar. The max discharge temperature
is 1500C and considers the index of
For maximum diagram efficiency, polytrophic as 1.25 and perfect inter
differentiating the equation with respect to ρ. cooling b/w stages.
dηD Solution:
 0  4cos α  8ρ
dp PN 1
 CN
cos α P1
ρ
2 1.25

Substituting the value of ρ in equation 8.8, Pn 1  T2  1.25


   5.57 N
get P1  T1 
cos   as   180
ηDmax  4  as     cos 2   5.57 N
2  2  1
ηDmax  cos 
2
N= 3
For n number of stages (Curtis stage)
cosα Q.2 A gas turbine plant operates on
ρopt  Brayton cycle, between Tmin =300K

and Tmax=1073K. Find the maximum
ηDmax  cos2 α work done per kg of air and cycle
efficiency. Compare the efficiency
3)Blade or diagram efficiency of with the efficiency of Carnot cycle
reaction turbine with same temperature limit.
2  2ρ cos α  ρ 2  Solution:
ηD 
 
2
1  2ρ cos α  ρ 2 Wnetmax  Cp TMax  TMin
For maximum diagram efficiency,
 
2
differentiating the equation with respect to  1.005  1073  300
, we get = 239.28 KJ/Kg
dηD Efficiency of the Brayton cycle
0
dρ 1
η  1  γ 1
ρ  cos α

Substituting the value of in equation of
diagram efficiency of reaction turbine, we TMin 300
 1  1  47%
get TMax 1073
2  2cos 2 α  Efficiency of the Carnot cycle
ηDmax 
1  cos 2 α T 300
ηC  1  min  1   72.1%
Tmax 1073
4) Stage efficiency η 0.47
It is the ratio of the net work done to   0.65
ηC 0.721
change of enthalpy at the inlet of the
turbine.

© Copyright Reserved by Gateflix.in No part of this material should be copied or reproduced without permission
Q.3 A gas turbine plant draws in air at Maximum possible heat from heat
1.013 bar, 10°C and has a pressure exchanger = Cp(T5 –T2)
ratio of 5.5. the maximum Actual Heat transfer = 0.7Cp(T5 –T2)
temperature in the cycle is limited
= 132 KJ/kg of air
to 750°C. Compression is conducted
in rotary compressor having an Cp(T3-T2) = (1+ m)×132
isentropic efficiency of 82%, and CpT3 = 634.43 +132.33 m
expansion takes place in a turbine m = 8.68 × 10–3 kJ/kg of air
with an isentropic efficiency of 85%. Heat addition (Q1)=Cp(T4-T3)=CpT4-
A heat exchanger with an efficiency CpT3
of 70% is fitted between the = 393.7 –132.33m
compressor outlet and combustion
= 392.6 kJ/kg of air
chamber. For an air flow of 40 kg/s,
find: WT = (1 + m) (h4 –h5)
a) The overall cycle efficiency, = (1 + m) Cp (T4 – T5’)
b) The turbine output, and =1.00868 × 1.005 × (1023 – 687.7)
Solution: kJ/kg of air
P1=101.3 KPa, T1=283 K, r=5.5, = 340 kJ/kg
T4 = 750℃ = 1023 K Wc=(h2–h1)=Cp(T2‘–T1)=1.005×
γ 1
(499.6 – 283)
T2  P2  γ γ 1 1.4 1
     r  γ   5.5 1.4 = 217.7 kJ/kg of air
T1  P1 
Wnet = WT - Wc = 122.32 KJ/kg
T2 = 460.6 K
a)   Networkdone  122.32  31.16%
Isentropic efficiency of compressor Heatsupplied 392.6
Isentropic work h 2  h1 T2  T1 b) Turbine output = (WT) = 122.32
ηC   1 
actual work h 2  h1 T21  T1 kJ/kg of air= 4893 kW
T  T 460.6  283
0.82  21 1 
T2  T1 T21  283 Q.4 A simple steam power cycle uses
T2’ = 499.6K solar energy for the heat input.
γ 1 1.4 1
Water in the cycle enters the pump
T4
  r  γ   5.5 1.4 as a saturated liquid at 40°C, and is
T5 pumped to 2 bar. It then evaporates
T5 = 628.6 K in the boiler at this pressure, and
Efficiency of turbine enters the turbine as saturated
actualwork T T vapour. At the turbine exhaust the
ηT   4 51 conditions are 40°C and 10%
Isentropicwork T4  T5 moisture. The flow rate is 150 kg/h.
1023  638.6 determine:
0.85 
1023 T51 a) the turbine isentropic efficiency,
b) The net work output
c) The cycle efficiency
Solution
From Steam Table
T1 = 120.23°C = 393.23 K, h3 =
2706.7 kJ/kg, s3 = 7.1271 kJ/kg – K
At 40°C saturated pressure 7.384
T5’ = 687.7 K kPa

© Copyright Reserved by Gateflix.in No part of this material should be copied or reproduced without permission
hf=167.57KJ/Kg, hfg=2406.7 KJ/Kg KJ
sf  0.4764 .K sfg  7.9187KJ / Kg.K
sf = 0.5725, sg = 8.2570 Kg
h4’ = hf + 0.9 × 2406.7 vf = 0.001005 m3/ Kg
= 2333.6 kJ/kg At 8 bar
hf = 721.11KJ/Kg hfg = 2048KJ/Kg,
KJ
sf  2.0462 .Ksfg  4.6166
Kg
KJ/Kg.K
vf = 0.001115 m3/ Kg
Regeneration cycle can be shown on
For h4s if there is dryness fraction x
T-s diagram
7.1271=0.5725+x×(8.2570– 0.5725)
x = 0.853
h4 = 167.57 + 0.853 × 2406.7
= 2220.4 kJ/kg
Isentropic efficiency,
h3 – h4 2706.7 – 2333.6
isentropic  
h3 – h4' 2706.7 – 2220.7
= 76.72% Process 1-2 is isentropic process
Net Turbine work output WT=h3 – h4 s1 = s2 , h1 = 3675.2
= 373.1 kJ/ kg 6.6502 = 2.0462 + 𝑥2 4.6166
Pump work, WP = v (P1 – P2) 𝑥2 = 0.997
= 1.008 × 10–3 (200 – 7.384) kJ/kg Enthalpy of point 2
= 0.1942 kJ/kg h 2  h f 2  x 2 h fg2  721.11  0.997  2048
Power = m × (W T - Wp) = 15.55 kW
h 2  2763.54kJ / kg
h3=2706.7kJ/kg, h2=167.76 kJ/kg
Q1=(h3–h2)=(2706.7 – 167.76) kJ/kg Process 1-3 is isentropic process
= 2539 kJ/kg 𝑠1 = 𝑠3
W  WP 373.1  0.1942 6.6502 = 0.4764 + 𝑥3 𝑋 7.9187
cycle  T   14.69 % 𝑥3 = 7796
Q1 2539
Enthalpy of Point 3
Q.5 A regeneration Rankin cycle as h3  h f 3  x3h fg 3
steam entering turbine at 200 bar  137.82  0.7796 X 2423.7
and 250℃ superheated state and  2027.45kJ / kg
leaving at 0.05 bar. Considering feed
Work done by pump 1
water to be open type. Determine
the efficiency of plant when feed w p1 =h5 -h 4 =vdp=0.001005 8-0.05×102
water is operating at 8 bar. =0.798
Solution: h5 =0.798+137.82=138.618kJ/kg
At 200 bar and 250℃
Work done by pump 1
h=3675.2KJ/Kg, s=6.6502KJ/Kg.K,
𝑣𝑓 = 0.001005 m3/Kg w p2 =h 7 -h 6 =0.001115  200-8 X102
At 0.005 bar hf = 137.82 KJ/Kg, = 22.08 𝑘𝐽/𝑘𝑔
hfg = 2423.7KJ/Kg, h 7 =h 6 +22.08=723.19kJ/kg

© Copyright Reserved by Gateflix.in No part of this material should be copied or reproduced without permission
Energy Balance equation at open 0.2 M 4 + M 2 − 0.645 = 0
feed water heater Solving the equation, we get
x  h 2  1  x  h 5  h 6 M = 0.76
T0 (γ  1)M 2 (1.4  1)  0.762
x  2763.54  1  x  138.618  721.11  1  1
T 2 2
x = 0.2919 T = 277.78K
w net   h1  h 2   1  x  h 2  h 3  C= γRT
  w p1  w p2  = 1.4  0.287  277.78 103
= 334.08 m/sec
  3675.2  2763.54   1  0.2919 
V = c × M = 334.08×0.76 = 253.9 m/s
 2763.54  2027.45  (798  22.08) γ 1 1.4 1
P0  T0  γ  310  1.4
 1462.4KJ / kg    
P T  277.78 
Qs =h1 -h 7 =3675.2-723.19
P0 = 40 × 1.468 = 58.72 KPa
= 2932.78 kJ/kg
Efficiency of the plant Q.7 A simple open cycle gas turbine has
w a compressor turbine and a free
η= net
Qs power turbine. It develops electrical
1462.4 power output of 250 MW .the cycle
  49.86% takes in air at 1 bar and 288 K. The
2932.78
total compressor pressure ratio is
Q.6 A stream of air flow in a duct of 14. The turbine inlet of compressor
100mm diameter at the rate of 1 is 1500 K. The isentropic efficiency
kg/sec. The stagnation temperature of compressor and turbine is 0.86
is 370C. At one section of the duct and 0.89 respectively. The
the static pressure is 40 KPa. mechanical efficiency of each shaft is
Calculate the Mach number, velocity 0.98. Combustion effect is 0.98
and stagnation pressure at that while combustion pressure loss is
section. 3% of compressor delivery
Solution: pressure. Alternator efficiency is
T0 = 37+ 273 =310 K, P= 40 KPa, and 0.98. Take calorific value of fuel is
γ = 1.4 42000 KJ/kg. Cpa = 1.005 KJ/Kg K
The mass flow rate per unit area is and Cpg = 1.15 KJ/Kg. K. Calculate
m P i) Air –fuel ratio
 ρV   M γRT ii) Specific work output
A RT
Solution:
γ PM T0 γ PM (γ-1)M 2 Electrical power =250mw
= × = × 1+
R T0 T R T0 2 Power output from power turbine
250
1 1.4 40M (1.4-1)M 2 
= × 1+ alternator efficiency  mechanical efficiency
π0.12 0.287 310 2 250
4  = 260.3082 MW
0.98  0.98
1.4 = 260308.2 KW
127.39   40 103 M 1  0.2M 2
0.287 10  310
3
0.4
 288 14   612.15 K
0.803 = M 1  0.2M2 1.4
 1 T 21  T1)
Squaring both the sides T2’  T1 14  T2  T2 
0.645 = M2 (1  0.2M2 )  ηc

© Copyright Reserved by Gateflix.in No part of this material should be copied or reproduced without permission
(1+0.0268) (1500 - T5 ) = 336.12
T5 = 1500 – 327.35
T5 = 1172.65 K
Ma (1+0.0268) (T5 − 869) = 226355
Ma = 725.987
Ma = 726 kg/sec
Special output =(1+F) Cpg(T5 − T6 )
= 1.0268 1.15 (1172.65 – 869)
= 358.56 KW/Kg

612.15  288 Q.8 In a multi-stage parson’s reaction


 288   664.92K
0.86 turbine at one of the stages the rotor
Compressor is run by compressor diameter is 125 cm and speed ratio
turbine 0.72. The speed of the rotor is
MaCpa(T2 - T1) = ma (1 + f) (T4 –T5 ) 3000rpm. Determine
cpg × 0.98 1) The blade inlet angel if the blade
1.005 × (664.92-288) = (1+t) (tut5) × outlet angle is 22°
1.15 × 0.98 2) Diagram efficiency
250 329.4 Solution:
1  f  T4 – T5  
Parson’s reaction turbine
0.98 xo.98 0.98
(1+F)(1500-T5) = 336.12 --------- (1) D1=1.25m, N=300rpm
Total efficiency of turbine = 0.89 ρ = u/v1=0.72m, β1=?, β2=22
T41.33 T61.33 For parson’s reaction turbine
 α1 =β2 and α2= β1
P41.331 P61.331
π  D  N π 1.25  3000
0.33 u 
 P1  1.33 60 60
T61  T4  6   791k = 196.34 m/s
 P4  V1 = u/V1 = u/0.72 = 272.69 m/s
T T 1500  T6 Vr2 = V1 = 272.69 m/s
0.89  4 61  =
(T6  T6 ) 1500  791 Inlet velocity triangle and Outlet
T6 = 1500 – 0.89 (1500 – 791) velocity triangle
T6 = 869K
Power output from power turbine,
Ma (1+F) Cpg (T5 − T6 ) = 260308.2
Ma 1  F 1.15  T5  869  260308.2
Ma (1+F) (T5 − 869) = 226355
Heat balance equation in turbine,
Ma 1  F CpgT4  Ma CpaT2 
Ma CV  combustion effiency 1  F
1.15 1500 –1.005  664.92  41160 f
1725 – 668.25 = (41160 – 1725) F
F = 0.0268
1 Inlet velocity triangle outlet velocity
i) Air fuel ratio = = 37.32
F triangle
ii) Specific work output Vw1 = 272.69 cos20 = 252.83 m/s
Substituting the value of F in Vf1= 272.69 sin20 = 102.15 m/s
equation 1

© Copyright Reserved by Gateflix.in No part of this material should be copied or reproduced without permission
Vf1 102.15
tan 1  
vw1  v1 252.83  196.34
So, β1=61.05 and α2=β1-61.05
diagram efficiency
power
d 
1
m[V12  V22  Vf12 ]
2
power = mu (vw1+vw2) Inlet velocity triangle Outlet velocity
u = 196.34 m/s triangle from velocity triangle
vw1 = 252.83 m/s Vw1= v1 cosα= 550 cos17 =525.96
From outlet velocity triangle m/s
vr2 cos β2 = u+Vw2 Vf1= v1 sinα = ssosinl7=160.80 m/s
272.69 cos 22=196.34+Vw2 vf1
tan1 
Vw2=56.49 m/s vw1  v1
Power = 1 × 196.34 × (25 + 56.49) 160.80
= 60.73KW tan1 
525.96  180
60 103 β1 = 24.92
d 
1 vf 2
m[272.692  272.692  102.152 ] Tan2 
2 v2
 0.90or 90 % vf1 160.80
tan2  
u 180
Q.9 A de level turbine has a mean blade β2 = 41.27°
speed of 180 mps. The nozzles are Power output = m (Vw1+Vw2) u
inclined at 170 to the tangent. The vw1 =. 525.96 m/s
steam flow velocity through the vw2=0 (due to axial outlet condition)
nozzle is 550 m/sec. For a mass flow u = 180 m/s
of 3300 kg/hour and for axial exit 3300
condition: Find P  525.96  0  180
3600
1) The inlet and outlet angles of the
P = 86.78 KW
blade system.
iii) Diagram efficiency
2) The power output.
2(vw1  vw2)  u
3) Diagram efficiency.  100
Solution: V12
u= 180 m/s 2  (525.96  0) 180
 100
Nozzle angle αͦ = 170, Mass flow rate (550)2
= 3300 kg/hr = 0.9167 kg/sec = 62.59 %
V1 =550 m/s
Inlet velocity triangle and outlet Q.10 A single acting two-stage air
velocity triangle compressor deals with 4 m3/min of
air at 1.013 bar and 15℃ with a
speed of 250 RPM. The delivery
pressure is 80 bar. Assuming
complete inter -cooling, find the
power required by compressor and
bore and stroke of the compressor.
Assume apiston speed of 3 m/s,
mechanical efficiency 75% and
volumetric efficiency of 80% per

© Copyright Reserved by Gateflix.in No part of this material should be copied or reproduced without permission
stage. Assume the polytropic index
of compression in both stage to be n
= 1.25 and neglect clearance.
Solution:
Intermediate pressure for perfect
intercooling
P2 = P1 P3 = 1.013 × 90 = 9 bar
Minimum power required by
compressor
 n 1

2n  
P2 n 1
W= P1V1    1 

n 1  P  η
 1   mech
 1.251

2 1.25 1.013 100 4  9  1.25 
     1
1.25  1 0.75 60  1.013  
 
1013  4
  0.548  49.34 KW
45
L is the length of stroke of the piston
N
2L  3 m / s
60
90 100
L  36 cm
250
Effective swept volume Vs = 4/ 250
= 0.016 m3
π
×DLP 2 ×L×ηVol =0.016
4
0.016  4
DLP 
3.14  0.36  0.8
= 0.266 m = 26.6 cm
Ideal gas equation
P1V1 P3V3

T1 T3
for perfect intercooling, T1 = T3 So.
V1 P3

V3 P1
π
 DLP 2  L
4 9

π
 DHP 2  L 1.013
4
1.013
DHP  0.266
9
= 0.89 m = 89 cm

© Copyright Reserved by Gateflix.in No part of this material should be copied or reproduced without permission
9 INTERNAL COMBUSTION ENGINE

9.1 HEAT ENGINE due to high temperature in the combustion


chamber caused by high compression.
Heat engine is a device which transforms
the chemical energy into heat energy and 9.2 APPLICATIONS OF IC ENGINES
the utilized it into mechanical work in an
efficient manner. Thus the thermal energy Mainly used as ‘prime movers’, e.g. for be
is transforms into mechanical energy into the propulsion of a vehicle car, bus, truck,
heat engine. There are two types of heat locomotive, marine vessel, or airplane.
engine: Other applications include stationary saws,
lawn mowers, bull-dozers, cranes, electric
generators, etc.

9.3 CLASSIFICATIONS OF IC ENGINES

IC engines can be classified according to:


1. Number of cylinders – 1, 2, 3, 4, 5, 6 to
16 cylinder engines.
The internal combustion engine is a heat 2. Arrangement of cylinders – Inline, V-
engine that converts heat energy (Chemical type, Flat type, etc.
energy of a fuel) into mechanical energy 3. Type of cooling – Air-cooled, Water-
inside the cylinder of the engine (usually cooled, etc.
made available on a rotating output shaft). 4. Number of strokes per cycle – 2-stroke,
The External combustion engine is a heat 4-stroke engines.
engine that converts heat energy (Chemical 5. Method of ignition – Spark Ignition (SI),
energy of a fuel) into mechanical energy Compression Ignition (CI).
outside the cylinder of the engine. The 6. Primary mechanical motion –
internal combustion engine can be Reciprocating, rotary.
categorized into two types:
9.4 BASIC COMPONENTS OF I.C. ENGINE
1) Spark Ignition (SI) Engine

An SI engine starts the combustion process


in each cycle by use of a spark plug. The
spark plug gives a high voltage electrical
discharge between two electrodes, which
ignites the air fuel mixture in the
combustion chamber surrounding the plug.
In early engine development, before the
inventor of electric spark plug, many forms
of torch holes were used to initiate
combustion from an external flame.

2) Compression Ignition (CI) Engine


The combustion process in a CI engine
starts when the air-fuel mixture self-ignites Fig: Internal Combustion Engine

© Copyright Reserved by Gateflix.in No part of this material should be copied or reproduced without permission
Engine components: crankshaft of the engine. The purpose of
The following is the list of major the flywheel is to store energy and furnish
components found in most reciprocating a large angular momentum that that keeps
internal combustion engines the engine rotating between power strokes
Block: Body of engine containing the and smoothes out engine operation.
cylinders made of cast iron or aluminium. Fuel pump: Electrically or mechanically
In many older engines the valves and the driven pump to supply fuel from the fuel
valve ports were contained in the block. On tank to the engine.
air cooled engines the exterior surface of
the block has cooling fins. 9.5 TERM SUSEDIN INTERNAL
COMBUSTION ENGINE
Camshaft: Rotating shaft used to push
open valves at the proper time in the i) Top-Dead-Centre (TDC): Position of
engine cycle either directly or through the piston when it stops at the furthest
mechanical or hydraulic linkage. Most point away from the crankshaft.
modern automobile engines have one or ii) Bottom-Dead-Centre (BDC): Position
more camshafts mounted in the engine of the piston when it stops at the point
head. closest to the crankshaft.
Carburettor: Venturi flow device that iii) Bore: Diameter of the cylinder or
meters the proper amount of fuel into the diameter of the piston face, which is the
air flow by means of pressure differential. same minus a very small clearance.
Catalytic converter: Chamber mounted in iv) Stroke: Movement distance of the
exhaust flow containing catalytical material piston from one extreme position to the
that promotes reduction of emission by other: TDC to BDC or BDC to TDC. It is
chemical reaction. denoted by L.
Choke: Butterfly valve at carburettor v) Clearance volume: Minimum Volume
intake, used to create rich fuel-air mixture in the combustion chamber with piston
in intake system for cold weather starting. at TDC. It is given by Vc.
Combustion chamber: The end of the
cylinder between the head and the piston vi) Displacement volume: Volume
face where the combustion occurs. displaced by the piston as it travels
through one stroke. It is also known as
Piston: The cylindrically shaped mass that swept volume (Vs).
reciprocates back and forth in the cylinder
transmitting the pressure forces in the vii) Air Fuel Ratio: It is the ratio of mass
combustion chamber rotating the crank shaft. air to mass of fuel input into engine.
The top of the piston is called crown and
the sides are called skirt. Pistons are made viii)Stroke to Bore Ratio: It is the ratio of
of cast iron Aluminium or steel. the stroke length to the diameter of the
Piston rings: Metal rings that fit into cylinder.
circumferential groups around the piston L
Stroke to bore ratio=
and form a sliding surface against the d
cylinder walls. The purpose of the rings is L
to form a seal between the piston and  If  1 It is known as Square Engine
d
cylinder walls. L
  1 It is known as Under Square
Flywheel: Rotating mass with large d
moment of inertia connected to the engine

© Copyright Reserved by Gateflix.in No part of this material should be copied or reproduced without permission
L efficiency.
  1 It is known as Over Square
d actually thermal eff.
ηr 
engine Air  standard efficiency
ix) Compression ratio: It is the ratio of the
total volume to clearance volume. 6) Mean effective pressure: M.E.P. is
V V  VS V average pr. inside the cylinder of an
rP  T  C  1 S
VC VC VC internal combustion engine on power
output
9.6 DIFFERENCE BETWEEN FOUR STROKE P X A X L X NXK
I.P.  im watt
AND TWO STROKE ENGINE 60
N
Where N  for4  stroke engine and
2
N  N for 2 – stroke engine the mean
effective pressure can be given as
area of indicator diagram
Pim 
Length of indicator diagram

9.8AIR STANDARD CYCLE AND EFFICIENCY


9.7 PERFORMANCE PARAMETERS The analysis of actual cycle is very
complicated. So the analysis of cycle is done
1) Indicated thermal efficiency: it is the on the basis of air standard cycle. The air
ratio of the energy in the indicated standard cycle is based on such
power to input fuel energy in the assumptions:
appropriate unit. I. The working medium is perfect gas.
I.P.(KJ / Sec)
ηith  II. There is no change of the mass of the
energy of fuel / sec working fluid.
III. All the processes in the cycle are
2) Brake thermal efficiency: it is the reversible processes.
ratio of the energy in the brake power
to input fuel energy in the appropriate 9.8.1 OTTO CYCLE
unit.
B.P.(KJ / Sec) Nicolaus Otto proposed a constant volume
ηith  heat addition and rejection process in place
energy of fuel / sec
of isothermal process of Carnot cycle. Otto
cycle is used now a day in spark ignition
3) Mechanical efficiency: it is the ratio of
engine.
the energy in the brake power to
4-stroke petrol engine operates on air
indicated power in the appropriate unit.
standard Otto cycle. It completes the Otto
B.P..(KJ / Sec)
ηm  cycle in 4 strokes (4 TDC to BDC movements
I.P..(KJ / Sec) of the piston). In 4 stroke crank shaft
rotates 7200.
4) Friction Power: friction power is the The four stroke of four stroke engine are:
difference between indicated thermal 1) Suction Stroke,
power and mechanical power or brake 2) Compression Stroke,
power. 3) Power Stroke,
F.P. =I.P.-B.P. 4) Exhaust Stroke.
Otto cycle shown below consists of four
5) Relative efficiency: it is the ratio of the processes:
actual thermal efficiency to air standard

© Copyright Reserved by Gateflix.in No part of this material should be copied or reproduced without permission
Process 1–2: Isentropic Compression process 1-2 is isentropic process so for the
Process: In this process the heat transfer is isentropic process,
γ 1
zero. dQ = 0 T2  V1 
    r
γ 1
Process 2 – 3 : Constant Volume Heat
T1  V2 
addition Process: The fuel and air mixture
is burnt at constant volume process. Heat Process 3-4 is the isentropic process so for
addition in this process is given by the isentropic process,
QS  m.Cv  T3  T2 
γ 1
T3  V4 
    r
γ 1

Process 3–4 : Isentropic Expansion Process: T4  V3 


In this process the heat transfer is zero. V1 V4
dQ = 0  r
V2 V3
Process 4–1 : Constant Volume Heat
rejection Process: The heat is reject in T2 T3

constant volume process. Heat rejection in T1 T4
this process is given by Substituting the value in equation 9.1
QR  m.Cv  T4  T1   T4 
  1
P-V and T-S diagram for Otto engine is T1  T1  T
shown in the fig. η  1  1 1
T2  T3  T2
  1
 T2 
1
  1  1 ------------ (9.2)
r
The thermal efficiency is the function of the
compression ratio and the ratio of specific
heat γ. if the is considered as constant the
efficiency depends upon the compression
ratio. The effect on efficiency of the Otto
cycle can be understood with the help of
the efficiency vs. compression ratio.

fig: Otto cycle: P-V and T- S diagram


Efficiency of the cycle is given by
Q
η  1 R
QS
m.CV  T4  T1 
 1
m.CV  T3  T2  fig: efficiency vs. compression curve

 1
 T4  T1  9.8.2 DIESEL CYCLE
 T3  T2  In actual spark ignition engine, upper limit
T  of compression ratio is limited due to self-
T1  4  1
T ignition temperature of the petrol. This can
 1  1  ----- (9.1) be overcome by compressing the air
T 
T2  3  1 separately. In diesel cycle fuel is burnt at
 T2  constant pressure. Other three processes
are the same processes as in Otto cycle.

© Copyright Reserved by Gateflix.in No part of this material should be copied or reproduced without permission
Diesel cycle shown below consists of four V4
Expansion ratio re 
processes: V3
Process 1–2: Isentropic Compression V4 V2
Process: In this process the heat transfer is  
zero. V3 V2
dQ = 0 V V r
 1 2 
Process 2 – 3 : Constant pressure Heat V2 V3 ρC
addition Process: The fuel is burnt at r  pC  re
constant pressure process. Heat addition in
Substituting the values of the temperature
this process is given by
in term of the compression ratio and cut off
QS  m.CP  T3  T2 
ratio, we get efficiency of diesel engine
Process 3–4 : Isentropic Expansion Process:
In this process the heat transfer is zero. 1  ρcγ  1 
ηD  1    ----------- (9.4)
dQ=0 r γ 1  γ  ρc  1 
Process 4–1 : Constant Volume Heat
rejection Process: The heat is rejected in The bracket term is always greater than
constant volume process. Heat rejection in one, so efficiency of diesel cycle is always
this process is given by less than efficiency Of Otto cycle for given
QR  m.Cv  T4  T1  compression ratio. The normal range of
compression ratio in petrol engine is from
P-V and T-S diagram for diesel engine is
6–10 and in diesel 15–20. So efficiency of
shown in the fig. diesel engine is more than efficiency of
gasoline engine.
9.8.3 DUAL CYCLE
Dual cycle is also called as mixed cycle or
limited pressure cycle. It is the compromise
between the Otto cycle and Diesel cycle.
Because for chemical reaction some time is
required to so combustion does not occurs
at constant volume and due to rapid
uncontrolled combustion in diesel engines,
combustion does not occur at constant
pressure.
Dual cycle shown below consists of five
Efficiency of the cycle is given by processes:
Q Process1-2: Isentropic Compression Process:
η  1 R In this process the heat transfer is zero.
QS
dQ = 0
m.CV  T4  T1  Process 2–3 : Constant volume Heat
ηD  1  
m.CP  T3  T2  addition Process: The fuel is burnt at
constant volume process. Heat addition in
1 T4  T1 
1      (9.3) this process is given by
γ  T3  T2  QS1  m.CV  T3  T2 
V1
Compression Ratio r  Process 3–4: Constant pressure Heat
V2 addition Process: The fuel is burnt at
V3 constant pressure process. Heat addition in
Cut off ratio ρC  this process is given by
V2

© Copyright Reserved by Gateflix.in No part of this material should be copied or reproduced without permission
QS2  m.CP  T4  T3  Substituting the values of temperatures in
Process 4–5 : Isentropic Expansion Process: the term of compression ratio, pressure
In this process the heat transfer is zero. ratio and cut off ratio, we get
dQ = 0 Efficiency of Dual cycle
Process 5–1: Constant Volume Heat 1  rpρcγ  1 
ηDUAL  1  γ 1   --- (9.5)
rejection Process: The heat is rejected in r  (rp  1)  γrp  ρc  1 
constant volume process. Heat rejection in
this process is given by 9.9 COMPARISON OF OTTO, DIESEL, DUAL
QR  m.Cv  T5  T1  CYCLE
P-V and T-S diagram of Dual cycle is shown
in the fig. 1) Same comp. ratio and heat addition
Otto cycle 1-2-3-4, Diesel cycle 1-2-3’-4’
and Dual cycle 1-2-2’-3”-4” are shown
on P-V and T-S diagram. From the T-S
diagram it is clear that heat input i.e.
area of T-S curve along S- axis is same.

V
Efficiency of the Dual cycle is given by
Q
η  1 R
QS
m.cv  T5  T1  But the heat rejected in Otto cycle is
 1 minimum and in Diesel cycle is
m.cp  T4  T3   m.c v  T3  T2  maximum So
 1
 T5  T1  η  1 R
Q
γ  T4  T3    T3  T2  QS
V1 So the efficiency of Otto cycle is
Compression Ratio r  maximum and the efficiency of diesel
V2
cycle will be minimum in this case.
P3
Pressure ratio rp  otto  Dual  Deisel
P2
V4 2) For Same compression Ratio and
Cut off ratio ρC 
V3 heat Rejection
V5 V5 V3 Otto cycle 1-2-3-4, Diesel cycle 1-2-3’-4
Expansion ratio re    and Dual cycle 1-2-3”-4”-4 are shown
V4 V4 V3
on P-V and T-S diagram. From the T-S
V1 V3 r
   diagram it is clear that heat rejection i.e.
V2 V4 ρC area of T-S curve in process 4-1 along S-
r  c  re axis is same.

© Copyright Reserved by Gateflix.in No part of this material should be copied or reproduced without permission
V3 1.95VC
ρ   1.95
V2 VC
 ρCγ  1 
1
η  1  γ 1  
 r   γ  ρC  1 
1  1.951.4  1 
 1    64.9%
 20  1.4 1.95  1 
4

Q.2 4-Cylinder, 2-stroke IC engine has


the following particulars: engine
speed = 3000 rpm, bore = 120 mm,
crank radius = 60 mm, mechanical
efficiency = 90% and the engine
But the heat supplied in Otto cycle is develops 75 BHP. Calculate the
maximum and in Diesel cycle is swept volume and mean effective
minimum So pressure (MEP).
QS Deisel  Qsdual  QS otto
Solution:
Q Mechanical efficiency,
and η  1  R
QS BrakePower bhp

So the efficiency of Otto cycle is Indicated power Ihp
maximum and the efficiency of diesel 75
cycle will be minimum in this case. 0.9  , So, P = 83.33 hp
P
otto  Dual  Deisel We know that
Engine Power, P = T ω
Examples Here, P = 83.33 HP = 83.33 x 746 W
Q.1 A Diesel engine has a compression = 62166.67 W
ratio of 20 and cut off takes place at 2πN 2  π  3000
 
5% of stroke find air standard 60 60
efficiency γ  1.4 = 314.16 rad/s
T =P/ ω = 62166.67/314.16
Solution: = 197.88 N.m
Mean Effective Pressure
(MEPorPmep)
2πN CT
MEP 
Vd
Vs = K.(π/4).d2. L
K = No. of the cylinder
Stroke L = 2 x crank radius
V1
r  20, V1  20V2  20V2 = 2 x 0.06 m
V2 = 0.12 m
VS  19V2  19VC 
Vd  4   0.122  0.12  5.43 103 m3
V3  0.05XVS  VC 4
 0.05X19VC  VC Nc = No. of cycle per power stroke =
1 for a 2-stroke engine
V3  1.95VC Therefore,

© Copyright Reserved by Gateflix.in No part of this material should be copied or reproduced without permission
2π 1197.88 a 10 minutes test, the dynamometer
MEP 
0.00543 reads 45kg and engine consumed 5
=228971.77 N/m2 kg of petrol of CV 45 MJ/kg. The
= 228.97 KPa carburettor receives air at 29 0 C and
1 bar at the rate of 10kg/min.
Calculate:-
Q.3 Two engines are to operate on Otto 1) Brake power
and Diesel cycles with the 2) Brake mean effective pressure
following data: Maximum 3) Brake specific fuel consumption
temperature 1400 K, exhaust 4) Brake specific air consumption
temperature 700 K. State of air at 5) Brake thermal efficiency
the beginning of compression 0.1 6) Air full ratio
MPa, 300 K. Estimate the
Solution:
compression ratios, the maximum
pressures, efficiencies, and rate of 6 cylinders, 4 strokes
work outputs (for 1 kg/min of air) Bore =10cm =0.1m
of the respective cycles. Stroke=0.12m
Solution: n=6, N=4800rpm
Arm of dynamometer=55cm= 0.55m
T3 = 1400 K
Dynamometer reads 45 Kg in 10min
T4 = 700 K
P1= 100 KPa 1. Brake power
T1 = 300 K B.P. = T×ω = 2πNT/60
RT 0.287  300 T= F r = 45× 9.81 × 0.55
v1  1 
P1 100 = 242.79 Nm.
= 0.861 m3/kg B.P. = 2×3.14×4800×242.79/60
γ 1 = 122.039KW
γ 1
T3  P3  γ V  2. Brake mean effective pressure
   4  pm  l  a  N 
T4  P4   V3  Brake power =   n
γ 1 60  2 
1400  V1 
  2 122.039 10 10 10  60  2
700  V2  Pm 
π 
γ 1 0.12    0.1 0.1  4800  6
T2  V1  4 
  2
T1  V2   5.39bar
3. Brake specific fuel consumption.
T2  2  300  600K
mf
V 1 Bsfc 
rc  1   2  γ 1  5.657 B.P.
V2 5
Work done W= Q1–Q2=Cv (T3–T2)– mf   60
10
Cv (T4 – T1) = 30kg/hr
=0.718[(1400–600)–(700– 30
300)]KJ/Kg 
= 287.2 KJ/Kg. 122.039
= 0.245kg/KWh
Q.4 A six cylinder ,four stroke spark
ignition engine of 10cm 12cm 4. Brake specific air consumption
ma
(bore stroke) with a compression bsac 
ratio of 6 is tested at 4800 rpm-on a bp
dynamometer of arm 55 cm. during

© Copyright Reserved by Gateflix.in No part of this material should be copied or reproduced without permission
kg 1) volumetric efficiency
ma =10×60
h  altual volume innaled / cycle 
v   
600  sweet volume. 
=  4.92kg / KWh
122.039 1.00729 105
5. Brake thermal efficiency Pair   1.1738kg / m3
287  299
brake power Now in orifice air enter per
ηbth 
m  C.V. second,
122.039  Cd 2Psair
  32.54%
5  45 10 10 10 π
10  60  0.6   0.0382  (2 1422 1.173)
4
6. Air fuel ratio = 0.03932 kg/s
 A   10  Sweet volume
    20%
 F   0.5  2600
 20006 106 
2  60
Q.5 A sharp edged circular orifice of = 0.0433 m /rev
3
diameter 3.8 cm and co-efficient of Actual mass enter/sec
discharge as 0.6 is used to measure = 0.03932 kg/s
air consumption of a four stroke Actual volume enter/sec
petrol engine. The pressure drop 0.03932
through the orifice is 145 cm of 
1.1738
water and barometer reads 75.5cm = 0.3349 m3/s
of Hg. The compression ratio of the 0.3349
engine is 6 and the piston ηv   77.36%
0.0433
displacement volume is 2000cm3.
2) air fuel ratio
The temp of air is taken to be 260C
ma= 0.03932 kg/s
at 2600 rpm, the engine brake
0.14
power recorded is 29.5 KW. The fuel Sfc= 0.14 kg/min  kg / s
consumption is 0.14 kg/min and the 60
calorific value of fuel used is 43960 =2.33×10-3 kg/s
KJ/kg. Calculate following:  A  0.03932
   16.85
(1) Volumetric efficiency  F  2.33 163
(2) Air – fuel radio 3) Brake mean effective Pressure
(3) Brake mean effective pressure Pm  l  a  N 
(4) Brake thermal efficiency B.P.   
60 2
Solution:
Diameter of orifice d =3.8 cm 60  2  29.5 103
  6.8bar
Coefficient of discharge=0.6 2600  2000 106
Pressure drop =145 mm of water 4) brake thermal efficiently
BP
= 1,422.45 Pa ηbth 
Barometer reading =75.5 cm of hg
mf  CV
29.5 103
= 1,00,729.08 Pa 
r = 6, Vs =2000 cm3 2.33 103  43960 103
 28.86%
Temperature of air =299k
B.P.=29.5 kw at 2600 rpm Q.6 The following data are know for a
F.C.=0.14 kg/min four cylinder stroke petrol engine:
CV=43960 kj/kg cylinder dimension = 11 cm bore, 13

© Copyright Reserved by Gateflix.in No part of this material should be copied or reproduced without permission
cm stroke engine speed = 2250 rpm, C7 h16  11O2  7CO2  8H2O
brake power=50kw, friction power If 10% more air is supplied
=15 KW
79
fuel consumption rate =10.5 kg/h, C7 H16  12.1O2  [12.1x( )]N 2 
calorific value of fuel= 50,000 KJ/kg, 21
air inhalation rate = 300 kg/h, 7CO2  8H 2O  1.1O2  45.52N 2
ambient condition =150C, 1.03 bar. Mass of O2 for 100 kg of fuel = 12.1 x
Estimate: 32 = 387.3 kg of O2
1. Brake mean effective pressure Mass of air for 100 kg of fuel =
2. Brake thermal efficiency 1683.4 kg of air
3. Mechanical efficiency Mass of air required for 1 kg of fuel
=16.82 kg
Solution:
Dry exhaust gas Analysis on volume
1. Brake mean effective pressure
basis
PLANK
B.P  Moles %
60  2 Co2 7 11.36 %
B.P.  60  2 O2 1.1 1.78 %
P H2O 8 12.9 %
LANK N2 45.52 73.87 %
50  60  2

2250  4  π / 4  0.112  0.13 Q.8 Find the percentage increase in the
= 5.396 bar efficiency of a Diesel engine having
2. Brake thermal efficiency a compression ratio of 16 and cut-
brakepower off ratio (r) is 10% of the swept
ηBTh  volume if CV by 2% take Cv = o.717
mf  CV
KJ/Kg.K and γ=1,4
50
  34.29% Solution:
10.5 / 3600 X50000 The efficiency of Diesel engine
3. Mechanical Efficiency 1 [ρ γ  1]
η  1  γ 1
BP = 50 KW r [γ(ρ  1)]
FP = 15 KW Where
IP = 50 + 15 = 65KW v1
BP 50 r  16
Mechanical efficiency   v2
IP 65 v3 cp
= 76.92% ρ = cut of ratio  , 
v2 cv
Q.7 A liquid fuel C7 h16 is burned with
10% more air than the
stoichiometric air assuming complete
combustion calculate:
1. The mass of air supplied per kg
of fuel and
2. The volumetric analysis of the
dry products of combustion. Cv = 0.717 KJ/kg k, γ = 1.4
Assume air contains 21 % O2 by Cp = γ× Cv = 1.0038 kj/kg k
volume. R = Cp-Cv = 1.0038 – 0.717
Solution: = 0.2868 kj/kg k
V3 – V2 = 0.1(V1 – V2)
Equation for stoichiometric dividing the equation by V2, we get
composition:

© Copyright Reserved by Gateflix.in No part of this material should be copied or reproduced without permission
 V3   V1 
  1  0.10   1
 V2   V2 
(ρ-1)=0.10(r - 1)
(ρ- 1)=0.10(16 - 1)
ρ=2.5
1 [ρ γ  1]
η  1  γ 1
r [γ(ρ  1)]
1 [2.51.4  1]
 1  1.41
16 [1.4  (2.5  1)]
= 0.5905 = 59.05 %
When cv is decreases by 2%
CV’ = 0.70266
R = 0.2868 = cp’-cv’
Cp’ = 0.98946
Cp'
’     1.4081
Cv'
1 [ρ γ  1]
η'  1  γ'1
r [γ'(ρ  1)]
1 [2.51.408  1]
 1  1.4081
16 [1.408  (2.5  1)]
= 0.5975 = 59.75 %
Increase in efficiency of diesel cycle
η'  η 0.5975  0.5905
 100 
η 0.5905
= 1.185 %

© Copyright Reserved by Gateflix.in No part of this material should be copied or reproduced without permission
GATE QUESTIONS
Topics

1. THERMODYNAMIC SYSTEM AND PROCESSES

2. FIRST LAW, HEAT, WORK AND ENERGY

3. SECOND LAW, CRANOT CYCLE AND ENROPY

4. AVALILABILITY AND IRREVERSIBILITY

5. PURE SUBSTANCES

6. POWER SYSTEM (RANKINE, BRAYTON, ETC.)

7. IC ENGINE

8. REFRIGERATION AND AIR CONDITIONING

© Copyright Reserved by Gateflix.in No part of this material should be copied or reproduced without permission
1 THERMODYNAMIC SYSTEM AND PROCESSES

Q.1 The following four figures have been Q.3 Match items from groups I, II, III, IV
drawn to represent a fictitious and V.
thermodynamic cycle, on the p -v
and T -s planes.

a) F-G-J-K-M b) E-G-I-K-M
E-G-I-K-N F-H-I-K-N
c) F-H-J-L-N d) E-G-J-K-N
E-H-I-L-M F-H-J-K-M
[GATE–2006]

Common Data for Q.4 and Q.5


According to the first law of A thermodynamic cycle with an ideal gas as
thermodynamics, equal areas are working fluid is shown below.
enclosed by
a) Figures 1 and 2
b) Figures 1 and 3
c) Figures 1 and 4
d) Figures 2 and 3
[GATE–2005]

Q.2 A reversible thermodynamic cycle


containing only three processes and
producing work is to be constructed.
The constraints are Q.4 The above cycle is represented on T
i) There must be one isothermal -s plane by
process, a) b)
ii) There must be one isentropic
process,
iii) The maximum and minimum
cycle pressures and the clearance
volume are fixed, and
iv) Polytropic processes are not c) d)
allowed.
Then the number of possible cycles
are
a) 1 b) 2
c) 3 d) 4
[GATE–2007]
[GATE–2005]

© Copyright Reserved by Gateflix.in No part of this material should be copied or reproduced without permission
Q.5 If the specific heats of the working Q.10 Which of the following statements
fluid are constant and the value of are TRUE with respect to heat and
specific heat ratio is 1.4, the thermal work?
efficiency (%) of the cycle is i) They are boundary phenomena
a) 21 b) 40.9 ii) They are exact differentials
c) 42.6 d) 59.7 iii) They are path functions
[GATE–2007] a) both (i) and (ii)
b) both (i) and (iii)
Q.6 If a closed system is undergoing an c) both (ii) and (iii)
irreversible process, the entropy of d) only (iii)
the system [GATE -2016(1)]
a) must increase Q.11 A mass m of a perfect gas at pressure p1
b) always remains constant and volume V1 undergoes an isothermal
c) Must decrease process. The final pressure is p2 and
d) can increase, decrease of remain volume is V2. The work done on the
constant system is considered positive. If R is the
[ME-GATE –2009] gas constant and T is the temperature,
then the work done in the process is
Q.7 Heat and work are V2 p1
a) p1 V1 ln b) − p1 V1 ln
a) intensive properties V1 p2
b) extensive properties
c) point functions V2 p2
d) path functions c) RT ln d) − mRT ln
[ME-GATE –2012] V1 p1

Q.8 A certain amount of an ideal gas is [ME-GATE -2017(1)]


initially at a pressure p1 and Q.12. The volume and temperature of air
temperature T1. First, it undergoes a (assumed to be an ideal gas) in a
constant pressure process 1-2 such closed vessel is 2.87 m3 and 300K,
that T2 = 3T1/4. Then, it undergoes a respectively. The gauge pressure
constant volume process 2-3 such indicated by a manometer fitted to
that T3 =T1/2. The ratio of the final the wall of the vessel is 0.5bar. If the
volume to the initial volume of the gas constant of air is R = 287 J/kg. K
ideal gas is and the atmospheric pressure is 1
a) 0.25 b) 0.75 bar, the mass of air (in kg) in the
c ) 1.0 d) 1.5 vessel is
[ME-GATE -2014(3)] a) 1.67 b) 3.33
c) 5. d) 6.66
Q.9 Two identical metal blocks L and M [ME-GATE -2018(2)]
(specific heat = 0.4kJ/ kg. K), each Q.13 A n engine operates on the reversible
having a mass of 5kg, are initially at cycle as shown in the figure. The
313K. A reversible refrigerator work output from the engine (in
extracts heat from block L and kJ/cycle) is ______ (correct to two
rejects heat to block M until the decimal places).
temperature of block L reaches [ME-GATE -2018(2)]
293K. The final temperature (in K)
of block M is _____.
[ME-GATE -2014(4)]

© Copyright Reserved by Gateflix.in No part of this material should be copied or reproduced without permission
Q.14 Air is held inside a non-insulated
cylinder using a piston (mass M=25
kg and area A=100 cm2) and
stoppers (of negligible area), as
shown in the figure. The initial
pressure Pi and temperature Ti of air
inside the cylinder are 200 kPa and
400 0 C , respectively. The ambient
pressure P∞ and temperature T∞ are
100 kPa and 27 0 C , respectively. The
temperature of the air inside the
cylinder ( 0 C ) at which the piston
will begin to move is ______ (correct
to two decimal places).

[ME-GATE-2018 (2)]

© Copyright Reserved by Gateflix.in No part of this material should be copied or reproduced without permission
ANSWER KEY:
1 2 3 4 5 6 7 8 9 10 11 12 13
(a) (d) (d) (c) (a) (d) (d) (b) (b) (b) (c) 62.5
14
146

© Copyright Reserved by Gateflix.in No part of this material should be copied or reproduced without permission
EXPLANATIONS

Q.1 (a) Given P-v diagram is clockwise. So


From the first law of T-s diagram must be clockwise.
thermodynamics for a cyclic process
∆U = 0 Q.5 (a)
And ∮δQ =∮δ W
The symbol ∮ δQ , which is called the
cyclic integral of the heat transfer
represents the heat transfer during
the cycle and ∮ δ W the cyclic
integral of the work represents the
work during the cycle.
We easily see that figure 1 and 2
satisfy the first law of thermo- This cycle shows the Lenoir cycle.
dynamics, both the figure are in For Lenoir cycle efficiency is given by
same direction (clockwise) and
 1γ 
satisfies the relation.  rp − 1 
∮ δQ = ∮ δ W ηL = 1 − γ  
 rp − 1 
 
Q.2 (d)
Two cycles having constant volume p 2 400
Where, = rp = = 4
process and two cycles having p1 100
constant pressure process can be
formed. Cp
Therefore a total of four cycles can γ
And = = 1.4 (Given)
Cv
be formed.
So,
Q.3 (d)  1.41 
 (4) − 1 
ηL = 1 − 1.4 1 − 0.789
=
 4 −1 
Q.4 (c)  
In the given p - v diagram, three = 0.211
processes are occurred.
ηL = 21.1% ; 21%
i) Constant pressure (Process 1 – 2)
ii) Constant Volume (Process 2 – 3)
iii) Adiabatic (Process 3 – 1) Q.6 (d)
We know that, Constant pressure If a closed system is undergoing an
and constant volume lines are irreversible process, the entropy of
inclined curves in the T-s curve, and the system can increase, decrease or
adiabatic process is drawn by a remain constant.
vertical line on a T-s curve.
Q.7 (d)
Q. 8 (b)
For constant pressure process (1to
V V
2): 1 = 2
T1 T2

© Copyright Reserved by Gateflix.in No part of this material should be copied or reproduced without permission
 T2  3V1 ∴
V2 P1
=
=
⇒ V2 V= 1  V1 P2
 T1  4
For constant volume process (2 to P1
3) ∴ w = P1V1 ln
P2

Final volume V3 V2 3
= = = = 0.75 If the work done on the system is + ive,
Initial volume V1 V1 4 then

Q.9 (333 to 335) P1


w = −P1V1 ln
P2
Q.10 (b)
Q.11 Sol: (b) Q.12 (c)

=
Pabs Pgauge + Patm
∵ w = Pdv∫
For ideal gas PV = mRT = const (c ) =50 + 100 =150kPa

C By ideal gas eqn


PV = C ⇒ P =
V Pabs V = mRT
V2
C 150 × 2.87
W= ∫ V dv
V1
m=
.287 × 300

= c[ln v]VV12 m = 5kg

Q.13 (62.5)
V2
w = c ln
V1 For a reversible process,

work output = ∫ PdV


Hence, for the given reversible cycle,
Work Output = Area enclosed by the
triangle

1
= =  0.5  250 62.5 kJ/cycle
2
Q.14 (146.03)
V
⇒ w = P1V1 ln 2
V1

When work is done on the system


(compression), V2 < V1 and P2 > P1

For isothermal process

P=
1V1 P=
2 V2 c

© Copyright Reserved by Gateflix.in No part of this material should be copied or reproduced without permission
mg
= Pamb +
Pinside
A
25  9.81
= 100 + −4
 10−3
100 10
P2 = 124.525 kN/m 2
As mass and volume remains constant
P1 P
= 2
T1 T2
200 124.525
=
673 T2
T2 = 419.02 K = 146.03 o C

© Copyright Reserved by Gateflix.in No part of this material should be copied or reproduced without permission
2 FIRST LAW, HEAT, WORK AND ENERGY

Q.1 A small steam whistle (perfectly compression being 5000 kJ. During
insulated and doing no shaft work) the process, heat interaction of 2000
causes a drop of 0.8 kJ/kg in the kJ causes the surroundings to be
enthalpy of steam from entry to exit. heated. The changes in internal
If the kinetic energy of the steam at energy of the gas during the process is
entry is negligible, the velocity of the a) -7000 kJ b) -3000 kJ
steam at exit is c) +3000 kJ d) +7000 kJ
a) 4 m/s b) 40 m/s [GATE–2004]
c) 80 m/s d) 120 m/s
Common Data For Q.6 and Q.7
[GATE–2001]
A football was inflated to a gauge pressure
of 1 bar when the ambient temperature
Q.2 A 2 kW, 40 liters water heater is
was 15°C. When the game started next day,
switched on for 20 minutes. The
the air temperature at the stadium was 5°C.
heat capacity Cp for water is 4.2
Assume that the volume of the football
kJ/kgK. Assuming all the electrical
remains constant at 2500 cm3.
energy has gone into heating the
water, increase of the water Q.6 The amount of heat lost by the air in
temperature in degree centigrade is the football and the gauge pressure
a) 2.7 b) 4.0 of air in the football at the stadium
c) 14.3 d) 25.25 respectively equal
[GATE–2003] a) 30.6 J, 1.94 bar b) 21.8 J, 0.93 bar
c) 61.1 J, 1.94 bar d) 43.7 J, 0.93 bar
Common Data for Q.3 and 4 [GATE–2006]
Nitrogen gas (molecular weight 28) is
enclosed in a cylinder by a piston, at the Q.7 Gauge pressure of air to which the
initial condition of 2 bar, 298 K and 1 m3. In ball must have been originally
a particular process, the gas slowly inflated so that it would be equal 1
expands under isothermal condition, until bar gauge at the stadium is
the volume becomes 2m3. Heat exchange a) 2.23 bar b) 1.94 bar
occurs with the atmosphere at 298 K c) 1.07 bar d) 1.00 bars
during this process. [GATE–2006]
Q.8 Which of the following relationships
Q.3 The work interaction for the is valid only for reversible processes
Nitrogen gas is undergone by a closed system of
a) 200 kJ b) 138.6 kJ simple compressible substance?
c) 2 kJ d) -200 kJ (neglect changes in kinetic and
[GATE–2003] potential energy)
Q.4) The entropy changes for the a) Q = dU + W b) Tds = dU+pdv
Universe during the process in kJ/K is c) Tds = dU + W d) Q = dU + pdv
a) 0.4652 b) 0.0067 [GATE–2007]
c) 0 d) -0.6711
[GATE–2003] Q.9 A gas expands in a frictionless
piston-cylinder arrangement. The
Q.5 A gas contained in a cylinder is expansion process is very slow, and
compressed, the work required for is resisted by an ambient pressure

© Copyright Reserved by Gateflix.in No part of this material should be copied or reproduced without permission
of 100 kPa. During the expansion a) is greater than 3500C
process, the pressure of the system b) is less than 3500C
(gas) remains constant at 300 kPa. c) is equal to 3500C
The change in volume of the gas is d) may be greater than, less than, or
0.01m3. The maximum amount of equal to, 3500C depending on the
work that could be utilized from the volume of the tank
above process is [GATE–2008]
a) 0 kJ b) 1 kJ
c) 2 kJ d) 3 kJ. Q.12 In a steady state flow process taking
[GATE–2008] place in a device with a single inlet
and a single outlet, the work done
Q.10 A balloon containing an ideal gas is per unit mass flow rate is given by
initially kept in an evacuated and outlet
insulated room. The balloon W= − ∫inlet vdp where v is the
ruptures and the gas fills up the specific volume and p is the
entire room. Which one of the pressure. The expression for W
following statements is TRUE at the given above
end of above process? a) is valid only if the process is both
a) The internal energy of the gas reversible and adiabatic
decreases from its initial value, b) is valid only if the process is both
but the enthalpy remains constant reversible and isothermal
b) The internal energy of the gas c) is valid for any reversible
increases from its initial value, process
but the enthalpy remains constant d) is incorrect; it must be
c) Both internal energy & enthalpy outlet

of the gas remain constant W= ∫


inlet
pdv
d) Both internal energy and
[GATE–2008]
enthalpy of the gas increase
[GATE–2008]
Q.13 A frictionless piston-cylinder device
Q.11 A rigid, insulated tank is initially contains a gas initially at 0.8MPa
evacuated. The tank is connected and 0.015 m3 . It expands quasi-
with a supply line through which air statically at constant temperature to
(assumed to be ideal gas with a final volume of 0.030 m3 . The
constant specific heats) passes at work output (in kJ) during this
1MPa, 3500C. A valve connected process will be
with the supply line is opened and a) 8.32 b) 12.00
the tank is charged with air until the c) 554.67 d) 8320.00
final pressure inside the tank [GATE–2009]
reaches 1MPa. The final temperature Q.14 A compressor undergoes a
inside the tank. reversible, steady flow process. The
gas at inlet and outlet of the
compressor is designated as state 1
and state 2 respectively. Potential
and kinetic energy changes are to be
ignored. The following notations are
used:
v = Specific volume and p = pressure
of the gas. The specific work
required to be supplied to the

© Copyright Reserved by Gateflix.in No part of this material should be copied or reproduced without permission
compressor for this gas
compression process is
2 2
a) ∫pdv b) ∫vdp
1 1

c) v1 ( p 2 -p1 ) d) −p 2 ( v1 − v 2 )
[GATE–2009]
Q.17 The density of air in kg/m3 at the
nozzle exit is
Common Data for Q.15 and Q.16
a) 0.560 b) 0.600
The inlet and the outlet conditions of steam
c) 0.727 d) 0.800
for an adiabatic steam turbine are as
[GATE–2011]
indicated in the figure. The notations are as
usually followed. Q.18 The mass flow rate of air through
the nozzle in kg/s is
a) 1.30 b) 1.77
c) 1.85 d) 2.06
[GATE–2011]
Q.19 The contents of a well-insulated
tank are heated by a resistor of 23 Ω
in which 10 A current is flowing.
Consider the tank along with its
Q.15 If mass rate of steam through the contents as a thermodynamic
turbine is 20 kg/s, the power output system. The work done by the
of the turbine (in MW) is system and the heat transfer to the
a) 12.157 b) 12.941 system are positive. The rates of
c) 168.001 d) 168.785 heat (Q), work (W) and change in
[GATE–2009] internal energy (∆U) during the
Q.16 Assume the above turbine to be part process in kW are
of a simple Rankine cycle. The a) Q = 0, W = -2.3, ∆U = +2.3
density of water at the inlet to the b) Q = +2.3, W = 0, ∆U = +2.3
pump is 1000 kg/m3 . Ignoring c) Q = -2.3, W = 0, ∆U = -2.3
kinetic and potential energy effects, d) Q = 0, W = +2.3, ∆U = -2.3
the specific work (in kJ/kg) supplied [GATE–2011]
to the pump is Common Data for Q.20 and Q.21
a) 0.293 b) 0.351 Air enters an adiabatic nozzle at 300 kPa,
c) 2.930 d) 3.510 500 K with the velocity of 10 m/s. It leaves
[GATE–2009] the nozzle at 100 kPa with a velocity of 180
Common Data For Q.17 and Q.18 m/s. The inlet area is 80 cm2. The specific
The temperature and pressure of air in a heat of air Cp is 1008 J/kgK.
large reservoir are 400 K and 3 bar Q.20 The exit temperature of the air is
respectively. A converging-diverging nozzle a) 516 K b) 532 K
of exit area 0.005 m2 is fitted to the wall of c) 484 K d) 468 K
the reservoir as shown in the figure. The [GATE–2012]
static pressure of air at the exit section for
isentropic flow through the nozzle is 50 Q.21 The exit area of the nozzle in cm2 is
kPa. The characteristic gas constant and the a) 90.1 b) 56.3
ratio of specific heats of air are c) 4.4 d) 12.9
0.287kJ/kgK and 1.4 respectively [GATE–2012]

© Copyright Reserved by Gateflix.in No part of this material should be copied or reproduced without permission
Q.22 For an ideal gas with constant
values of specific heats, for Q.26 A mixture of ideal gases has the
calculation of the specific enthalpy, following composition by mass
a) it is sufficient to know only the
temperature
b) both temperature and pressure If the Universal gas constant is 8314
are required to be known J/mol-K, the characteristic gas
c) both temperature and volume constant of the mixture (in J/kg.K) is
are required to be known _______.
d) both temperature and mass are [GATE-2015 Set-3]
required to be known
[GATE-2015 Set-1] Q.27 An ideal gas undergoes a reversible
Q.23 A well insulated rigid container of process in which the pressure varies
volume 1m3 contains 1.0 kg of an linearly with volume. The conditions
ideal gas [Cp = 1000J/kgK) and Cv = at the start (subscript 1) and at the
800J/kgK] at a pressure of 105 Pa. A end (subscript 2) of the process
stirrer is rotated at constant rpm in with usual notation are: 𝑝𝑝1 = 100
the container for 1000 rotations and kPa, 𝑉𝑉1 = 0.2 m3 and 𝑝𝑝2 = 200 kPa, 𝑉𝑉2
the applied torque is 100 N-m. The = 0.1 m3 and the gas constant, R =
final temperature of the gas (in K) is 0.275 kJ/kg-K. The magnitude of the
______. work required for the process (in kJ)
[GATE-2015 Set-1] is ________

Q.24 Work is done on adiabatic system Q.28 The internal energy of an ideal gas is
due to which its velocity changes a function of
from 10 m/s to 20 m/s, elevation a) temperature and pressure
increases by 20m and temperature b) volume and pressure
increases by 1 K. The mass of the c) entropy and pressure
system is 10 kg, Cv=100 J/kgK and d) temperature only
gravitational acceleration is 10 [GATE-2016 Set-2]
m/s2. If there is no change in any Q.29 A piston-cylinder device initially
other component of the energy of contains 0.4 m3 of air (to be treated
the system, the magnitude of total as an ideal gas) at 100 kPa and 80℃.
work done (in kJ) on the system is The air is now isothermally
_____. compressed to 0.1 m3. The work
[GATE-2015 Set-2] done during this process is _______ kJ.
(Take the sign convention such that
Q.25 Steam enters a turbine at 30 bar, work done on the system is
300℃ (u = 2750 kJ/kg, h = 2993 negative)
kJ/kg) and exits the turbine as [GATE-2016 Set-2]
saturated liquid at 15 kPa (u = 225
kJ/kg, h = 226 kJ/kg). Heat loss to Q.30 Steam at an initial enthalpy of 100
the surrounding is 50 kJ/kg of steam kJ/kg and inlet velocity of 100 m/s,
flowing through the turbine. enters an insulated horizontal
Neglecting changes in kinetic energy nozzle. It leaves the nozzle at 200
and potential energy, the work m/s. The exit enthalpy (in kJ/kg) is
output of the turbine (in kJ/kg of __________
steam) is________ [GATE-2016 Set-3]
[GATE-2015 Set-3]

© Copyright Reserved by Gateflix.in No part of this material should be copied or reproduced without permission
Q.31 The molar specific heat at constant
volume of an ideal gas is equal to 2.5
times the universal gas constant
(8.314 J/mol.K). When the
temperature increases by 100K, the
change in molar specific enthalpy is
_______________ J/mol
[GATE-2017 Set-1]

Q.32 A frictionless circular piston of area


10 −2 m2 and mass 100 kg sinks into
a cylindrical container of the same
area filled with water of density
1000  kg / m3 as shown in the figure.
The container has a hole of area
10 −3 m2 at the bottom that is open to
the atmosphere. Assuming there is
no leakage from the edges of the
piston and considering water to be
incompressible, the magnitude of
the piston velocity (in m/s) at the
instant shown is _____ (correct to
three decimal places).

[GATE-2018 Set-2]

© Copyright Reserved by Gateflix.in No part of this material should be copied or reproduced without permission
ANSWER KEY:
1 2 3 4 5 6 7 8 9 10 11 12 13 14
(b) (c) (b) (a) (c) (d) (c) (d) (c) (c) (a) (c) (a) (b)
15 16 17 18 19 20 21 22 23 24 25 26 27 28
(a) (c) (c) (d) (a) (c) (d) (a) 4.5 2717 (d)
29 30 31 32
- - 29.09 1.456

EXPLANATIONS

Q.1 (b) Q.5 (c)


C 2
C 2
W = -5000 kJ (Negative sign shows
h1 + =h 2 +
1 2
that work is done on the system)
2 2
C1 = 0 Q = - 2000 kJ (Negative sign shows
that heat is rejected by the system)
C2 From the first law of
h1 − h 2 =2
2 thermodynamics,
C2 ∆ Q = ∆W + ∆ U
0.8 ×103 = 2
2 So
C2 = 40m / s ∆U = ∆Q − ∆ W = −2000 −
(−5000) = 3000 kJ
Q.2 (c)
Heat Supplied Q.6 (d)
H = Power × Time Given
P1 = 1 bar (Gauge)
=2 × 1000 × 20 × 60
(P1 ) absolute = 1+1.013= 2.013 bar.
=2400 × 1000 J
T1 =15℃=288K
But heat required in increasing the
T2 =5℃=278K
water temperature
H= mCp ∆T Vol. = Constant = 2500 cm3
at constant vol. (15℃→5℃)
2400×1000=1000×0.04
Q= mCv dT
×4.2×1000×∆T
Q= m×718×(278-288)
∆T=14.3℃
We need to calculate the mass first,
Using PV=mRT
Q.3 (b)
For isothermal process, 2.013 ×105 × 2500 ×10−6 =m × 287 × 288
V m = 6.088 × 10−3 kg
Work = P1V1 ln 2 Q = 6.088 × 10−3 ×718 × (278-288)
V1
2 Q = 43.7 J (-ve sign indicates that
= ( 2 ×100 ) × (1) × ln heat lost by the air in the ball)
1
T1 = 288
= 138.63 kJ
P2 = ?
T2 = 278
Q.4 (a)

© Copyright Reserved by Gateflix.in No part of this material should be copied or reproduced without permission
Vol. = C (Heat) between system (room) and
T2 P2 surrounding (atmosphere).
=
T1 P1 It means internal Energy dU = 0 and
U = constant.
278
=
⇒ P2 2.013 × Now flow work pv must also remain
288 constant thus we may conclude that
=1.93 bar. (absolute) during free expansion process pv i.e.
⇒ Pabsol
= Patm + Pgauge product of pressure and specific
⇒ Pgauge = 0.93bar volume change in such a way that
their product remains constant.
Q.7 (c) So, it is a constant internal energy
We know that and constant enthalpy process.
Since volume is constant
P T Q.11 (a)
∴ 1 = 1 Given: p1 = 1 MPa, T1 = 350oC=
P2 T2
623 K
∴ P2 = 1 bar (gauge)
For air 𝛾𝛾 = 1.4
= 2.013 bars (absolute)
We know that final temperature
Substituting the values, we get
(T2 ) inside the tank is given by,
P1 = 2.085 bars (absolute)
T2 = γ T1 = 1.4 × 623 = 872.2 K =
= 1.07 bar (gauge)
599.2o C
T2 is greater than 350o C
Q.8 (d)
δ q = dU + pdv This equation
Q.12 (c)
holds good for a closed system when
only pdv work is present. This is
Q.13 (a)
true only for a reversible (quasi-
static) process.

Q.9 (c)
Given: pa = 100 kPa, ps = 300 kPa,
∆v = 0.01 m3
Net pressure across the piston
p = ps − pa = 300 − 100 =
200 kPa
P1 = 0.8 MPa
V1 = 0.015 m3
V2 = 0.030 m3
Isothermal process
Work output
Maximum work that can be v 
utilized from the system is given by: W = P1V1 ln  2 
W = p∆v = 200 × 0.01 = 2 kJ  v1 

Q.10 (c)
= ( 0.8 ×10 ) ( 0.015) ln  0.030
3 

0.015 
We know that enthalpy, =0.8 × 103 × 0.015 × ln(2)
= 8.32 kJ
Given that room is insulated. So
there is no interaction of Energy Q.14 (b)

© Copyright Reserved by Gateflix.in No part of this material should be copied or reproduced without permission
Reversible steady flow process. =12156.78 kW=12.157 MW

Q.16 (c)
Neglecting Kinetic and potential
energy effects
SFEE :
h1 + q = h2 + wT (For Turbine)
⇒ 3200=2600+wT
State 1 : Inlet ⇒ wT = 600kJ/kg
State 2: Outlet Specific work supplied to the pump
Potential and Kinetic energy change is
are to be ignored. (Since the cycle works between the
v= Specific volume of the gas pressure limits of 3 MPa and 70
P= Pressure of the gas kPa)
Specific work required to be
supplied to the compressor for this Wp = � υdP = υ( P1 − P2 )
gas compression process is P1 − P2
2 =
w = ∫1 vdp ρ
(Compressor is an open flow system) ( 3000 − 70 ) kN / m 2
= = 2.930kJ/kg
1000kg / m3

Q.17 (c)
Density of air in (kg/m3) at nozzle
exit is
P1 P2
Q.15 (a) =
ρ1γ ρ 2γ
Where ρ=density
1
 P γ
ρ 2 =  2  .ρ1
 P1 
3 ×105
1/1.4
Apply steady flow energy equation  50 
=ρ2   ×
[SFEE]  300  0.287 ×1000 × 400
V12 V22 ρ 2 = 0.727kg / m3
h1 + + gz1 + q =h 2 + + gz 2 + w
2 2
q = 0; because it is an adiabatic Q.18 (d)
steam turbine Given:
(100) 2 ρ2 = 0.787 kg/m3, A2 = 0.005m2,
⇒ 3200×103 + + 9.81×10 + 0 V2 =?
2
For isentropic expansion,
(100) 2
= 2600×103 + + 9.81× 6 + W V2 = 2C p (T1 − T2 )
2
⇒ W=607839.24 J/kg = 2 × 1.005 × 103 × ( 400 − 239.73)
= 607.84kJ/kg = 567.58 m/s
Mass flow rate of steam through Mass flow rate at exit
turbine is 20kg/s. m = ρ2 A2 V2 =0.727×0.005×567.78
The power output of the turbine is = 2.06 kg/s
=20×607.84

© Copyright Reserved by Gateflix.in No part of this material should be copied or reproduced without permission
Q.19 (a) A2 = 12.9 cm2

Q.22 (a)

Q.23 (1283.4 to 1287.4)

Q.24 (4.5)
Using SFEE
 V 2 V 2  
W=−I2 R [∵work is done on the W m  1 − 2  + ( z1 − z 2 ) g + ( h1 − h 2 ) 
=
system]  2 2  
= −(10)2 × 23 = − 2300W = −2.3kW 102 202 
Q = 0 [∵ system is insulated] =W 10  − + ( −20 ) ×10 + (100 × −1) 
From 1st law: - ∆Q = ∆U + ∆W  2 2 
∆U + ∆W = 0 W = − 4.5 kJ
⇒ ∆U = −∆W = +2.3 kW ∴ Work done on the system is 4.5 kJ.

Q.20 (c) Q.25 (2717)


From energy balance for steady flow By Steady flow energy equation:
system W = (h1 − h2 ) q
Ein = Eout W= (2993 226)− 50 = 2717 kJ/kg
 V12   V22  (Neglecting kinetic and potential
m  h1 +  = m  h2 +  ...(i) energy changes)
 2   2 
As Equation (1) become Q.26 (274 to 276)
V12 V22
Cp T1 + = Cp T2 +
2 2 Q.27 (14.75 to 15.25)
 V − V2  Given: Pressure varies linearly with
2 2
102 − 1802
=T2  1 = + + 500 volume
 2 × Cp  1
T
  2 × 1008 ∴ p = aV+b --------- (A)
= −16.02 + 500 at V1 = 0.2m3 , p1 = 100kPa
= 483.98 ≃ 484 K ∴100 = 0.2a + b ----- (1)
3
at V2 = 0.1m , p2 = 200kPa
Q.21 (d) 200 = 0.1 a + b --------- (2)
From Mass conservation From equation (1) & (2), we get
min = mout a = −1000 and b = 300
V1A1 V2 A 2
= ...(i) ∴ equation (A) ⇒ p = 300 – 1000V
v1 v2 V2

where v = specific volume of air = Work = ∫ pdV


V1
RT V2
P
Therefore Eq. (1) becomes
= ∫ [300 − 1000V ] dV
V1
P1V1A1 P2 V2 A 2 V2
=  V2 
RT1 RT2 = 300V − 1000  
P × V × A1 × T2  2  V1
A2 = 1 1 = 300[0.1 – 0.2] – 500 [0.12 – 0.22]
P2 × V2 × T1
Work = −15kJ (-ve sign implies
300 ×10 × 80 × 484 work is done on the system)
=
100 ×180 × 500

© Copyright Reserved by Gateflix.in No part of this material should be copied or reproduced without permission
∵ in the question only magnitude is Cp − Cv =
R
asked.
∴ Work = 15kJ Cp = C v + R = 2.5R + R = 3.5R

Q.28 (d) For ideal gas, Molar specific enthalpy


change ( (∆ H)
Q.29 (-55.6 to -55.4)
∆H =
nCp dT

∆H
∆ H= = Cp dT= 3.5RdT
n
=3.5 × 8.314 × 100

Given:- = 2909.9J / mole


P1 = 100 kPa Q.32 (1.456)
V1 = 0.4 m3
Given. Piston mass = 100 kg
V2 = 0.1 m3
Work done in an isothermal process Pressure at point (1)
is given by
V  100 10
= P1V1 ln  2  =P1 = −2
105 N/m 2
 V1  10
 0.1 
= 100 × 0.4 ln  
 0.4 
W = 55.45kJ

Q.30 (84 to 86)

Applying Bernoulli’s equation for


By SFEE:
points (1) and (2):
V12 V22
h1 + =h 2 + [Neglecting
2 2 P1 V12 P2 V22
potential head, q = 0 & Wc.v = 0] + + Z1 = + + Z 2 ...... (1)
γ w 2g γ w 2g
1 100 − 200 
2 2

=
h 2 100 +
2 103 where P2 = 0 & Z2 = 0 (datum line)
h2 = 85kJ/kg
From continuity equation:
Q.31 (29.09)
A1 V1 = A2 V2 (discharge from hole=
Given that C v = 2.5R volume swept by the piston per unit
time)

© Copyright Reserved by Gateflix.in No part of this material should be copied or reproduced without permission
10−2 V1 = 10−3 V2
⇒ V2 =
10 V1 .....(2)

From eq. (1) and (2)

(10V1 )
2
105 V12
+ + 0.5 =
10 10 2  10
3
2  10
99V12
or 10.5 =
20
⇒ V1 =
1.456 m/s

© Copyright Reserved by Gateflix.in No part of this material should be copied or reproduced without permission
3 SECOND LAW, CARNOT CYCLE AND ENROPY

Q.1 A cyclic heat engine does 50 kJ of a) 12.50 b) 14.29


work per cycle. If the efficiency of c) 33.33 d) 57.14
the heat engine is 75%, the heat [GATE–2007]
rejected per cycle is
2 1 Q.5 A cyclic device operates between
a) 16 kJ b) 33 kJ three thermal reservoirs, as shown
3 3 in the figure. Heat is transferred
1 2 to/from the cycle device. It is
c) 37 kJ d) 66 kJ
2 3 assumed that heat transfer between
[GATE–2001] each thermal reservoir and the
cyclic device takes place across
Q.2 A Carnot cycle is having an
negligible temperature difference.
efficiency of 0.75. If the temperature
Interactions between the cyclic
of the high temperature reservoir is
device and the respective thermal
7270C, what is the temperature of
reservoirs that are shown in the
low temperature reservoir?
figure are all in the form of heat
a) 230C b) -230C
transfer.
c) 0 C
0 d) 2500C
[GATE–2002]
Q.3 A solar collector receiving solar
radiation at the rate of 0.6 kW/m2
transforms it to the internal energy
of a fluid at an overall efficiency of
50%. The fluid heated to 350 K is The cyclic device can be
used to run a heat engine which a) a reversible heat engine
rejects heat at 315 K. If the heat b) a reversible heat pump or a
engine is to deliver 2.5 kW power, reversible refrigerator
the minimum area of the solar c) an irreversible heat engine
collector required would be d) an irreversible heat pump or an
irreversible refrigerator
a) 83.33 m 2 b) 16.66 m 2
[GATE–2008]
c) 39.68 m 2
d) 79.36 m 2
[GATE–2004] Q.6 An irreversible heat engine extracts
heat from a high temperature
Q.4 A heat transformer is device that source at a rate of 100 kW and
transfers a part of the heat, supplied rejects heat to a sink at a rate of 50
to it at an intermediate temperature, kW. The entire work output of the
to a high temperature reservoir heat engine is used to drive a
while rejecting the remaining part reversible heat pump operating
to a low temperature heat sink. In between a set of independent
such a heat transformer, 100 kJ of isothermal heat reservoirs at 17°C
heat is supplied at 350 K. The and 75°C. The rate (in kW) at which
maximum amount of heat in kJ that the heat pump delivers heat to its
can be transferred to 400 K, when high temperature sink is
the rest is rejected to a heat sink at a) 50 b) 250
300 K is

© Copyright Reserved by Gateflix.in No part of this material should be copied or reproduced without permission
c) 300 d) 360 Internal energy,
[GATE–2009] u=CV T
Q.7 One kilogram of water at room
temperature is brought into contact Q.9 If the air has to flow from station P
with a high temperature thermal to station Q, the maximum possible
reservoir. The entropy change of the value of pressure in kPa at station Q
universe is is close to
a) equal to entropy change of the a) 50 b) 87
reservoir c) 128 d) 150
b) equal to entropy change of water [GATE–2011]
c) equal to zero Q.10 If the pressure at station Q is 50 kPa,
d) always positive the change in entropy ( SQ − SP ) in
[GATE–2010]
kJ/kgK is
Q.8 Consider the following two a) -0.155 b) 0
processes ; c) 0.160 d) 0.355
a) A heat source at 1200 K loses [GATE–2011]
2500 kJ of heat to a sink at 800 K Q.11 An ideal gas of mass m and
b) A heat source at 800 K loses temperature T1 undergoes a
2000 kJ of heat to a sink at 500 K reversible isothermal process from
Which of the following statements is an initial pressure P1 to final
true ? pressure P2. The heat loss during the
a) Process I is more irreversible process is Q. The entropy change ∆s
than Process II of the gas is
b) Process II is more irreversible
P  P 
than Process I a) mRln  2  b) mRln  1 
c) Irreversibility associated in both  P1   P2 
the processes are equal P  Q
d) Both the processes are reversible c) mRln  2  − d) zero
[GATE–2010]  P1  T1
[GATE–2013]
Common Data for Q.9 and Q.10 Q.12 Which one of the following pairs of
In an experimental set up, air flows equations describes an irreversible
between two stations P and Q adiabatically. heat engine?
The direction of flow depends on the δQ
pressure and temperature conditions a) ∮δQ>0and∮ <0
maintained at P and Q. The conditions at T
station P are 150 kPa and 350 K. The δQ
b) ∮δQ < 0and∮ < 0
temperature at station Q is 300 K. T
The following are the properties and δQ
c) ∮δQ > 0and∮ > 0
relations pertaining to air: T
Specific heat at constant pressure, CP δQ
= 1.005 kJ/kgK; d) ∮δQ < 0and∮ > 0
T
Specific heat at constant volume, CV [GATE-2014 (3)]
= 0.718 kJ/kgK;
Characteristic gas constant, Q.13 A source at a temperature of 500 K
R=0.287 kJ/kgK provides 1000 kJ of heat. The of
Enthalpy, temperature environment is 27℃.
h = CP T The maximum useful work (in kJ)

© Copyright Reserved by Gateflix.in No part of this material should be copied or reproduced without permission
that can be obtained from the heat Q.18 One kg of air (R=287 J/kgK)
source is _______. undergoes an irreversible process
[GATE-2014 (3)] between equilibrium state1 (20℃,
Q.14 A reversible heat engine receives 2 0.9 m3) and equilibrium state2
kJ of heat from a reservoir at 1000 K (20℃, 0.6 m3). The change in
and a certain amount of heat from a entropy s2 – s1 (in J/kg.K) is ________.
reservoir at 800 K. It rejects 1 kJ of [GATE-2015 (2)]
heat to a reservoir at 400 K. The net
work output (in kJ) of the cycle is Q.19 The heat removal rate from a
a) 0.8 b) 1.0 refrigerated space and the power
c) 1.4 d) 2.0 input to the compressor are 7.2 kW
[GATE-2014 (1)] and 1.8 kW, respectively. The
coefficient of performance (COP) of
Q.15 An amount of 100 kW of heat is the refrigerator is ______
transferred through a wall in steady [GATE-2016 (2)]
state. One side of the wall is
maintained at 127℃ and the other Q.20 A reversible cycle receives 40 kJ of
side at 27℃. The entropy generated heat from one heat source at a
(in W/K) due to the heat transfer temperature of 127℃ and 37 kJ
through the wall is ____. from another heat source at 97℃.
[GATE-2014 (3)] The heat rejected (in kJ) to the heat
sink at 47℃ is __________
Q.16 A closed system contains 10 kg of [GATE-2016 (2)]
saturated liquid ammonia at 10℃. Q.21 A heat pump absorbs 10 kW of heat
Heat addition required to convert from outside environment at 250 K
the entire liquid into saturated while absorbing 15 kW of work. It
vapour at a constant pressure is delivers the heat to a room that
16.2 MJ. If the entropy of the must be kept warm at 300K. The
saturated liquid is 0.88 kJ/kg.K, the Coefficient of Performance (COP) of
entropy (in kJ/kg. K) of saturated the heat pump is ___________.
vapor is ____________ [GATE-2017 (1)]
[GATE-2014 Set-4]
Q.22 One kg of an ideal gas (gas constant,
Q.17 A Carnot engine (CE-1) works R = 400 J/kg.K; specific heat at
between two temperature reservoirs constant volume,
A and B, where TA = 900 K and TB = c v = 1000J/kg.K at 1 bar, and 300 K
500 K. A second Carnot engine (CE-
is contained in a sealed rigid
2) works between temperature
cylinder. During an adiabatic
reservoirs B and C, where TC = 300
process, 100kJ of work is done on
K. In each cycle of CE-1 and CE-2, all
the system by a stirrer. The increase
the heat rejected by CE-1 to
in entropy of the system is _________
reservoir B is used by CE-2. For one
J/K.
cycle of operation, if the net Q
[GATE-2017 (1)]
absorbed by CE-1 from reservoir A
is 150 MJ, the net heat rejected to
Q.23 A n ideal gas undergoes a process
reservoir C by CE-2(in MJ) is
from state 1
____________.
[GATE-2015 (1)] = (T1 300
= K,p1 100 kPa) to state 2
=( T2 600
= K,p2 500 kPa) . The

© Copyright Reserved by Gateflix.in No part of this material should be copied or reproduced without permission
specific heats of the ideal gas are : cp [GATE-2018 Set-2]
= 1 kJ/kg-K and cv = 0.7 kJ/kg-K.
The change in specific entropy of the
ideal gas from state 1 to state 2 (in
kJ/kg-K) is __________(correct to two
decimal places).

[GATE-2018 Set-1]

Q.24 Steam flows through a nozzle at a


mass flow rate of m  = 0.1 kg / s . with
a heat loss of 5 kW. The enthalpies
at inlet and exit are 2500kJ/kg and
2350 kJ/kg, respectively. Assuming
negligible velocity at inlet ( C1 ≈ 0 ),
the velocity ( C2 ) of steam (in m/s)
at the nozzle exit is __________
(correct to two decimal places).

[GATE-2018 Set-1]

Q.25 For an ideal gas with constant


properties undergoing a quasi-static
process, which one of the following
represents the change of entropy (
∆s ) from state 1 to 2?

T   P2 
a ) ∆s = CP ln  2  − R ln  
 T1  P1 
T   V2 
b) ∆s = CV ln  2  − CP ln  
 T1  V1 
T  P 
c) ∆s = CP ln  2  − CV ln  2 
 T1   P1 
T  V 
d) ∆s = CV ln  2  + R ln  1 
 T1   V2 

© Copyright Reserved by Gateflix.in No part of this material should be copied or reproduced without permission
ANSWER KEY:
1 2 3 4 5 6 7 8 9 10 11 12 13 14
(a) (b) (a) (d) (a) (c) (d) (b) (b) (c) (b) (a) 400 (c)
15 16 17 18 19 20 21 22 23 24 25
83.33 6.6 50 -16.36 4 64 1.67 287 0.21 447 (a)

© Copyright Reserved by Gateflix.in No part of this material should be copied or reproduced without permission
EXPLANATIONS

Q.1 (a) Q.4 (d)


W Given: T1 = 400K, T2 = 300 K,
η=
QS T = 350 K, Q = 100kJ
50 Q1 → Heat transferred to the source
0.75 = by the transformer
QS
Q2 → Heat transferred to the sink
QS = 66.67 by the transformer
∴ Q R = 66.67 − 50
=16.67kJ
2
= 16 kJ
3

Q.2 (b)
TL
η= 1 −
TH
TL Applying energy balance on the
0.75 = 1 − system
1000
TL = Q1 + Q 2
Q
= 0.25 Q 2 =Q − Q1 =100 − Q1 ...(i)
1000
TL = 250K Apply Claudius inequality on the
TL = −23℃
system.
Q Q1 Q 2 100 Q1 Q
= + = = + 2
T T1 T2 350 400 300
Q.3 (a)
Internal energy of fluid after Substitute the value of Q 2 from
absorbing solar radiation equation (i)
=0.5×600=300W/m2 100 Q1  100 − Q1  Q1 100 Q1
= + = + −
 315  350 400  300  400 300 300
ηengine= 1 − 
 350  100 100  1 1 
= + Q1  −
=
W 350 300  400 300 
Q Solving the above equation, we get
2500 Q1 =57.14 kJ
0.1 =
Q1 Therefore the maximum amount of
∴ Q1 = 25000W heat that can be transferred at 400 K
is 57.14 kJ
Let A be the minimum area of
collector
Q.5 (a)
∴ Q1 =A × 300
A heat engine cycle is a
25000 thermodynamic cycle in which there
∴A =
300 is a net heat transfer from higher
A= 83.33m2 temperature to a lower temperature
device. So it is a Heat Engine.

© Copyright Reserved by Gateflix.in No part of this material should be copied or reproduced without permission
Applying Clausius inequality on the ( ∆S)system + ( ∆S)surrounding > 0
system for checking the reversibility
Q.8 (b)
of the cyclic device
We know from the clauses
dQ
∮T =0 Inequality,
dQ
Q1 Q 2 Q3 If ∮ = 0, the cycle is reversible
+ − = 0 T
T1 T2 T3 dQ
100 × 103 50 × 103 60 × 103 ∮ T < 0 the cycle is irreversible
+ − = 0
1000 500 300 and possible
100+100–200 = 0 For case (I)
Here, the cyclic integral of is zero. dQ 2500 2500
This implies, it is a reversible heat ∮= −
T 1200 800
engine. 25 25
=− = −1.041kJ / kg
Q.6 (c) 12 8
For case (II)
dQ 2000 2000
∮= T

800 500
20 20
=− = −1.5kJ / kg
8 5
dQ
Since, ∮ for the process II is
T
more in magnitude than process I.
Therefore, process (II) is more
irreversible than process (I)

Q.9 (b)
Maximum possible value of pressure
We know that coefficient of has been asked in the question, so
performance of a Heat pump for the we take it as a case of reversible
given system is, flow.
Q Tds = dh − vdp
(COP) H.P. = 3
W T  P 
Since it is a reversible heat pump ds = Cp ln  2  − R ln  2 
TH  T1   P1 
(COP) H.P. = The maximum possible value of
TH − TL
pressure at station Q can be found
348 Q as follows:
= 3
348 − 290 50 T  P 
348 × 50 Cp ln  Q  − R ln  Q  = 0
∴=Q3 = 300K  P
T  P
P
58
 300   PQ 
1.005 ln   = 0.287 ln  
Q.7 (d)  350   150 
We know that, PQ = 87.43kPa
Entropy of universe is always
increases.
Q.10 (c)
∆Suniverse > 0

© Copyright Reserved by Gateflix.in No part of this material should be copied or reproduced without permission
By using Tds=dh–vdP, after Q1 Q 2 Q3
Or + − = 0
simplifying we get: T1 T2 T3
T  P  2 Q 1
(SQ=− Sp ) CP ln  Q  − R ln  Q  Or + 2 − =
0
 TP   PP  1000 800 400
 300   50  Or Q 2 = 0.4kJ
= 1.005ln   − 0.287 ln  
 350   150  From 1st law of thermodynamics for
= 0.160 a cycle:
Q1 + Q 2 = W + Q3
Q.11 (b) Or 2+0.4 = W+1
Or W = 1.4 kJ
Q.12 (a)
Clausius inequality for irreversible Q. 15 (83.33)
dQ
heat engine, ∮ < 0. Heat content
T
of irreversible heat engine dQ > 0

Q.13 (400)
Maximum work can be derived by
using a reversible engine between 2
dQ
the two temperature limits. S2 − S=
1 ∫
T
+ Sgen
Wmax = Qs ηmax 1
100 00
 T  ∴0 = − + Sgen
= Qs  1 − L  400 300
 TH  1 1 1
Qs Or Sgen = − = kW / K
= Qs − TL
Wmax 3 4 12
TH 1000 W
Given, Qs = 1000kJ ∴ Sgen = = =83.33W / K
12 K
TL = 273 + 27 = 300K
TH = 500K Q. 16 (6.6)
h fg 16.2 ×103
∴ Wmax = 1000 − 300 ×
1000 =
Sfg = = 57.24 kJ / K
T 283
500
= 400 kJ 57.24
s fg = =5.724 kJ / kgK
10
Q.14 (c) entropy
Since specific entropy = )
mass
s g = s fg + s f
= 5.724 + 0.88
= 6.604 kJ/ kg K

Q.17 (50)

From Clausius inequality


dQ
∮ T = 0 , for a reversible cycle

© Copyright Reserved by Gateflix.in No part of this material should be copied or reproduced without permission
TL1 900 − 500 Q.21.
η1 =
1− = = 0.444
TH1 900
Q
Also, η1 = 1 − 2
Q1
∴ Q 2 = 83.33MJ
T 300
Also, η2 = 1 − L2 = 1− = 0.4
T2 500
Q
η2 = 1 − 3
Q2
∴ Q 2 = 50 MJ
Q = 15 + 10 = 25 kw

Q.18 (-116.36) desired effect


Cop =
V2 0.6 work input
s 2=
− s1 R In= 287 ln
V1 0.9
=−116.368 J/kg/K 25 5
= =
15 3
Q.19 (4)
Cop = 1.67
Q
COPref = L
Wi/p
Q.22. By first law
7.2
COP= = 4
ref
1.8 δQ= du + δw

Q.20 (64) =
0 C v [Tf − Ti ] − 100

1× [ Tf − 300] =
100

Tf = 400k

Entropy change of an ideal gas is given by

Tf V
S2 =
− S1 C v ln + R ln f
Ti Vi

Q Vf = Vi
By Clausius inequality
δQ ∴ S2 − S1 =
Tf
∮T = 0 C v ln
Ti
[For reversible cycle]
 400 

40 37
+ −
Q
= 0 = 1000 ln  
400 370 320  300 
Q = 64kJ
S2 − S1 =
287.6J / k

Q.23 (0.21)

© Copyright Reserved by Gateflix.in No part of this material should be copied or reproduced without permission
Ideal gas
State − 1 :=
T1 300 K,=
P1 100 kPa
State − 2 : =
T2 600 K, =
P2 500 kPa
c=
p 1kJ/kg-K, c p − c v= R, c=
v 0.7kJ/kg-K,
⇒ c p − c v =1 − 0.7 =R
R = 0.3 kJ/kg-K
Change in specific entropy
T2 P
s 2=
− s1 c p ln − R ln 2
T1 P1
600 500
= 1  ln − 0.3ln = 0.21 kj/kg-K
300 100

Q.24 (447.213)
.
kg/s, Q 5 kW ( heat loss )
.
= =
m 0.1
Applying SFEE
.  1  .
m  h1 + c12 + gz1  + Q =
 2 
.  1  .
m  h2 + c 22 + gz2  + w cv
 2 
.
=c1 0=
and w cv 0
z1 = z2 ( assume )
. . . . 1
m h1 + Q= m h 2 + m c 22
2
⇒ m  c 2= m ( h1 − h 2 ) + Q
.
. 1 2 .

2
1
⇒ 0.1  c 22 10−=3
0.1( 2500 − 2350 ) − 5
2
c 2 = 447.213 m/s

Q.25 (a)

© Copyright Reserved by Gateflix.in No part of this material should be copied or reproduced without permission
4 AVALILABILITY AND IRREVERSIBILITY

Q.1 Considering the relationship to equilibrium with a reference


Tds = dU + Pdv between the entropy environment, is called
(s), internal energy (u), pressure a)Entropy b) Enthalpy
(P), temperature (T) and volume (v), c) Exergy d) Rothalpy
which of the following statements is [GATE-2014 (ME) Set-1]
correct ?
a) It is applicable only for a Q.5 One side of a wall is maintained at
reversible process 400 K and the other at 300 K. The
b) For an irreversible process, rate of heat transfer through the
Tds > du + Pdv wall is 1000 W and the surrounding
c) It is valid only for an ideal gas temperature is 25℃. Assuming no
d) It is equivalent to Ist law, for a generation of heat within the wall,
reversible process the irreversibility (in W) due to heat
[GATE–2003(ME)] transfer through the wall is _______
[GATE-2015 (ME) Set-3]
Q.2 A steel billet of 2000 kg mass is to
be cooled from 1250 K to 450 K. The Q.6 One kg of an ideal gas (gas constant
heat released during this process is R = 287 J/kg.K) undergoes an
to be used as a source of energy. The irreversible process from state-1 (1
ambient temperature is 303 K and bar, 300 K) to state -2 (2 bar, 300 K).
specific heat of steel is 0.5 kJ/ kgK. The change in specific entropy (s2 –
The available energy of this billet is s1) of the gas (in J/kg. K) in the
a) 490.44 MJ b) 30.95 MJ process is ___________
c) 10.35 MJ d) 0.10 MJ
[GATE–2004(ME)] [GATE-2017 Set-2]

Q.7` A calorically perfect gas (specific


Q.3 The pressure, temperature and heat at constant pressure 1000
velocity of air flowing in a pipe are 5 J/kg.K) enters and leaves a gas
bars, 500 K and 50 m/s, turbine with the same velocity. The
respectively. The specific heats of temperatures of the gas at turbine
air at constant pressure and at entry and exit are 1100 K and 400 K.
constant volume are 1.005 kJ/ kg K respectively. The power produced is
and 0.718 kJ/ kg K, respectively. 4.6 MW and heat escapes at the rate
Neglect potential energy. If the of 300 kJ/s through the turbine
pressure and temperature of the casing. The mass flow rate of the gas
surrounding are 1 bar and 300 K, (in kg/s) through the turbine is.
respectively, the available energy in
kJ/kg of the air stream is a) 6.14 b) 7.00
a) 170 b) 187
c) 191 d) 213 c) 7.50 d) 8.00
[GATE–2013(ME)]
[GATE-2017 Set-2]
Q.4 The maximum theoretical work
obtainable, when a system interacts

© Copyright Reserved by Gateflix.in No part of this material should be copied or reproduced without permission
ANSWER KEY:
1 2 3 4 5 6 7
(d) (a) (b) (c) 248.23 -198.9 (b)

© Copyright Reserved by Gateflix.in No part of this material should be copied or reproduced without permission
EXPLANATIONS

Q.1 (d) interacts to equilibrium with dead


TdS=du+Pdv state or reference environment.
This equation holds good for any
process reversible or irreversible, Q.5 (248.23)
undergone by a closed system, since By Gauy-Studola theorem:
it is a relation among properties =I T0 (∆s) uni
which are independent of both. 1000 1000
(∆s) uni = −
300 400
Q.2 (a) = 0.8333 W/K
Q = mCρ ∆T
∴ Irreversibility:-I= (25 + 273) 0.833
Q=2000×(0.5kJ/kg K)×(1250-450) =248.23 W
= 800000 kJ Q.6 (-198.9)
T1 1250
∆=S mCln = 2000 × 0.5 × ln Change in specific entropy of ideal
T2 450
gas
=1021.165
A.E= Q − T0 (∆s) Tf P
Sf =
− Si Cp ln − R ln f
=490439.67 k Ti Pi
=490.44 MJ
2
Sf − Si =−287 ln  
Q.3 (b) 1
Available energy
 C2 C2  Sf − Si =−198.93J / kg.k
= ( h1 − h 0 ) − T0 ( s1 − s 0 ) +  1 − 0 
 2 2 
Q.7 (b)
….. (i)
T1 P1
s1 =
− s 0 Cp ln − Rln
T0 P0
 500  5
= 1.005 ln   − 0.287 ln  
 300  1
=0.05147
∴ eq.(i)
Available energy
=1.005 (500 – 300) 300(0.05147)
(50) 2
+ ×10−3
2 By SFEE
=186.8 kJ/ kg 2 2
= 187 kJ/ kg C1 C
h1 + + z1g + q = h 2 + 2 + z 2 g + w
2 2
Q.4 (c)
Exergy is also known as available C1 = C2 (given)
energy and it is maximum theoretical
work obtainable, when a system z1 = z 2 (assume)

© Copyright Reserved by Gateflix.in No part of this material should be copied or reproduced without permission
Q mC
 p T1 −=
mC 
 p T2 + W

W +Q 
 =
m
Cp [T2 − T1 ]

4600 + 300
=
1× [1100 − 400]

4900
=
m = 7kg / sec
700

© Copyright Reserved by Gateflix.in No part of this material should be copied or reproduced without permission
5 PURE SUBSTANCES

Q.1 Which combination of the following a) -Rln2 b) 0


statements is correct? c) Rln2 d) Rln4
P: A gas cools upon expansion only [GATE–2008]
when its Joule-Thomson
coefficient is positive in the Common Data For Q.4, 5 and 6
temperature range of expansion. In the figure shown, the system is a pure
Q: For a system undergoing a substance kept in a piston-cylinder
process, its entropy remains arrangement. The system is initially a two-
constant only when the process phase mixture containing 1 kg of liquid and
is reversible. 0.03 kg of vapour at a pressure of 100 kPa.
R: The work done by closed system Initially, the piston rests on a set of stops,
in an adiabatic is a point function. as shown in the figure. A pressure of 200
S: A liquid expands upon freezing kPa is required to exactly balance the
when the slope of its fusion weight of the piston and the outside
curve on pressure- Temperature atmospheric pressure. Heat transfer takes
diagram is negative. place into the system until its volume
a) R and S b) P and Q increases by 50%. Heat transfer to the
c) Q, R and S d) P, Q and R system occurs in such a manner that the
[GATE–2007] piston, when allowed to move, does so in a
very slow (quasi-static/quasi-equilibrium)
Q.2 Water has a critical specific volume
process. The thermal reservoir from which
of 0.003155m3/kg. A closed and
heat is transferred to the system has a
rigid steel tank of volume 0.025m3
temperature of 400°C. Average
contains a mixture of water and
temperature of the system boundary can be
steam at 0.1MPa. The mass of the
taken as 175°C. The heat transfer to the
mixture is 10 kg. The tank is now
system is 1 kJ, during which its entropy
slowly heated. The liquid level
increases by 10 J/K.
inside the tank
a) will rise
b) will fall
c) will remain constant
d) may rise or fall depending on the
amount of heat transferred
[GATE–2007]
Q.3 2 moles of oxygen are mixed
adiabatically with another 2 moles
of oxygen in mixing chamber, so that Specific volume of liquid (vf) and vapour
the final total pressure and (vg) phases, as well as values of saturation
temperature of the mixture become temperatures, are given in the table below.
same as those of the individual
constituents at their initial states.
The universal gas constant is given
as R. The change in entropy due to
mixing, per mole of oxygen, is given
by

© Copyright Reserved by Gateflix.in No part of this material should be copied or reproduced without permission
Q.4 At the end of the process, which one J m3
of the following situations will be true? a) b)
kg.K kg
a) superheated vapour will be left 5
in the system m Pa
c) d)
b) no vapour will be left in the
2
kgs kg
system [GATE-2015 (2)]
c) a liquid + vapour mixture will be
left in the system Q.10 A rigid container of volume 0.5 m3
d) the mixture will exist at a dry contains 1.0 kg of water at 120℃
saturated vapour state = =
(υf 0.00106 m3 / kg, ν g 0.8908 m3 / kg)
[GATE–2008]
The state of water is
Q.5 The work done by the system during a) Compressed liquid
the process is b) Saturated liquid
a) 0.1 kJ b) 0.2 kJ c) A mixture of saturated liquid and
c) 0.3 kJ d) 0.4 kJ saturated vapour
[GATE–2008] d) Superheated vapor
[GATE-2015 (3)]
Q.6 The net entropy generation
(considering the system and the dps
thermal reservoir together) during Q.11 For water at 25℃, = 0.189
dTs
the process is closest to
a) 7.5 J/K b) 7.7 J/K kPa/K (ps is the saturation pressure
c) 8.5 J/K d) 10 J/K in kPa and 𝑇𝑇s is the saturation
[GATE–2008] temperature in K) and the specific
volume of dry saturated vapour is
Q.7 A pure substance at 8 MPa and 43.38 m3/kg. Assume that the
400℃ is having a specific internal specific volume of liquid is
energy of 2864 kJ/kg and a specific negligible in comparison with that of
volume of 0.03432 m /kg. Its 3 vapour. Using the Clausius-
specific enthalpy (in kJ/ kg) is ____. Clapeyron equation, an estimate of
[GATE-2014 (2)] the enthalpy of evaporation of water
at 25℃ (in kJ/kg) is __________
Q.8 1.5 kg of water is in saturated liquid [GATE-2016 (1)]
state at 2bar (Vf=0.001061 m3/kg,
uf=504.0 kJ/kg, hf = 505kJ/kg). Heat Q.12 The INCORRECT statement about
is added in a constant pressure the characteristics of critical point of
process till the temperature of a pure substance is that
water reaches 400℃ (v=1.5493m3/ a) there is no constant temperature
kg, u=2967.0kJ/kg, h=3277.0 kJ/kg). vaporization process
The heat added (in kJ) in the process b) it has point of inflection with
is ____________. zero slope
[GATE-2014 (1)] c) the ice directly converts from
solid phase to vapor phase
Q.9 The Vander Waals equation of state d) saturated liquid and saturated
 a  vapor states are identical
is  p + 2  ( υ − b ) = RT , where p is [GATE-2016 (3)]
 υ 
Q.13 Which one of the following
pressure, υ is specific volume, T is statements is correct for a
temperature and R is characteristic superheated vapour?
gas constant. The SI unit of a is

© Copyright Reserved by Gateflix.in No part of this material should be copied or reproduced without permission
(A) Its pressure is less than the
saturation pressure at a given
temperature.

(B) Its temperature is less than the


saturation temperature at a given
pressure.

(C) Its volume is less than the


volume of the saturated vapour at a
given temperature.

(D) Its enthalpy is less than the


enthalpy of the saturated vapour at
a given pressure.

[GATE-2018 Set-1]

Q.14 A vehicle powered by a spark


ignition engine follows air standard
Otto cycle ( γ =1.4). The engine
generates 70 kW while consuming
10.3 kg/hr of fuel. The calorific
value of fuel is 44,000 kJ/kg. The
compression ratio is _______ (correct
to two decimal places).

[GATE-2018 Set-1]

Q.15 A tank of volume 0.05 m3 contains a


mixture of saturated water and
saturated steam at. The mass of the
liquid present is 8 kg. The entropy
(in kJ/kg K) of the mixture is
___________ (correct to two decimal
places).

Property data for saturated steam


and water are:

At 200 0 C,psat = 1.5538 MPa


=vf 0.001157
= m3 / kg,v g 0.12736 m3 / kg
=s fg 4.1014kJ
= / kg K, s f 2.3309kJ / kg K

[GATE-2018 Set-1]

© Copyright Reserved by Gateflix.in No part of this material should be copied or reproduced without permission
ANSWER KEY:
1 2 3 4 5 6 7 8 9 10 11 12 13 14 15
(a) (a) (b) (a) (d) (c) 3138 4158 (c) (c) 2443 (c) (a) 7.61 2.5

EXPLANATIONS
Q.1 (a) Given:=
T1 T=
2, p1 p2
Universal Gas constant = R
Q.2 (a) Here given oxygen are mixed
Given adiabatically
υcri = 0.003155m3 / kg, So, dQ = 0
= =
V 0.025m 3
, P 0.1MPa and dQ
We know = ds = 0 (since dQ=0)
M=10 kg. T
We know, Rigid means volume is
constant. Q.4 (a)
Specific volume, When the vapour is at a
V 0.025 temperature greater than the
υ= = = 0.0025m3 / kg
s
m 10 saturation temperature, it is said to
We see that the critical specific exist as superheated vapour. The
volume is more than the specific pressure and temperature of super
volume and during the heating heated vapour are independent
process, both the temperature and properties, since the temperature
the pressure remain constant, but may increase while the pressure
the specific volume increases to the remains constant.
critical volume (i.e. critical point). Here vapour is at 400℃ and
The critical point is defined as the saturation temperature is 200℃
point at which the saturated liquid So, at 200 kPa pressure,
and saturated vapour states are superheated vapour will be left in
identical. the system.

Q.5 (d)
= =
Given p1 100KPa, p 2 200kPa
= =
m l   1kg and m v   0.03 kg
0.03
x1 = = 0.029
1.03
v=
1 v f + x1v fg
= 0.001 + 0.0029(0.1 – 0.001)
So, point (B) will touch the = 0.0038
saturated liquid line and the liquid
= =
 V1 mv =1   0.0039   
1.03v
line will rise the Point O. 1
Now, given that heat transfer takes
Q.3 (b) place into the system until its
volume increases by % 50

© Copyright Reserved by Gateflix.in No part of this material should be copied or reproduced without permission
Therefore,= =
V2 1.5V1 0.00585 Psat @ T1 → saturation pressure at T1 temperature
Now Work done = P2 (V2 − V1 ) P1 → pressure of superheated vapour at state 1.
= 200(0.00585 - 0.0039) P1 < Psat @ T1
=0.4 kJ
Q.14 (7.61)
Q.6 (c)
−1000 γ = 1.4
Sgen = + 10
673 B.P. = 70 kN
= 8.514 J/K
.
m f = 10.3kg/hr
Q.7 (3138.56)
CV = 44000 kJ/kg
h = u + pv
h = 2864 + 8 ×103×0.03432 η= .
B.P.
=
70
 100 %
h = 3138.56 kJ/kg m f  CV 10.3
 44000
3600
Q.8 (4158) η = 0.556
Q = m (h2−h1)
1
=1.5(3277−505) ηotto =1− = 0.556
(r )
γ −1
= 4158 kJ
1
Q.9 (c) ⇒ 1− =
0.556
(r )
1.4 −1

Q.10 (c) 1
= 0.4439
(r )
0.4
0.5 3
=v = m / kg 0.5m3 / kg
1 r = 7.61
Since v f < v < v g the state of water
is mixture of saturated water and
saturated vapour. Q.15 (2.49)

Q.11 (2443.24)
By Causius-Clapeyron equation
dPs s g − s f h fg
= =
dTs vg Tsat × v g
∴ h fg= 0.189 × 298 × 43.38
h fg = 2443.24kJ / kg

Q.12 (c)
Q.13 (a)

© Copyright Reserved by Gateflix.in No part of this material should be copied or reproduced without permission
Volume of liquid = m f  Vf
0.001157 9.256  10−3 m3
8=
= 0.009256 m3
So volume of steam = 0.05 − 0.009256
= 0.040744 m3
Volume of steam
Mass of steam =
vg
0.040744
=
0.12736
= 0.319912 kg = ms
Total mass of mixture = m f + ms
= 8 + 0.319912
= 8.319912 kg
ms
dryness fraction = x=
m f + ms
0.319912
= = 0.03845
8.319912
So, the entropy of the mixture is given by
s = s f + x s fg
= 2.3309 + 0.03845  4.1014
= 2.488 kg/kgK

© Copyright Reserved by Gateflix.in No part of this material should be copied or reproduced without permission
6 POWER SYSTEM (RANKINE, BRAYTON, ETC.)

Q.1 The Rateau turbine belongs to the Q.5 In a gas turbine, hot combustion
category of` products with the specific heats Cp =
a) pressure compounded turbine 0.98 kJ/kgK, and Cv = 0.7538 kJ/kgK
b) reaction turbine enters the turbine at 20 bar, 1500 K
c) velocity compounded turbine and exits at 1 bar. The isentropic
d) radial flow turbine efficiency of the turbine is 0.94. The
[GATE–2001] work developed by the turbine per
Q.2 The efficiency of superheat Rankine kg of gas flow is
cycle is higher than that of simple a) 689.64 kJ/kg b) 794.66 kJ/kg
c) 1009.72 kJ/kg d) 1312.00 kJ/kg
Rankine cycle because
a) the enthalpy of main stream is [GATE–2003]
higher for superheat cycle Q.6 The compression ratio of a gas
b) the mean temperature of heat power plant cycle corresponding to
addition is higher for superheat maximum work output for the given
cycle temperature limits of Tmin and Tmax
c) the temperature of steam in the will be
condenser is high γ γ

d) the quality of steam in the  T  2( γ −1)  T  2( γ −1)


a)  max  b)  min 
condenser is low.  Tmin   Tmax 
[GATE–2002] γ −1 γ −1

Q.3 In Rankine cycle, regeneration T  γ T  γ


c)  max  d)  min 
results in higher efficiency because  Tmin   Tmax 
a) pressure inside the boiler [GATE–2004]
increases
b) heat is added before steam Common Data for Q.7 and Q.8:
enters the low pressure turbine Consider a steam power plant using a
c) average temperature of heat reheat cycle as shown. Steam leaves the
addition in the boiler increases boiler and enters the turbine at 4 MPa ,
d) total work delivered by the 3500C (h3 = 3095 kJ/kg). After expansion in
turbine increases the turbine to 400 kPa (h4 = 2609 kJ/kg),
[GATE–2003] the steam is reheated to 350℃ (h5 =
3170 kJ/kg) and then expanded in a low
Q.4 Considering the variation of static
pressure turbine to 10 kPa (h6 = 2165
pressure and absolute velocity in an
kJ/kg). The specific volume of liquid
impulse steam turbine, across one
handled by the pump can be assumed to be
row of moving blades
a) both pressure and velocity
decreases
b) pressure decreases but velocity
increases
c) pressure remains constant,
while velocity increases
d) pressure remains constant,
while velocity decreases
[GATE–2003]

© Copyright Reserved by Gateflix.in No part of this material should be copied or reproduced without permission
Q.7 The thermal efficiency of the plant Q.10 In the velocity diagram shown
neglecting pump work is below, u = blade velocity, C =
a) 15.8% b) 41.1% absolute fluid velocity and W =
c) 48.5% d) 58.6% relative velocity of fluid and the
[GATE–2004] subscripts 1 and 2 refer to inlet and
outlet. This diagram is for
Q.8 The enthalpy at the pump discharge
(h2) is
a) 0.33 kJ/kg b) 3.33 kJ/kg
c) 4.0 kJ/k d) 33.3 kJ/kg
[GATE–2004]
a) an impulse turbine
Q.9 A p -v diagram has been obtained
b) a reaction turbine
from a test on a reciprocating
c) a centrifugal compressor
compressor. Which of the following
d) an axial flow compressor
represents that diagram?
[GATE–2005]

Common data for Q.11 and Q.12

In two air standard cycles-one operating in


the Otto and the other on the Brayton cycle
air is isentropically compressed from 300
a) to 450K. Heat is added to raise the
temperature to 600K in the Otto cycle and
to 550K in the Brayton cycle.

Q.11 In ηo and ηB are the efficiencies of


the Otto and Bray ton cycles, then
a) ηo = 0.25, ηB = 0.18
b) ηo = ηB = 0.33
b)
c) ηo = 0.5, ηB = 0.45
d) it is not possible to calculate the
efficiencies unless the
temperature after the expansion
is given
[GATE–2005]

c) Q.12 If Wo and WB are work outputs per


unit mass, then
a) Wo > WB
b) Wo < WB
c) Wo = WB
d) it is not possible to calculate the
work outputs unless the
temperature after the expansion
d)
is given
[GATE–2005] [GATE–2005]

© Copyright Reserved by Gateflix.in No part of this material should be copied or reproduced without permission
Common Data for Q.13 and Q.14: Reason (R): The regenerative feed
The following table of properties was water heating raises the average
printed out for saturated liquid and temperature of heat addition in the
saturated vapour of ammonia. The titles for Rankin cycle.
only the first two columns are available. All a) Both (A) and (R) are true and (R)
that we know that the other columns is the correct reason for (A)
(column 3 to 8) contain data on specific b) Both (A) and (R) are true but (R)
properties, namely, internal energy (kJ/kg), is NOT the correct reason for (A)
enthalpy (kJ/kg) and entropy (kJ/kgK) c) Both (A) and (R) are false
d) (A) is false but (R) is true
[GATE–2006]

Q.17 Determine the correctness or


otherwise of the following
Assertion (A) and the Reason (R).
Assertion (A): Condenser is an
Q.13 The specific enthalpy data are in essential equipment in a steam
columns power plant.
a) 3 and 7 b) 3 and 8 Reason (R): For the same mass flow
c) 5 and 7 d) 5 and 8 rate and the same pressure rise, a
[GATE–2005] water pump requires substantially
less power than a steam compressor.
Q.14 When saturated liquid at 400C is a) Both (A) and (R) are true and (R)
throttled to – 20°C, the quality at is the correct reason for (A)
exit will be b) Both (A) and (R) are true and (R)
a) 0.189 b) 0.212 is NOT the correct reason for (A)
c) 0.231 d) 0.788 c) Both (A) and (R) are false
[GATE–2005] d) (A) is false but (R) is true
[GATE–2006]
Q.15 Given below is an extract from Q.18 Group I shows different heat
steam tables. addition process in power cycles.
Likewise, Group II shows different
heat removal processes. Group III
lists power cycles. Match items from
Groups I, II and III.
Group I Group II Group III
P. Pressure S. Pressure 1.Rankine Cycle
Specific enthalpy of water in kJ/kg Constant Constant
Q. Volume T. Volume 2. Otto cycle
at 150 bar and 45°C is Constant Constant
a) 203.60 b) 200.53 R. U. 3. Carnot cycle
c) 196.38 d) 188.45 Temperature Temperature
Constant Constant
[GATE–2006] 4. Diesel cycle
Q.16 Determine the correctness or 5.Brayton cycle

otherwise Assertion (A) and the a) P-S-5, R-U-3, P-S-1, Q-T-2


Reason (R) Assertion (A): In a b) P-S-1, R-U-3, P-S-4, P-T-2
power plant working on a Rankin c) R-T-3, P-S-1, P-T-4, Q-S-5
cycle, the regenerative feed water d) P-T-4, R-S-3, P-S-1, P-S-5
heating improves the efficiency of [GATE–2006]
the steam turbine.

© Copyright Reserved by Gateflix.in No part of this material should be copied or reproduced without permission
Q.19 Which combination of the following subscripts f and g denote saturated liquid
statements is correct? The state and saturated vapour state.
incorporation of reheater in a steam
power plant:
P: always increases the thermal
efficiency of the plant.
Q: always increases the dryness
fraction of steam at condenser inlet Q.21 The network output (kJ/kg) of the
R: always increases the mean cycle is
temperature of heat addition. a) 498 b) 775
S: always increases the specific c) 860 d) 957
work output. [GATE–2010]
a) P and S b) Q and S
c) P, R and S d) P, Q, R and S Q.22 Heat supplied (kJ/kg) to the cycle is
[GATE–2007] a) 2372 b) 2576
c) 2863 d) 3092
Q.20 A thermal power plant operates on a [GATE–2010]
regenerative cycle with a single
open feed water heater, as shown in Q.23 The values of enthalpy of steam at
the figure. For the state points the inlet and outlet of a steam
shown, the specific enthalpies are: turbine in a Rankine cycle are 2800
h1 = 2800 kJ/kg and h2 = 200 kJ/kg. kJ/kg and 1800 kJ/kg respectively.
The bleed to the feed water heater is Neglecting pump work, the specific
20% of the boiler steam generation steam consumption in kg/kWh is
rate. The specific enthalpy at state 3 is a) 3.60 b) 0.36
c) 0.06 d) 0.01
[GATE–2011]
Q.24 An ideal Brayton cycle, operating
between the pressure limits of 1 bar
and 6 bar, has minimum and
maximum temperature of 300 K and
1500 K. The ratio of specific heats of
the working fluid is 1.4. The
approximate final temperatures in
a) 720 kJ/kg b) 2280 kJ/kg Kelvin at the end of compression &
c) 1500 kJ/kg d) 3000 kJ/kg expansion processes are respectively
[GATE–2008] a) 500 and 900 b) 900 and 500
c) 500 and 500 d) 900 and 900
Common Data For Q.21 and Q.22 [GATE–2011]
In a steam power plant operating on the
Rankine cycle, steam enters the turbine at Q.25 Specific enthalpy and velocity of
4MPa, 350° C and exists at a pressure of 15 steam at inlet and exit of a steam
kPa. Then it enters the condenser and exits turbine, running under steady state,
as saturated water. Next, a pump feeds are as given below:
back the water to the boiler. The adiabatic
efficiency of the turbine is 90%. The
thermodynamic states of water and steam
are given in table h is specific enthalpy, s is
specific entropy and the specific volume;

© Copyright Reserved by Gateflix.in No part of this material should be copied or reproduced without permission
The rate of heat loss from the 1 1
a) 1 − b) 1 −
turbine per kg of steam flow rate is γ −1
r rpγ
p
5 kW. Neglecting changes in
1 1
potential energy of steam, the c) 1 − 1/ γ
d) 1 − (γ −1)/ γ
power developed in kW by the r p r p

steam turbine per kg of steam flow [GATE-2014 (2)]


rate is
a) 901.2 b) 911.2 Q.30 For a gas turbine power plant,
c) 17072.5 d) 17082.5 identify the correct pair of
statements.
[GATE–2013]
P. Smaller in size compared to
Common Data For Q.26 and 27 steam power plant for same
In a simple Brayton cycle, the pressure power output
ratio is 8 and temperatures at the entrance Q. Starts quickly compared to
of compressor and turbine are 300 K and steam power plant
1400 K, respectively. Both compressor and R. Works on the principle of
gas turbine have isentropic efficiencies Rankine cycle
equal to 0.8. For the gas, assume a constant S. Good compatibility with solid fuel
value of Cp (specific heat at constant a) P and Q b) R and S
pressure) equal to 1 kJ/kg-K and ratio of c) Q and R d) P and S
specific heats as 1.4. Neglect changes in [GATE-2014 (3)]
kinetic and potential energies.
Q.31 An ideal reheat Rankine cycle
Q.26 The power required by the operates between the pressure
compressor in kJ/kg of gas is limits of 10 kPa and 8 MPa, with
a) 194.7 b) 243.4 reheat being done at 4 MPa. The
c) 304.3 d) 378.5 temperature of steam at the inlets of
[GATE–2013] both turbines is 500℃ and the
enthalpy of steam is 3185 kJ/kg at
Q.27 The thermal efficiency of the cycle in the exit of the high pressure turbine
percentage (%) is and 2247 kJ/kg at the exit of low
a) 24.8 b) 38.6 pressure turbine. The enthalpy of
c) 44.8 d) 53.1 water at the exit from the pump is
[GATE–2013] 191 kJ/ kg. Use the following table
Q.28 In a power plant, water (density for relevant data
=1000 kg/ m3) is pumped from 80
kPa to 3 MPa. The pump has an
isentropic efficiency of 0.85.
Assuming that the temperature of
the water remains the same, the
specific work (in kJ/ kg) supplied to Disregarding the pump work, the
the pump is cycle efficiency (in percentage) is
a) 0.34 b) 2.48 ____
c) 2.92 d) 3.43 [GATE-2014 (1)]
[GATE-2014 (1)]
Q.32 In an ideal Brayton cycle,
Q.29 The thermal efficiency of an air- atmospheric air (ratio of specific
standard Brayton cycle in terms of heats,Cp /CV = 1.4, specific heat at
pressure ratio rp and γ (= Cp /Cv ) is
constant pressure = 1.005 kJ/kgK)
given by at 1 bar and 300 K is compressed to

© Copyright Reserved by Gateflix.in No part of this material should be copied or reproduced without permission
8 bar. The maximum temperature in the remaining steam expands
the cycle is limited to 1280 K. If the isentropically to 9 kPa
heat is supplied at the rate of 80 Inlet to turbine: P = 14MPa, T =
MW, the mass flow rate (in kg/s) of 560℃, h= 3486 kJ/kg, s = 6.6 kJ/kgK
air required in the cycle is Intermediate stage: h = 2776 kJ/kg
[GATE-2014 (2)] Exit of turbine: P = 9kPa,
hf =174 kJ/kg, hg =2574 kJ/kg,
Q.33 Steam at a velocity of 10m/s enters sf = 0.6kJ/kgK, sg = 8.1kJ/kgK
the impulse turbine stage with
If the flow rate of steam entering the
symmetrical blading having blade
turbine is 100 kg/s, then the work
angle 30°. The enthalpy drop in the
output (in MW) is ____.
stage is 100 kJ. The nozzle angle is
[GATE-2015 (1)]
20°.The maximum blade efficiency
(in percent) is ________.
Q.37 In a Rankine cycle, the enthalpies at
[GATE-2014 (2)]
turbine entry and outlet are 3159
kJ/kg. and 2187 kJ/kg respectively.
Q.34 Steam with specific enthalpy (h)
If the specific pump work is 2 kJ/kg,
3214 kJ/kg enters an adiabatic
the specific steam consumption (in
turbine operating at steady state
kg/kW-h) of the cycle based on the
with a flow rate 10 kg/s. As it
net output is _____.
expands, at a point where h is 2920
[GATE-2015 (2)]
kJ/kg, 1.5 kg/s is extracted for
heating purposes. The remaining 8.5
Q.39 The INCORRECT statement about
kg/s further expands to the turbine
regeneration in vapor power cycle is
exit, where h = 2374 kJ/kg.
that
Neglecting changes in kinetic and
a) it increases the irreversibility by
potential energies, the net power
adding the liquid with higher
output (in kW) of the turbine is
energy content to the steam
______.
generator
[GATE-2014 Set-4]
b) heat is exchanged between the
expanding fluid in the turbine
Q.35 Which of the following statements
and the compressed fluid before
regarding a Rankine cycle with
heat addition
reheating are TRUE?
c) the principle is similar to the
i) increase in average temperature
principle of Stirling gas cycle
of heat addition
d) it is practically implemented by
ii) reduction in thermal efficiency
providing feed water heaters
iii) drier steam at the turbine exit
[GATE-2016(1)]
a) only (i) and (ii) are correct
b) only (ii) and (iii) are correct
Q.40 In a steam power plant operating on
c) only (i) and (iii) are correct
an ideal Rankine cycle, superheated
d) (i),(ii) and (iii) are correct
steam enters the turbine at 3 MPa
[GATE-2015 (2)]
and 350℃. The condenser pressure
is 75 kPa. The thermal efficiency of
Q.36 Steam enters a well insulated
the cycle is ________ percent.
turbine and expands isentropically
Given data:
throughout. At an intermediate
For saturated liquid,
pressure, 20 percent of the mass is
extracted for process heating and

© Copyright Reserved by Gateflix.in No part of this material should be copied or reproduced without permission
At P=75kPa, hf =384.39kJ/kg,
νf =0.001037m /kg,
3

sf =1.213kJ/kg-K Q.44 In the Rankine cycle for a steam


At 75 kPa, hfg = 2278.6 kJ/kg, sfg = power plant the turbine entry and
6.2434 kJ/kg-K exit enthalpies are 2803 kJ/kg and
At P=3MPa & T=350℃ (superheated 1800 kJ/kg, respectively. The
steam), h = 3115.3 kJ/kg, s = 6.7428 enthalpies of water at pump entry
kJ/kg-K
and exit are 121 kJ/kg and 124
[GATE-2016(1)]
kJ/kg, respectively. The specific
Q.41 Consider a simple gas turbine steam consumption (in kg/k W.h) of
(Brayton) cycle and a gas turbine the cycle is ______
cycle with perfect regeneration. In
both the cycles, the pressure ratio is [GATE-2017 Set-2]
6 and the ratio of the specific heats
of the working medium is 1.4. The
ratio of minimum to maximum
temperatures is 0.3 (with
temperatures expressed in K) in the
regenerative cycle. The ratio of the
thermal efficiency of the simple
cycle to that of the regenerative
cycle is _________
[GATE-2016(2)]

Q.42 In a 3-stage air compressor, the inlet


pressure is 𝑝𝑝1, discharge pressure is
𝑝𝑝4 and the intermediate pressures
are 𝑝𝑝2 and 𝑝𝑝3 (p2 < 𝑝𝑝3). The total
pressure ratio of the compressor is
10 and the pressure ratios of the
stages are equal. If 𝑝𝑝1 = 100 kPa, the
value of the pressure 𝑝𝑝3 (in kPa) is
__________
[GATE-2016(3)]
Q.43 The Pressure ratio across a gas
turbine (for air, specific heat at
constant pressure,

c p = 1040J/kg.K and ratio of specific


heats, γ = 1.4 ) is 10. If the inlet
temperature to the turbine is 1200K
and the isentropic efficiency is 0.9,
the gas temperature at turbine exit
is ______ K.

[GATE-2017 Set-1]

© Copyright Reserved by Gateflix.in No part of this material should be copied or reproduced without permission
ANSWER KEY:
1 2 3 4 5 6 7 8 9 10 11 12 13 14
(a) (b) (c) (d) (a) (a) (b) (d) (d) (b) (b) (a) (d) (b)
15 16 17 18 19 20 21 22 23 24 25 26 27 28
(d) (a) (b) (a) (b) (a) (c) (c) (a) (a) (a) (c) (a) (d)
29 30 31 32 33 34 35 36 37 38 39 40 41 42
(d) (a) 40.73 108.07 88.3 7581 (c) 125.56 (a) 3.7 26.01 0.8 464.14

43 44
679 3.6

© Copyright Reserved by Gateflix.in No part of this material should be copied or reproduced without permission
EXPLANATIONS
Q.1 (a) Q.4 (d)
The Rateau turbine is a pressure In impulse turbine, the steam
compounded turbine. pressure remains constant while it
flows through the moving blades,
Q.2 (b) where only the kinetic energy
converts into mechanical energy.

The average temperature at which


heat is transferred to steam can be
increased without increasing the
boiler pressure by superheating the
steam to high temperatures. The
effect of superheating on the
performance of vapour power cycle
is shown on a T - s diagram. Thus
both the net work and heat input
increase as a result of superheating Q.5 (a)
the steam to a higher temperature. ∴ Given, T3 = 1500 K
The overall effect is an increase in Cp = 0.98 kJ/kg K
thermal efficiency. CV = 0.7538 kJ/ kg K.
Since, the average temperature at Cp
which heat is added increases. ∴γ= = 1.3
CV
Q.3 (c)

γ −1
The object of the regenerative feed T4  P4  r

heating cycle is to supply the Now, = 


T3  P3 
working fluid to the boiler at same 0.3
state between 2 and 2’ (rather than T4  1  1.3
at state 2) there by increasing the ⇒ = 
1500  20 
average temperature of heat
T4 = 751.37K
addition to the cycle.
Now turbine efficiency;

© Copyright Reserved by Gateflix.in No part of this material should be copied or reproduced without permission
T3 − T4' 1
η=  T  2x
T3 − T4 ∴ rp =  3 
 T1 
1500 − T4' γ
⇒0.94 =
1500 − 751.34  T  2( γ−1)

⇒ rp =  max 
∴ T4 = 796.25 K  Tmin 
Turbine work;
Wt = cp (T3 − T4′ ) Q.7 (b)
= 0.98 (1500 – 796.25) Power obtained from plant
= 689.67 kJ/kg K = (h 3 − h 4 ) + (h 5 − h 6 )
Heat supplied to the plant.
Q.6 (a) = (h3 − h1 ) − (h5 − h4 )
Thermal efficiency
Power obtained
=
Heat supplied

=
( 3095 − 2609 ) + (3170 − 2165)
( 3170 − 2609 ) + (3095 − 29.3)
=0.4111=41.11%
Q.8 (d)
Work output during the cycle Enthalpy at exit of pump must be
= mCp (T3 − T2 ) − mCp (T4 − T1 ) greater than enthalpy at inlet of
= mCp (T3 − T4 ) − mCp (T2 − T1 ) pump i.e. h2 must be greater, then h1
=29.3kJ/kg. Among the given four
  T  T 
= mCp T3 1 − 4  − T1  2 − 1  options only one option is greater
  T3   T1   than h1 =29.3kJ/kg, which is
Since . 33.3kJ/kg. Hence option (D) is
γ−1 correct.
= ( rp ) γ
T3 T2
= = Or Pump work = vdP
T4 T1 1
 γ −1  = (Boiler pressure Condenser
ρ
 Let= x,= mCp K 
 γ  pressure)
∴ Work/cycle 1
= 3 (4000 10)=3.99 kJ/kg
  1 T  10
= K T3 1 − x  − T1  x4 − 1  h2 =29.3 + 3.99 = 33.3 kJ/kg
  rp  r
 p


For maximum power, differentiate Q.9 (d)
with respect to
dW  x 
⇒ = K T3 × ( x −1) − T1xrp( x −1)  = 0
drp  rp 
(For max.)
= T1x ( rp )
xT3 ( x −1)
⇒ ( x +1)
rp
T
⇒ rp2x = 3
T1

© Copyright Reserved by Gateflix.in No part of this material should be copied or reproduced without permission
From above figure, we can easily see T2  V1 
γ −1

that option (d) is same. = 


T1  V2 
Q.10 (b)
Velocity of flow is constant T T 
throughout the stage and the So, 3 =  2 
T4  T1 
diagram is symmetrical hence it is a
T 600
diagram of reaction turbine. T4 = 3 × T1 = × 300 = 400K
T2 450
Q.11 (b) And
We know that efficiency. W0 = CV (600 – 450) −CV (400–300)
T = 150 CV – 100 CV = 50 CV ...(ii)
ηOtto = ηBrayton = 1 − 1
T2 From P − v diagram of Brayton
300 cycle, work done, is,
ηOtto = ηBrayton = 1 − WB = Q1– Q2=Cp(T3–T2) – Cp (T4 – T1)
450 And
6
=1 − =0.33 T
T4 = 1 × T3 =
300
× 550 = 366.67K
9 T2 450
So,= ηOtto η= 33%
W= Cp ( 550 − 450 ) − ( 366.67 − 300 )
Brayton
s

Q.12 (a) = 33.33 cp ... (iii)


Dividing equation (ii) by (iii), we get
WO 50C v 50
= =
WB 33.33Cp 33.33γ
Cp
= γ and γ=1.4
Cv
50 50
= = >1
33.33 ×1.4 46.662
From this, we see that,
WO > WB

Q.13 (d)
From saturated ammonia table
column 5 and 8 are the specific
enthalpy date column.

From the previous part of the Q.14 (b)


question The enthalpy of the fluid before
T3(Otto) = 600 K, T3(Brayton) = throttling is equal to the enthalpy of
fluid after throttling because in
550 K,
throttling process enthalpy remains
From the P − v diagram of Otto
constant.
cycle, we have
h1 = h2
W0=Q1–Q2 = CV(T3– T2) − CV (T4–T1)
371.43 = 89.05 + x (1418 – 89.05)
...(i)
For process 3 – 4, h= hf + x ( hg − hf )
γ −1 γ −1
T3  V4   V1  = 89.05 + x (1328.95)
=  =   = V4 V=
1 , V3 V2 282.38
T4  V5   V2  =x = 0.212
1328.95
For process 1 – 2,

© Copyright Reserved by Gateflix.in No part of this material should be copied or reproduced without permission
Q.15 (d)
When the temperature of a liquid is and the same pressure rise, water
less than the saturation temperature pump require very less power
at the given pressure, the liquid is because the specific volume of liquid
called compressed liquid (state 2 in is very less as compare to specific
figure).The pressure and temperature volume of vapour.
of compressed liquid may vary
independently and a table of Q.18 (a)
properties like the superheated
vapour table could be arranged, to Q.19 (b)
give the properties at any p and T. Whether the cycle efficiency
The properties of liquids vary little increases or not depends upon the
with pressure. Hence, the properties mean temperature of heat addition.
are taken from the saturation table In practice the use of reheat only
at the temperature of the compressed gives a small increase in cycle
liquid. So, from the given table at T = efficiency, but it increases the net
450 C, Specific enthalpy of water work output by making possible the
=188.45 kJ/kg. use of higher pressures, keeping the
quality of steam at the outlet of the
turbine within permissible limits.

Q.20 (a)
Given: 2800kJ/kg h2=200kJ/ kg
From the given diagram of thermal
power plant, point 1 is directed by
the Boiler to the open feed water
heater and point 2 is directed by the
pump to the open feed water heater.
Q.16 (a)
The bleed to the feed water heater is
20% of the boiler steam generation
Q.17 (b)
i.e. 20% of h1
(a) Condenser is essential
equipment in a steam power plant
because when steam expands in the
turbine and leaves the turbine in the
form of super saturated steam. It is
not economical to feed this steam
directly to the boiler. So, condenser
is used to condense the steam into
water and it is an essential part So,h2 = 20% of h1+80% of h2
(equipment) in steam power plant. = 0.2×2800 + 0.8× 200 = 720 kJ/kg
Assertion (a) is correct.
(b) The compressor and pumps Q.21 (c)
require power input. The compressor
h1 = 3092.5kJ / kg
is capable of compressing the gas to
very high pressures. Pump work hf + x ( hg − hf )
h2 =
very much like compressor except
s2 sf 2 + x ( sg 2 − sf 2 )
Now, s1 ==
that they handle liquid instead of
gases. Now for same mass flow rate

© Copyright Reserved by Gateflix.in No part of this material should be copied or reproduced without permission
6.5821 =0.7549 + x ( 8.0085 − 0.7549 ) 𝛾𝛾 = 1.4
x = 0.8033
Dryness fraction at exit of turbine

γ −1
T2  P2  γ
= 
T1  P1 
0.4
T2  6  1.4
Hence, = 
h2=225.94+0.8033(2599.1-225.94) 300  1 
h2=2132.3kj/kg T2 = 500.5K
Turbine work = h1- h2 0.4
T3  6  1.4
=3092.5 – 2132.3 = 
=960.2 kJ/kg T4  1 
W= Wth × ηisen ⇒ T4 = 900K
act
= 0.9×960.2
Q.25 (a)
= 864.2kJ/kg
Applying SFEE
Pump work =vdP
= 0.001014 (4000 – 15) mV12 mV22
mh1 + + Q =mh 2 + +W
= 4.04079 kJ/kg 2 2
Net work= Turbine work – Pump 1802 52
work ⇒ 3250 + − 5 = 2360 + +W
2000 2000
= 864.2 – 4.04 (Since m = 1 kg/s)
= 860.16 kJ/kg ⇒ W = 901.2 kW
Q.22 (c) Q.26 (c)
Heat supplied =h1-h4
h4=h3+Pump work
=225.94 + 4.04078
= 229.98 kJ/kg
Heat supplied = h1-h4
= 3092.5 229.98
= 2862.519 kJ/kg

Q.23 (a) WC = Cp (T2’−T1)


Specific steam consumption T2
γ −1
γ
(kg/kWh) = (rp )
T1
3600 3600
= = 0.4
Network Done ( 2900 − 1900 ) ⇒= T2 300 × (8) 1.4
=3.6kg/kWh T2 = 543.43K
Efficiency of compressor
Q.24 (a) T −T
P1 = 1bar Tminimum= 300 K ηc = 2' 1
T2 − T1
P2 = 6 bar Tmaximum = 1500 K

© Copyright Reserved by Gateflix.in No part of this material should be copied or reproduced without permission
543.43 − 300 Q.30 (a)
0.8=
T2 ' − T1
Q.31 (40.73)
T2 ′ = 604.28k
Wc = 1(604.28 − 300)
Wc = 304.28 kJ/kg
Power required by the compressor

Q.27 (a)
γ −1
T3
= (γ p ) γ
T4
1400
=
⇒ T4 = 0.4
772.86K
1.4
(8) Given,
Efficiency of turbine
T −T ' P2=10kPa
ηT = 3 4
T3 − T4 P1=8kPa
P3=4kPa
1400 − T4 '
0.8 = h2s=3185 kJ/kg
T3 − T4 h4s= 2247 kJ/kg
1400 − T4 ' h1=3399 kJ/kg
0.8 = h3=3446 kJ/kg
1400 − 772.86
T4 ′ = 898.288 K h6s=191 kJ/kg
Work of turbine, WT = (h1− h2s) + (h3− h4s)
WT = Cp (T3 − T4 ′) QS = (h1− h6s) + (h3− h2s)
∴WT = (3399 − 3185) + (3446 − 2247)
= 1 (1400 – 898.288)
=1413 kJ/kg
= 501.712 kW/ kg
QS = (3399−191) + (3446−3185)
Thermal efficiency of the cycle
=3469 kJ/kg
W − WC
ηT = T WT 1413
Q =
η =
Qs 3469
501.712 − 304.28
= = 24.8% = 0.4073
1(1400 − 604.28) Or = 40.73%

Q. 28 (d) Q.32 (108.071)


work isen = vdp
1
= ( P2 − P1 )
ρwater

=
( 3000 − 80 )
1000
2920
= = 2.92kJ / kg Cp
1000 Given, = γ = 1.4
w 2.92 Cv
work act= = = 3.43
η 0.85 Cp = 1.005 kJ/ kgK
P1= 1bar
Q.29 (d) T1= 300K
P2= 8bar

© Copyright Reserved by Gateflix.in No part of this material should be copied or reproduced without permission
T3 = Tmax = 1280 K
For process 1-2, Q.36 (125.56)
γ−1
T2  P2  0.4 γ
= =  8 1.4
T1  P1 
= 1.8114
∴ T2 = 300×1.8114
= 543.43K
s1 = s3
=
We know that Qs mC  p ( T3 − T2 )
⇒ 6.6 = 0.6 + x3 [7.5]
= 80MW (given) x3 = 0.8
80 ×106 h3 = 174 + 0.8 [2574 − 174]
m =
1.005 ×103 (1280 − 543.43) = 2094 kJ/kg
= 108.071 kg/s ∴ W = 100[h1 − h2 ] + 80[h2 − h3 ]
=100[3486-2776]+80[2776- 2094]
Q.33 (88.3) = 125560 kW = 125.56 MW
We know that for maximum blade
efficiency, Q.37 *
cos α
ρ= Q.38 (3.7)
2
u 3600
where ρ = Specific steam consumption =
V1 Wnet
And maximum balde efficiency is Now, W= T h 2 − h1
given by: = 3159 2187
( ηb )max = cos 2 α ∴ WT = 972kJ / kg
= (=
cos 20 ) o 2
0.883 And Wp = 2kJ / kg
= 88.3% Thus specific steam consumption
3600
= kg/ kWh= 3.7815 kg/ kWh
Q.34 (7581) 972 − 2

Q.39 (a)

Q.40 (26.01)

m1 = 10kg/s
m3 = 1.5kg/s
m2 = 8.5kg/s
h1 = 3214kJ/kg
h2 = 2920kJ/kg
h3 = 2374 kJ/kg For process 1→ 2:-
WT = m1 (h1− h2) + m2 (h2 − h3) s1 = s 2
WT=10(3214−2920)+8.5(2920 6.7428 = 1.213 + x 2 (6.2434)
2374) x 2 = 0.8857
WT = 7581kW
∴ h 2 = 384.39 + 0.8857[2278.6]
Q.35 (c) h 2 = 2408.54kJ / kg

© Copyright Reserved by Gateflix.in No part of this material should be copied or reproduced without permission
W= h1 − h 2 Where N-Number of stages
∴ ( rp ) = 3 10
T
= 3115.3 2402.54 s
WT = 712.76kJ / kg
(r )
p s = 2.1544
WP v f [ P2 − P1 ]
=
p2
= 0.001037 [3000 75] = 2.1544
p1
WP = 3.033kJ / kg
=p 2 2.1544 ×100
Wnet = WT − WP = 709.727kJ / kg = 215.44 kPa
Q= h1 − h 4 ps
3
Also = 2.1544
Also, W= p h4 − h2 p2
∴ h= 4 Wp + h 2 = p3 2.1544 × 215.44
= 3.033+384.3 ∴
h 4 = 387.423kJ / kg p3 = 464.14kPa
=
∴ Qs 3115.3 − 387.423 Q.43 (679.38)
= 2727.87 kJ/ kg
W
ηth = net
Qs
709.727
=
∴ ηth = 0.2601
2727.87
ηth =26.01%

Q.41 (0.8)
Efficiency without regeneration:- T − T4
ηisen =3
1 T3 − T4 '
ηth =1 −
( rp )
γ−1/ γ

P2 P3
1 Q = = 10
ηth =1 − = 0.4006 P1 P4
( 6)
0.4/1.4

r −1
Efficiency with regeneration:- T4 '  P4  r
γ−1 = 
1 − ( rp ) γ min
T
( ηth )reg = T3  P3 
Tmax
1 − ( 6)
1.4 −1
= × 0.3
0.4/1.4
 1  1.4
T4 ' = 1200  
= 0.4994 ≈ 0.5  10 
ηth 0.4
=Ratio =
( ηth )reg 0.5 T4 ' = 621.53k
= 0.8
0.9[1200 − 621.53] =
T3 − T4
Q.42 (464.14)
T4 = 679.38k
We know that in multi-stage
compression Q.44 (3.6)
(r ) = (r )
N
p 0 p s

( r ) = ( rp )
p s
1/ N
0

© Copyright Reserved by Gateflix.in No part of this material should be copied or reproduced without permission
Given

h1 = 2803kJ / kg

h 2 = 1800kJ / kg

h 3 = 121kJ / kg

h 4 = 124kJ / kg

WT = h1 − h 2 = 2803 − 1800

WT = 1003kJ / kg

W=
p h 4 − h3

= 124 − 121
Wp = 3kJ / kg

W=
net WT − Wp

= 1003 − 3

Wnet = 1000kJ / kg

3600 kg
Q SSC =
Wnet kwh

3600
=
SSC = 3.6kg / kwh
1000

© Copyright Reserved by Gateflix.in No part of this material should be copied or reproduced without permission
7 IC ENGINE

Q.1 A single-acting two-stage Q.5 A diesel engine is usually more


compressor with complete inter efficient than a spark ignition engine
cooling delivers air at 16 bar. because
Assuming an intake state of 1 bar at a) diesel being a heavier
15°C, the pressure ratio per stage is hydrocarbon, releases more heat
a) 16 b) 8 per kg than gasoline
c) 4 d) 2 b) the air standard efficiency of
[GATE–2001] diesel cycle is higher than the
Otto cycle, at a fixed
Q.2 In a spark ignition engine working compression ratio
on the ideal Otto cycle, the c) the compression ratio of a diesel
compression ratio is 5.5. The work engine is higher than that of an
output per cycle (i.e., area of the P-v SI engine
diagram) is equal to d) self ignition temperature of
23.625×105×Vc(in Joules), where Vc diesel is higher than that of
is the clearance volume in m3. The gasoline
indicated mean effective pressure is [GATE–2003]
a) 4.295 bar b) 5.250 bar
c) 86.870 bar d) 106.300 bar Q.6 An automobile engine operates at a
[GATE–2001] fuel air ratio of 0.05, volumetric
efficiency of 90% and indicated
Q.3 An ideal air standard Otto cycle has thermal efficiency of 30%. Given
a compression ratio of 8.5. If the that the calorific value of the fuel is
ratio of the specific heats of air (𝛾𝛾) is 45 MJ/kg and the density of air at
1.4, what is the thermal efficiency in intake is 1 kg/m3, the indicated
percentage of the Otto cycle ? mean effective pressure for the
a) 57.5 b) 45.7 engine is
c) 52.5 d) 95 a) 6.075 bar b) 6.75 bar
[GATE–2002] c) 67.5 bar d) 243 bar
[GATE–2003]
Q.4 For a spark ignition engine, the
equivalence ratio (∅) of mixture Q.7 For an engine operating on air
entering the combustion chamber standard Otto cycle, the clearance
has values volume is 10% of the swept volume.
a) ∅ < 1 for idling and ∅ > 1 for The specific heat ratio of air is 1.4.
peak power conditions The air standard cycle efficiency is
b) ∅> 1 for both idling and peak a) 38.3% b) 39.8%
power conditions c) 60.2% d) 61.7%
c) ∅ > 1 for idling and ∅ < 1 for [GATE–2003]
peak power conditions
d) ∅ < 1 for both idling and peak Q.8 At the time of starting, idling and
power conditions low speed operation, the carburetor
[GATE–2003] supplies a mixture which can be
termed as

© Copyright Reserved by Gateflix.in No part of this material should be copied or reproduced without permission
a) Lean Q.12 Which one of the following is NOT a
b) slightly leaner than stoichiometric necessary assumption for the air-
c) stoichiometric standard Otto cycle?
d) rich a) All processes are both internally
[GATE–2004] as well as externally reversible.
Q.9 During a Morse test on a 4 cylinder b) Intake and exhaust processes are
engine, the following measurements constant volume heat rejection
of brake power were taken at processes.
constant speed. c) The combustion process is a
All cylinders firing 3037 kW constant volume heat addition
Number 1 cylinder not firing 2102 process.
kW d) The working fluid is an ideal gas
Number 2 cylinder not firing 2102 with constant specific heats
kW [GATE–2008]
Number 3 cylinder not firing 2100
kW Q.13 In an air-standard Otto-cycle, the
Number 4 cylinder not firing 2098 compression ratio is 10. The
kW condition at the beginning of the
The mechanical efficiency of the compression process is 100 kPa and
engine is 27°C. Heat added at constant
a) 91.53% b) 85.07% volume is 1500 kJ/kg, while 700
c) 81.07% d) 61.22% kJ/kg of heat is rejected during the
[GATE–2004] other constant volume process in
the cycle. Specific gas constant for
Q.10 An engine working on air standard air = 0.287 kJ/kg K. The mean
Otto cycle has a cylinder diameter of effective pressure (in kPa) of the
10 cm and stroke length of 15 cm. cycle is
The ratio of specific heats for air is a) 103 b) 310
1.4. If the clearance volume is 196.3 c) 515 d) 1032
cc and the heat supplied per kg of [GATE–2009]
air per cycle is 1800 kJ/kg, the work
output per cycle per kg of air is Q.14 A turbo-charged four-stroke direct
a) 879.1 kJ b) 890.2 kJ injection diesel engine has a
c) 895.3 kJ d) 973.5 kJ displacement volume of 0.0259 m3
[GATE–2004] (25.9 litres). The engine has an
output of 950 kW at 2200 rpm. The
Q.11 The stroke and bore of a four stroke mean effective pressure (in MPa) is
spark ignition engine are 250 mm closest to
and 200 mm respectively. The a) 2 b) 1
clearance volume is 0.001m3. If the c) 0.2 d) 0.1
specific heat ratio � = 1.4, the air- [GATE–2010]
standard cycle efficiency of the
engine is Q.15 The crank radius of a single-cylinder
a) 46.40% b) 56.10% I.C. engine is 60 mm and the
c) 58.20% d) 62.80% diameter of the cylinder is 80 mm.
[GATE–2007] The swept volume of the cylinder in
cm3 is
a) 48 b) 96
c) 302 d) 603

© Copyright Reserved by Gateflix.in No part of this material should be copied or reproduced without permission
[GATE–2011] If 𝛾𝛾 and T denote the specific heat
ratio and temperature, respectively,
Q.16 In an air-standard Otto cycle, air is the efficiency of the cycle is
supplied at 0.1 MPa and 308 K. The T −T T −T
a) 1 − 4 1 b) 1 − 4 1
ratio of the specific heats (γ) and the T3 − T2 γ[T3 − T2 ]
specific gas constant (R) of air are
1.4 and 288.8 J/kgK respectively. If c) 1 −
γ[T4 − T1 ]
d) 1 −
( T4 − T1 )
the compression ratio is 8 and the T3 − T2 (γ − 1) ( T3 − T2 )
maximum temperature in the cycle [GATE-2015 (3)]
is 2660 K, the heat (in kJ/kg)
supplied to the engine is ________ Q.21 For the same values of peak
[GATE-2014 (1)] pressure, peak temperature and
Q.17 A diesel engine has a compression heat rejection, the correct order of
ratio of 17 and cut-off takes place at efficiency for Otto, Dual and Diesel
10% of the stroke. Assuming ratio of cycles is
specific heats (γ) as 1.4, the air– a) ηOtto > ηDual > ηDiesel
standard efficiency (in percent) is b) ηDiesel > ηDual > ηOtto
_____________. c) ηDual > ηDiesel > ηOtto
[GATE-2014 (3)] d) ηDiesel > ηOtto > ηDual
[GATE-2015 (2)]
Q.18 In a compression ignition engine, Q22 Air contains 79% N2 and 21% O2 on a
the inlet air pressure is 1 bar and molar basis. Methane (CH4) is burned
the pressure at the end of isentropic with 50% excess air than required
compression is 32.42 bars. The stoichiometrically. Assuming
expansion ratio is 8. Assuming ratio complete combustion of methane, the
of specific heats (γ) as 1.4, the air molar percentage of N2 in the
standard efficiency (in percent) is products is _________
_______________.
[GATE-2014 Set-4] [GATE-2017 Set-1]

Q.23 An engine working on air standard


Q.19 Air enters a diesel engine with a
Otto cycle is supplied with air at 0.1
density of 1.0 kg/m3. The
compression ratio is 21. At steady MPa and 350 C . The compression ratio
state, the air intake is 30×10−3 kg/s is 8. The heat supplied is 500 kJ/kg.
and the net work output is 15 kW. Property data for air: cp = 1.005 kJ/kg
The mean effective pressure (kPa) is
______. K, c v = 0.718 kJ/kg K, R = 0.287 kJ/kg
[GATE-2015 (1)] K. The maximum temperature (in K)
of the cycle is _________ (correct to one
Q.20 An air-standard Diesel cycle consists decimal place).
of the following processes:
1-2 : Air is compressed [GATE-2018 Set-1]
isentropically.
2-3 :Heat is added at constant
pressure.
3-4 :Air expands isentropically to
the original volume.
4-1:Heat is rejected at constant
volume.

© Copyright Reserved by Gateflix.in No part of this material should be copied or reproduced without permission
ANSWER KEY:
1 2 3 4 5 6 7 8 9 10 11 12 13 14
(c) (b) (a) (b) (c) (a) (d) (d) (c) (d) (c) (b) (d) (a)
15 16 17 18 19 20 21 22 23
(d) 525 (b) (b) 73.8 1403

© Copyright Reserved by Gateflix.in No part of this material should be copied or reproduced without permission
EXPLANATIONS

Q.1 (c) For same compression ratio and the


Given that the intercooling is perfect same heat supplied, Otto cycle is
∴ P2 = �P1 P3 most efficient and diesel cycle is
P2 = √1 × 16 least efficient.
P2 = 4 bar In practice, however, the
P2 P3 compression ratio of the Diesel
= =
Pressure ratio per stage = 4 engine ranges between 14 and 25
P1 P2
whereas that of the Otto engine
between 6 and 12. Because of it the
Q.2 (b) efficiency of Diesel cycle is higher
W W efficiency than that of Otto engine.
mep = =
Vs V1 − VC
23.625 ×105 × VC Q.6 (a)
= actual volume
5.5VC − VC Volumetric efficiency =
swept volume
23.625 ×105 VC
= Va
4.5VC = = 0.9
Vs
=5.25× 105 Pa ∴ Va = 0.9Vs
=5.250 bar Mass of air,
m a ρ=
= air Vs 0.9Vs
Q.3 (a)
γ −1 mf = 0.05 × 0.9Vs = 0.045Vs
1 Pmep × LAN
η= 1 −  
r ηthermal =
m f × C.V
0.4
 1  Pmep × Vs
= 1−   ⇒ 0.3 =
 8.5  0.045vs × 45 ×106
= 0.5751 or 57.51%
∴ Pmep = 6.075bar
Q.4 (b)
Equivalence ratio is defined as the Q.7 (d)
γ −1
actual fuel air ratio to stoichiometric v 
fuel air ratio ηOtto = 1 −  c 
 v1 
F γ −1
   Vc 
φ =  actual
A = 1−  
F  Vc + Vs 
  Where Vc = clearance vol
 A Stoi
For both idling and peak power = 10% of Vs =0.1Vs
γ −1
conditions the actual fuel-air ratio  0.1Vs 
required is more than the ∴ ηOtto = 1 −  
 0.1Vs + Vs 
stoichiometric fuel-air ratio. 1.4 −1
 0.1 
= 1−  
Q.5 (c)  1.1 
=0.6167=61.67%

© Copyright Reserved by Gateflix.in No part of this material should be copied or reproduced without permission
Cp
Q.8 (d) = ,γ
v c 0.001m3= = 1.4
Cv
Q.9 (c)
When cylinder 1 is not firing then Swept Volume
power is 2102 kW and when all π
Vs = A × L = (D) 2 × L
cylinders are firing then power is 4
3037 kW. π
= (0.2) 2 × 0.25= 0.00785m3
Hence power supplied by cylinder 4
No.1= 3037-2102 = 935 kW Compression ratio
Similarly power supplied by v r v c + vs
cylinder No. 2 = 3037-2102 = 935 kW =
r =
vc vc
Similarly power supplied by
0.001 + 0.00785
cylinder No. 3 = 3037-2100 = 937 kW =
Similarly power supplied by 0.001
cylinder No.4 = 3037-2098 = 939 kW = 8.85
IPtotal = 935 + 935 + 937 + 939 = 3746 Air standard efficiency
1 1
3037 η= 1 − γ −1 = 1−
Hence ηmech = (r) (8.85)1.4−1
935 + 935 + 937 + 939
1
ηmech =
BP = 1− = 1 − 0.418 = 0.582
IP 2.39
or 58.2 %
3037
= = 0.8107
3746 Q.12 (b)

∴ ηmech =81.07%

Q.10 (d)
r −1
 Vc 
ηOtto = 1 −  
 Vc + Vs 
VC = 196.3cc Assumptions of air standard otto
π π cycle
Vs = × D 2 L = ×102 ×15 a) All processes are internally
4 4
reversible.
=1178.097cc
b) Air behaves as ideal gas
𝛾𝛾=1.4
1.4 −1
c) Specific heats remains constant
 196.3  (Cp & Cv)
∴ ηOtto = 1 −  
 196.3 + 1178.097  d) Intake process is constant
=0.5408 volume heat addition process
∴ ηOtto = 54.08% and exhaust pro-cess is constant
∴ Work output volume heat rejection process.
= ηOtto × (Heat supplied) Intake process is a constant
volume heat addition process,
= 0.5408 ×1800 =973.58kJ
from the given options; option
(2) is incorrect.
Q.11 (c)
Given L=250 mm = 0.25 m, D = 220 Q.13 (d)
m = 0.2 m.

© Copyright Reserved by Gateflix.in No part of this material should be copied or reproduced without permission
Pm LAn
Power = kw
60, 000
As four stroke
N
n= = 1100rpm
2
950 × 60000
Pm =
P1 = 100 kPa 0.0259 ×1100
Compression ratio, = 2 ×106 N/m2 = 2 MPa
r = 10
T1 = 27 + 273 = 300K Q.15 (d)
Heat added, Given : r = 60 mm, D = 80 mm= 8 cm
Q S = 1500kJ/kg Stroke length,
Heat rejected, L = 2r = 2 × 60 = 120 mm =12 cm
Q R =700kJ/kg Swept Volume, Vs = A × L
Mean effective pressure π π 2
Work done per cycle = D 2 ×= L (8) ×12
= 4 4
Swept volume = 603.18 ≃ 603 cm3
Compression ratio,
r = V1 /V2 = 10 Q.16 (1400 to 1420)
V1 = 10V2
Swept volume
= V1 − V2
V
= V1 − 1 = 0.9V1
10
For initial air
P1 V1 = RT1 Given,
⇒ V1 = 1
RT P1 = 0.1 MPa
P1 T1= 308 K
0.287kJ / kgK × 300K 𝛾𝛾 = 1.4
= R = 288.8 J/kg K
100kPa
r=8
= 0.861m3 /kg
T3 = 2660 K
Swept volume = 0.9 V1
R 288.8
= 0.9(0.861) =Cv =
= 0.7749m3 /kg γ −1 0.4
Work done in cycle, = 722 J/kgK
γ −1
Wnet out = Q supply − Q rej T2  V1 
=   = (8)
0.4
= 1500 − 700 = 800 kJ/kg T1  V2 
Mean effective pressure
Wnet T2 = 308 × 80.4 = 707.6K
= Qs = Cv ∆T
Swept volume
= 722(2660 − 707.6)
800
Pmep. = = 1409.6 kJ/kg
0.7749
Pmep. = 1032.39 kPa Q.17 (58 to 62)

Q.14 (a)

© Copyright Reserved by Gateflix.in No part of this material should be copied or reproduced without permission
γ
 V  P2
∴  1 = = 32.42
 V2  P1
V1
=
Or =
(32.42)1/1.4
11.999 ≈ 12
V2
V V 12
rc = 3 = 1 × = 1.5
V1 V2 8 V1
= 17
V2 1  rcγ − 1 
ηd = 1 −  
Cp r γ −1  γ(rc − 1) 
= γ= 1.4
Cv 1 1.51.4 − 1 
= 1 − 0.4 
or V3 − V=
2 0.1(V1 − V2 ) 12 1.4 × 0.5 
V3 V  = 0.596 or 59.6 %
or = − 1 0.1 1 − 1
V2  V2  Q.19 (525)
V3
or = 0.1×16 + 1= 2.6
V2 Q.20 (b)
1  r −1  γ Heat applied,Q s = Cp (T3 − T2 )
ηDiesel = 1 −  γ −1
c
 Heat rejected, Q r = Cv (T4 − T1 )
r  γ(rc − 1) 
1  2.61.4 − 1 
= 1 − 0.4 
17 1.4(2.6 − 1) 
1  3.81 − 1 
= 1−  
17 0.4  1.4 × 1.6 
= 0.596 or 59.6%

Q.18 (59 to 61) Qr 1 (T4 − T1 )


η = 1− = 1−
Qs γ (T3 − T2 )

Q.21 (b)
Q.22 (73.83)

100 Ltr air contains 79 ltr of N2 and


21 ltr of O2

The combustion equation with 50%


excess air is
Given
P1= 1 bar, P2=32.42bar CH 4 + 1.5 × 2[3.762N 2 + O 2 ]
Cp → 2H 2 O + CO 2 + 3 × 3.3762N 2 + O 2
γ= = 1.4
CV
It is assumed that nitrogen is insert
V4 V1 and does not participate in the
= = 8
V3 V3 reaction. Whatever nitrogen is there
for process 1 ⟶2, in reactants, same will come out in
P1V1γ = P2 V2γ the products.

On the basis of Molar or volume

© Copyright Reserved by Gateflix.in No part of this material should be copied or reproduced without permission
=N2
3 × 3.762
×100 0.718 (T3 − 707.6 ) =
500
2 + 1 + 3 × 3.762 + 1
T3 = 1403.97 K
N 2 = 73.83%

Q.23 (1403.9)

= =
P1 0.1 MPa, T1 35o C = 308 K
V1
= n= 8
V2
Qs = 500 kJ/kg
c p = 1.005 kJ/kgK
c v = 0.718 kJ/kgK
R = 0.287 kJ/kgK
cp
γ
= = 1.399  1.40
cv
=
T3 T=
max ?
For process1 − 2

For process 1 − 2
P1 V1γ = P2 V2γ
γ
 V1 
=P2 P= 0.1  ( 8 )
1.4
1  
 V2 
P2 = 1.8379 MPa
P1 V1 PV P2 V2
and = 2 2 ⇒ T=
2 T1
T1 T2 P1 V1
1.8379 1
T2 =  308
0.1  8
T2 = 707.6 K
For process 2 → 3
Q=
s C v ( T3 − T2 =
) 500 kJ/kg

© Copyright Reserved by Gateflix.in No part of this material should be copied or reproduced without permission
7 IC ENGINE

Q.1 An industrial heat pump operates Q.4 The COP of the refrigerator is
a) 2.0 b) 2.33
between the temperatures of 27 0 C and
c) 5.0 d) 6.0
−130 C .
The rates of heat addition and heat [GATE-03]
rejection are 750 W and 1000 W,
respectively. Q.5 In the window air conditioner, the
The COP for the heat pump is expansion device used is
a) 7.5 b) 6.5 a) capillary tube
c) 4.0 d) 3.0 b) thermostatic expansion valve
[GATE-03] c) automatic expansion valve
d) float valve
Q.2 For air with a relative humidity of [GATE-04]
80%
a) the dry bulb temperature is less than Q.6 During the chemical
the wet bulb temperature dehumidification process of air
b) the dew point temperature is less a) dry bulb temperature and specific
than wet bulb temperature humidity decreases
c) the dew point and wet bulb b) dry bulb temperature increases and
temperature are equal specific humidity decreases
d) the dry bulb and dew point c) dry bulb temperature decreases and
temperature are equal specific humidity increases
d) dry bulb temperature and specific
Common Data For Q.3 and Q.4 humidity increases
A refrigerator based on ideal vapour [GATE-04]
compression cycle operates between the
temperature limits of −200 C and 400 C . Q.7 Environment friendly refrigerant
The refrigerant enters the condenser R134 is used in the new generation
as saturated vapour and leaves as domestic
saturated liquid. The enthalpy and refrigerators. Its chemical formula is
entropy a) CHClF2 b) C2Cl3F3
values for saturated liquid and vapour c) C2Cl2F4 d) C2H2F4
at these temperatures are given in the [GATE-04]
table below.
Q.8 A heat engine having an efficiency of
70% is used to drive a refrigerator
having
Q.3 If refrigerant circulation rate is
a coefficient of performance of 5. The
0.025 kg/s, the refrigeration effect is
energy absorbed from low temperature
equal to
reservoir by the refrigerator for each kJ
a) 2.1 kW b) 2.5 kW
of energy absorbed from high
c) 3.0 kW d) 4.0 kW
temperature
[GATE-03]
source by the engine is
a) 0.14 kJ b) 0.71 kJ

© Copyright Reserved by Gateflix.in No part of this material should be copied or reproduced without permission
c) 3.5 kJ d) 7.1 kJ
[GATE-04]

Q.9 Dew point temperature of air at one


atmospheric pressure (1.013 bar) is
180 C .
The air dry bulb temperature is 300 C .
The saturation pressure of water at a) 6.35 ×10−3 m3 / s b) 63.5 ×10−3 m3 / s
180 C c) 635 ×10−3 m3 / s d) 4.88 ×10−3 m3 / s
and 300 C are 0.02062 bar and 0.04241 [GATE-04]
bar respectively. The specific heat of air
and water vapour respectively are 1.005
and 1.88 kJ/kg K and the latent heat of Q.11 For a typical sample of ambient air
vaporization of water at 00 C is 2500 (at 350 C , 75% relative humidity and
kJ/kg. The specific humidity (kg/kg of standard
dry atmosphere pressure), the amount of
air) and enthalpy (kJ/kg or dry air) of moisture in kg per kg of dry air will be
this moist air respectively, are approximately
a) 0.01051, 52.64 b) 0.01291, a) 0.002 b) 0.027
63.15 c) 0.25 d) 0.75
c) 0.01481, 78.60 d) 0.01532, [GATE-05]
81.40
[GATE-04] Q.12 Water at 420 C is sprayed into a
stream of air at atmospheric pressure,
Q.10 A R-12 refrigerant reciprocating dry bulb
compressor operates between the
temperature of 400 C and a wet bulb
condensing
temperature of 300 C and evaporator temperature of 200 C . The air leaving
the
temperature of −200 C . The clearance
spray humidifier is not saturated. Which
volume ratio of the compressor is 0.03. of the following statements is true ?
Specific heat ratio of the vapour is 1.15 a) Air gets cooled and humidified
and the specific volume at the suction is b) Air gets heated and humidified
0.1089 m3/kg. Other properties at c) Air gets heated and dehumidified
various states are given in the figure. To d) Air gets cooled and dehumidified
realize 2 tons of refrigeration, the actual [GATE-05]
volume displacement rate considering
the effect of clearance is Q.13 The vapour compression
refrigeration cycle is represented as
shown in the figure
below, with state 1 being the exit of the
evaporator. The coordinate system used
in this figure is

© Copyright Reserved by Gateflix.in No part of this material should be copied or reproduced without permission
The minimum value of heat required (in
W) is

a) p-h b) T -s
c) p-s d) T –h
[GATE-05]

Q.14 Various psychometric processes a) 167 b) 100


are shown in the figure below c) 80 d) 20
[GATE-05]

Q.16 Dew point temperature is the


temperature at which condensation
begins when
the air is cooled at constant
a) volume b) entropy
c) pressure d) enthalpy
[GATE-06]
Q.17 The statements concern
psychometric chart.
1. Constant relative humidity lines are
uphill straight lines to the right
2. Constant wet bulb temperature lines
are downhill straight lines to the right
The matching pairs are 3. Constant specific volume lines are
a) P-(i), Q-(ii), R-(iii), S-(iv), T-(v) downhill straight lines to the right
b) P-(ii), Q-(i), R-(iii), S-(v), T-(iv) 4. Constant enthalpy lines are coincident
c) P-(ii), Q-(i), R-(iii), S-(iv), T-(v) with constant wet bulb temperature
d) P-(iii), Q-(iv), R-(v), S-(i), T-(ii) Lines Which of the statements are
[GATE-05] correct ?
a) 2 and 3 b) 1 and 2
Q.15 A vapour absorption refrigeration c) 1 and 3 d) 2 and 4
system is a heat pump with three [GATE-06]
thermal
reservoirs as shown in the figure. A Q.18 A building has to be maintained at
refrigeration effect of 100 W is required 210 C (dry bulb) and 14.50 C (wet bulb).
at The dew point temperature under these
250 K when the heat source available is
conditions is 10.17 0 C . The outside
at 400 K. Heat rejection occurs at 300 K.

© Copyright Reserved by Gateflix.in No part of this material should be copied or reproduced without permission
temperature is −230 C (dry bulb) and bulb temperature of 200 C is humidified
the internal and external surface heat in an air washer operating with
transfer coefficients are 8 W/m2 K and continuous water recirculation. The wet
23 W/m2 K respectively. If the building bulb depression (i.e. the difference
wall has a thermal conductivity of 1.2 between the dry and wet bulb
W/m K, the minimum thickness (in m) temperature) at the exit is 25% of that
of at the inlet. The dry bulb
the wall required to prevent temperature at the exit of the air washer
condensation is is closest to
a) 0.471 b) 0.407 a) 100 C b) 200 C
c) 0.321 d) 0.125 c) 250 C d) 300 C
[GATE-07] [GATE-08]
Q .19 Atmospheric air at a flow rate of 3 Q.22 In an ideal vapour compression
kg/s (on dry basis) enters a cooling and refrigeration cycle, the specific enthalpy
dehumidifying coil with an enthalpy of of refrigerant (in kJ/kg) at the following
85 kJ/ kg of dry air and a humidity ratio states is given as:
of 19 grams/kg of dry air. The air leaves Inlet of condenser :283
the coil with an enthalpy of 43 kJ/kg of Exit of condenser :116
dry air and a humidity ratio of 8 Exit of evaporator :232
grams/kg of dry air. If the condensate The COP of this cycle is
water a) 2.27 b) 2.75
leaves the coil with an enthalpy of 67 c) 3.27 d)3.75
kJ/kg, the required cooling capacity of
the [GATE-09]
coil in kW is
a) 75.0 b) 123.8 Q.23 A moist air sample has dry bulb
c) 128.2 d) 159.0 temperature of 300 C and specific
[GATE-07] humidity of
11.5 g water vapour per kg dry air.
Assume molecular weight of air as
Q.20 Moist air at a pressure of 100 kPa 28.93. If the
is compressed to 500 kPa and then saturation vapour pressure of water at
cooled to 350 C in an aftercooler. The air 300 C is 4.24 kPa and the total pressure
at the entry to the aftercooler is is
unsaturated and becomes just saturated 90 kPa, then the relative humidity (in
at the exit of the aftercooler. The %) of air sample is
saturation pressure of a) 50.5 b) 38.5
water at 350 C is 5.628 kPa. The partial c) 56.5 d) 68.5
pressure of water vapour (in kPa) in the [GATE-10]
moist air entering the compressor is Q.24 If a mass of moist air in an airtight
closest to vessel is heated to a higher temperature,
a) 0.57 b) 1.13 then
c) 2.26 d) 4.52 a) specific humidity of the air increases
[GATE-08] b) specific humidity of the air decreases
c) relative humidity of the air increases
Q.21 Air (at atmospheric pressure) at a d) relative humidity of the air decreases
dry bulb temperature of 400 C and wet [GATE-11]

© Copyright Reserved by Gateflix.in No part of this material should be copied or reproduced without permission
Q.25 The rate at which heat is extracted,
in kJ/s from the refrigerated space is relevant properties are given below. The
a) 28.3 b) 42.9 enthalpy (in kJ/kg) of the refrigerant at
c) 34.4 d) 14.6 isentropic compressor discharge is
_______
[GATE-12]
Q.26 The power required for the
compressor in kW is
a) 5.94 b) 1.83
c) 7.9 d) 39.5
[GATE-12]
Q.27 The pressure, dry bulb
temperature and relative humidity of air
in a room are
1 bar, 300 C and 70%, respectively. If the [GATE-
saturated pressure at 300 C is 4.25 kPa, 2014 (2)]
the specify humidity of the room air in
kg water vapour/kg dry air is Q. 31 A reversed Carnot cycle
a) 0.0083 b) 0.0101 refrigerator maintains a temperature of
c) 0.0191 d) 0.0232 − 5°C. The ambient air temperature is
35°C. The heat gained by the
[GATE-13] refrigerator at a continuous rate is 2.5
Q.28 which one of the following is a kJ/s. The power (in watt) required to
CFC refrigerant ? pump this heat out continuously is ____ .

(a) R744 (b) R290 [GATE-2014 (4)]

(C) R502 (d) R718 Q.32 Air in a room is at 35°C and 60%
relative humidity (RH). The pressure in
[GATE-2014 (1)] the room is 0.1 MPa. The saturation
Q.29 A sample of moist air at a total pressure of water at 35°C is 5.63 kPa.
pressure of 85 kPa has a dry bulb The humidity ratio of the air (in
temperature of 300 C (saturation vapour gram/kg of dry air)is________.
pressure of water = 4.24 kPa). If the air
sample has a relative humidity of 65%, [GATE-2015(3)]
the absolute humidity (in gram) to Q.33 Refrigerant vapor enters into the
water vapour per kg of dry air is_______. Compressor of a standard vapor
[GATE-2014 (3)] compression cycle at - 10oC (h = 402
kJ/kg) and leaves the compression at
Q.30 A heat pump with refrigerant R22 50°C (h = 432 kJ/kg). It leaves the
is used for space heating between condenser at 30°C (h = 237 kJ/kg). The
temperature limits of − 20°C and 25°C. COP of the Cycle is ______________
The heat required is 200 MJ/h. Assume
specific heat of vapour at the time of [GATE-2015(3)]
discharge as 0.98 kJ/kg-K. Other

© Copyright Reserved by Gateflix.in No part of this material should be copied or reproduced without permission
Q.34 The thermodynamic cycle shown kg of dry air (gw/kgda) is __________.
figure (T- diagram) indicates [GATE-2016 (3)]
Q.38 A refrigerator uses R-134 a as its
refrigerant and operates on a ideal
vapour-compression refrigeration cycle
between 0.14 MPa and 0.8 MPa. If the
mass flow rate of the refrigerant is 0.05
kg/s the rate of heat rejection to the
environment is ____kW.
Given data :
At P = 0.14 MPa, h 236.04 kJ/kg,
s = 0.9322 kJ/kgK
At P = 0.8 MPa, h = 272.05 kJ/kg
(a) reversed Carnot cycle (superheated vapour)

(b) reversed Brayton cycle At P = 0.8 MPa, h = 93.42


kJ/kg(saturated liquid)
(c) vapor compression cycle
[GATE-2016 (2)]
(d) vapor absorption cycle
[GATE-2015(3)]
Q.39 In the vapour compression cycle
Q.35 The COP of a Carnot heat pump shown in the figure, the evaporating and
operating between 6°C and 37°C condensing temperatures are 260 K and
is ______ 310K, respectively. The compressor
[GATE-2015(2)] takes in liquid-vapour mixture (state 1)
and isentropically compresses it to a dry
saturated vapour condition (state 2).
Q.36 A stream of moist air (mass flow The specific heat of the liquid
rate 10.1 kg/s) with humidity ratio of refrigerant is 4.8 kJ/kgK and may be
0.01 kg/kg dry air mixes with a second treated as constant. The enthalpy of
stream of superheated water vapour evaporation for the refrigerant at 310 K
flowing at 0.1 kg/s. Assuming proper is 1054 kJ/kg.
and Uniform mixing with no
condensation, the humidity ratio of the
final stream in (kg/kg dry air) is _______.
[GATE-2015(1)]
Q.37 In a mixture of dry air and water
vapor at a total pressure of 750 mm of
Hg, the partial pressure of water vapor
is 20 mm of Hg. The humidity ratio of
the air in grams of water vapor per

© Copyright Reserved by Gateflix.in No part of this material should be copied or reproduced without permission
The difference between the enthalpies b) relative humidity increases
at state point 1 and 0 (in kJ/kg)
c) specific humidity increases
is____________.
d) specific humidity decreases
[GATE-2016 (3)]
Q.40 The partial pressure of water
[GATE-2017 (2)]
vapour in a moist air sample of relative
humidity Q.43 Ambient air is at a pressure of 100
kPa, dry bulb temperature of 30 0 C
70% is 1.6 kPa, the total pressure being
and 60% relative humidity. The
101.325 kPa. Moist air may be treated as
saturation pressure of water at 30 0 C is
an ideal gas mixture of water vapour 4.24 kPa. The specific humidity of air (in
and dry air. The relationbetween g/kg of dry air) is ________ (correct to two
saturation temperature (Ts in K) and decimal places).
saturation pressure (Ps in kPa) for water
is given [GATE-2018 (1)]
Q.44 A standard vapor compression
 p  14.317 − 5304 refrigeration cycle operating with a
by ln  s  =
 p0  Ts condensing temperature of 35 0 C and
an evaporating temperature of -10 0 C
where p0 = 101.325 kPa. The dry bulb develops 15 kW of cooling.
temperature of the moist air sample (in The p-h diagram shows the enthalpies at
oC) is_________
various states. If the isentropic
efficiency of the compressor is 0.75, the
[GATE-2016 (2)]
magnitude of compressor power (in
Q.41 Moist air is treated as an ideal kW) is _________ (correct to two decimal
places).
gas mixture of water vapor and dry air
[GATE-2018 (2)]
(molecular weight of air = 28.84 and
molecular weight of water = 18). At a
location, the total pressure is 100 kPa,
the temperature is 30°C and the relative
humidity is 55%. Given that the
saturation pressure of water at 30°C is
4246 Pa, the mass of water vapor per kg
of dry air is ______ grams.

[GATE-2017 (1)]

Q.42 If a mass of moist air contained in


a closed metallic vessel is heated,
then its
a) relative humidity decreases

© Copyright Reserved by Gateflix.in No part of this material should be copied or reproduced without permission
ANSWER KEY:
1 2 3 4 5 6 7 8 9 10 11 12 13
(c) (b) (a) (b) (a) (b) (d) (c) (b) (a) (b) (b) (a)
14 15 16 17 18 19 20 21 22 23 24 25 26
(b) (c) (c) (a) (b) (c) (b) (c) (a) (b) (d) (a) (c)
27 28 29 30 31 32 33 34 35 36 37 38 39
(c) (b) 20.84 (b) 433.3 373.13 21.74 5.5 (b) 10 0.02 17.04 8.93

27 28 29 30 31 32

1103.51 19.89 14.9 (a) 16.236 10

© Copyright Reserved by Gateflix.in No part of this material should be copied or reproduced without permission
EXPLANATIONS

Q.1(c)

Q1 1000
So, (COP) H.P. == = 4
Q1 − Q 2 1000 − 750

Q.2 (b) We know that for saturated air,


the relative humidity is 100% and the dry
bulb temperature, wet bulb temperature
and dew point temperature is same. But
when air is not saturated, dew point
temperature is always less than the wet
bulb temperature.
DPT < WBT

Q.3(a)

© Copyright Reserved by Gateflix.in No part of this material should be copied or reproduced without permission
 Given : T1 = T4 =- 200C = (- 20 + 273)K = 253 K,

m = 0.025 kg/ sec
T2 =T3 = 400C = (40 + 273)K = 313 K
From the given table,
At, T2 = 400C, h 2 = 200 kJ/kg
And h 3 = h 4 = 80 kJ/kg
From the given T s - curve
s1 = s 2
s 2 = sf + xsfg
{s 2 is taken 0.67 because s 2 at the
temperature 400C & at 2 high temperature In the process of chemical
dehumidification of air , the air is passed
and pressure vapour refrigerant exist.}
over chemicals which have an affinity for
0.67 = 0.07 + x(0.7366 - 0.07) moisture and the moisture of air gets
x=0.90 condensed out and gives up its latent
And Enthalpy at point 1 is, heat. Due to the condensation, the
h1 = h f + xh fg specific humidity decreases and the heat
of condensation supplies sensible heat for
= 20 + 0.90(180 - 20) = 164 kJ/kg
heating the air and thus increasing its dry
Now refrigeration effect is bulb temperature.
produce in the evaporator. So chemical dehumidification increase dry
Heat extracted from the bulb temperature & decreases specific
humidity
evaporator or refrigerating effect,

RE = m (h1 - h 4 ) Q.7 (d) If a refrigerant is written in the
= 0.025(164 - 80) = 2.1 kW form of Rabc.
The first digit on the right (c) is the
number of fluorine (F) atoms, the second
Q.4 (b)
digit from the right (b) is one more than
Refrigerating effect
(COP)refrigerator = the number of hydrogen (H) atoms
Work done required & third digit from the right (a) is
h1 − h 4 164 − 80 one less than the Number of carbon
= = = 2.33
h 2 − h1 200 − 16 (C) atoms in the refrigerant. So, For R134
[GATE-03] First digit from the Right = 4 = Number of
Fluorine atoms
Q.5 (a)Air conditioner mounted in a Second digit from the right = 3 - 1 = 2 =
window or through the wall are self- Number of hydrogen atoms
contained units of small capacity of 1 TR Third digit from the right = 1 + 1 = 2 =
to 3 TR. The capillary tube is used as an Number of carbon atoms
expansion device in small capacity Hence, Chemical formula is C2H2F4
refrigeration units

Q.6 (b)

© Copyright Reserved by Gateflix.in No part of this material should be copied or reproduced without permission
Q.8 (c) Enthalpy,
h = 1.022 Tdb + ω ( h fgdp + 2.3 tdp )
KJ
h = 63.15
kg

Q.10 (a)
Q.11 (b)
From steam table, saturated air pressure
corresponding to dry bulb temperature
( COP=
)ref 5, ( η=
)H.E 70%
= 0.7 of 350C is ps = 0.05628 bar.
Q3 Relative humidity
( COP )=
ref
= 5........(i ) P
W Relative humidity φ = v
W Pvs
( η )H.E
= = 0.7..........(ii )
Pv
Q1
0.75 = ⇒ Pv = 0.04221 bar
Multiplying ( i ) and ( ii ) Pvs
pv
Q3 W
= 5 × 0.7 Specific humidity ω = 0.622
W Q1 pb − pv
 0.04221 
Q3
= 3.5 ω = 0.622  
Q1  1.01 − 0.04221 
Hence, Energy absorbed (Q3) = 0.0271 kg/kg of dry air
from low temperature reservoir
by the refrigerator for each kJ of Q.12 (b)Given :
energy absorbed (Q1) from high Hence air gets heated, Also water is added to
temperature source by the it, so it gets humidified.
engine= 3.5 kJ = =
t sp 42 0
=
C, t db 40 0
C, t wb 200 C
Here we see that t sp > t db
Q.9(b)
Given : tdp = 180C
= (273 + 18)K Q.13 (a)Given curve is the theoretical p-h
= 291K, curve for vapour compression
p = patm = 1.013 bar refrigeration cycle.
tdb = 300C = (273 + 30)K = 303 K
pv = 0.02062 bar (for water Q.14(b)
vapour at dew point).
cair = 1.005 kJ/kg K, Q.15(c)
cwater = 1.88 kJ/kg K
Latent heat of vaporization of Q.16(c)
water at 00C.
hfgdp = 2500 kJ/kg
pv
=
Specific humidity, W 0.622 ×
p − pv
kg
W = 0.01291
kg of dry air

© Copyright Reserved by Gateflix.in No part of this material should be copied or reproduced without permission
limiting condition to prevent
condensation
Ts1=10.17 0C
Here Ts1 & Ts2 are internal &
external wall surface
temperature of building.
Hence, heat flux per unit area
inside the building,
= h1 (TDBT 1 − Ts1 )
Q
It is the temperature of air recorded by a q=
…….(i)
i
thermometer, when the moisture (water A
vapour) present in it begins to condense. qi = 86.64 W/m 2

If a sample of unsaturated air, containing


superheated water vapour, is cooled at Heat flux per unit area outside
constant pressure, the partial pressure the building is
(pv) of each constituent remains constant q0 = h2(Ts2- TDBT2)
until the water vapour reaches the = 23(Ts2+ 23) …....(ii)
saturated state as shown by point B. At Heat flow will be same at inside
this & outside the building. So from
point B the first drop of dew will be equation (i) & (ii)
qi = q0
formed and hence the temperature at
Ts2 = -19.23 0C
point
For minimum thickness of the
B is called dew point temperature
wall, use the Fourier’s law of
conduction for the building. Heat
Q.17(a) flux through wall,
k (Ts1 − Ts 2 )
Q.18 (b) q=
x
x = 0.407 m

Q.19 (c)
Given : ma = 3 kg/sec,
Using subscript 1 and 2 for the
inlet and outlet of the coil
Let h1 & h2 be the internal and respectively.
external surface heat transfer h1 = 85 kJ/kg of dry air,
coefficients respectively and W1 = 19 grams/kg of dry air
building wall has thermal =19 × 10-3 kg/kg of dry air
conductivity k . h2 = 43 kJ/kg of dry air,
Given : h1= 8 W/m K W2 = 8 grams/kg of dry air
h2 =23W/m2 K, = 8 × 10-3 kg/kg of dry air
k=1.2 W/m K, h3 = 67 kJ/kg
TDPT=10.17 0C Mass flow rate of water vapour at
Now to prevent condensation, the inlet of the coil is,
temperature of inner wall should mv1 = W1 × m1 = 57 × 10-8 kg/sec
be more than or equal to the dew And mass flow rate of water
point temperature. It is the vapour at the outlet of coil is,
mv2 = W2 × ma

© Copyright Reserved by Gateflix.in No part of this material should be copied or reproduced without permission
mv2 =24 × 10-8 kg/sec tWBT(inlet) = tWBT(outlet)
So, mass of water vapour So, tDBT(exit)- 20 = 0.25*20
condensed in the coil is, tDBT(exit) = 20 + 5 = 25 0C
Therefore, required cooling
capacity of the coil = change in
enthalpy of dry air + Q.22 (a)
change in enthalpy of condensed p-h curve for vapour
water compression refrigeration cycle
= (85 - 43) × 3 + 67 × 33 × 10-3 is as follows
= 128.211 kW

Q.20 (b) Given : p1 = 100 kPa,


p2 = 500 kPa,
pv1 = ?
pv2 = 5.628 kPa
(Saturated pressure at 350C)
We know that,
pv
=
Specific humidity, W 0.622 ×
p − pv The given specific enthalpies are
Inlet of condenser h2 = 283 kJ/kg
For CASE-II
kJ
 0.628  Exit of condenser h3 = 116
W = 0.622  
kg
 500 − 5.628  =h4
W 7.08 ×10−8 kg / kg of dry air
= (from p-h curve)
For saturated air specific Exit of evaporator h1 = 232 kJ/kg
humidity remains same. So, for Now,
case (I) : COP=
Refrigeration Effect
pv1 Work Done
=
W 0.622 ×
p1 − pv1 h1 − h4
= = 2.27
On substituting the values, we h2 − h1
get
pv1 = 1.13 kPa
Q.23 (b)
Given : tDBT = 30 0C,
W = 11.5 g water vapour/kg dry
Q.21 (C) air
Given: At inlet tDBT =40 0C ps = 4.24 kPa, p = 90 kPa
tWBT=20 0C
We know that,
Wet bulb depression = tDBT -tWBT
= 40 - 20 = 20 0C
And given wet bulb depression at
the exit = 25% of wet bulb
depression at inlet
This process becomes adiabatic
saturation and for this process,

© Copyright Reserved by Gateflix.in No part of this material should be copied or reproduced without permission
pv mass flow rate of the refrigerant is 0.2
=
Specific humidity, W 0.622 × kg/s. Properties for R134a are as follows :
p − pv
 pv 
11.5 ×10−8 =0.622  
 90 − pv 
pv = 1.634 kPa
pv
Relative humidity, (φ ) =
ps
1.634
=
4.24
Q.25 (a)
=φ 0.385
= 38.5%
Q.4 (d) T-S
From the given curve, we easily see that
relative humidity of air decreases, when
temperature of moist air in an airtight
vessel increases.
So, option (d) is correct.
Specific humidity remain constant with
temperature increase, so option a & b
are incorrect

T-S Diagram for given Refrigeration


cycle is given above
Since Heat is Extracted in Evaporation
process.
So rate of heat extracted = m ( h1 − h4 )
From above diagram (h3 = h4) for
throttling process, so
Heat extracted = m (h1- h3)
From given table
h1 = hg at 120 kPa, hg = 237 kJ/kg
h3 = hf at 120 kPa, hf = 95.5 kJ/kg
Hence Heat extracted = m (hg - hf)
= 0.2(237 - 95.5)
= 28.3 kJ/s
A refrigerator operates between 120 kPa
Q.26 (c)
and 800 kPa in an ideal vapour
Since power is required for compressor
compression cycle with R-134a as the
in refrigeration is in compression cycle
refrigerant. The refrigerant enters the (1-2)
compressor as saturated vapour and Hence, Power required = h2-h1
leaves the condenser as saturated liquid. = h2-hf
The Since for isentropic compression
process.

© Copyright Reserved by Gateflix.in No part of this material should be copied or reproduced without permission
s1 = s2 from figure. = 0.95 Total pressure : p = 85 kPa
For entropy s = 0.95 the enthalpy Dry bulb temperature,
h = 276.45 kJ/kg
h = h2 = 276.45 (From table) Tdb = 30o C
Hence Power = 0.2(276.45 - 237) ps = 4.24 kPa
= 7.89 = 7.9 kW =φ 65= % 0.65
Q.27 (c)
Relative humidity,
Specific Humidity is given by pv
φ =
pv ps
=ω 0.622 × …...(i)
pa − pv pv
0.65 =
4.24
Where,
= =
or pv 0.65  4.24 2.756 kPa
 Relative   Saturated 
=pv    ×  
 humidity   steam pressure  Absolute humidity,
=φ × ps =0.7 × 0.0425 0.622 pv
ω=
= 0.02975 bar p − pv
= 0.02084 kg of w.v/kg of dry air
So that from equation (i), we have
= 20.84 gram of w.v./kg of dry air
0.02975
ω=
0.622 ×  Pa =
1 bar  ω2 < ω1
1 − 0.02975  Q Dehumidifier
= 0.0191 kg/kg of dry air Q.30
Q.28

R744-CO 2
R290-C3 H 3 ( Propane )
R502-CHCIF3 + CCIF2 CF3
R718-Water

Q.29
T2
s2 − s2' =
c p log e
T2'
s1 = s2 ( for isentropic process )
s2 = 1.7841 kJ/kgK
s 2 = 1.7183 kJ/kgK
c p = 0.98 kJ/kgK
T2 s2 − s2'
∴ log e =
T2' cp
T 1.7841 − 1.7183
=
log e 2 = 0.0671
T2' 0.98

© Copyright Reserved by Gateflix.in No part of this material should be copied or reproduced without permission
= =
or T2 1.0694  T2' 1.0694  298 Given data :
= 318.695 K Tdb = 35°C
h 2 − h 2=
' c p ( T2 − T2' ) φ = 60 % = O.60
413.02 + 0.98 ( 318.695 − 298 )
h2 =
p = 0.1 MPa = 100 kPa
h 2 = 433.3 kJ/kg
ps = 5.63 kPa at 35°C
Q.31 Relative humidity,
Given data :
pv
φ=
T2 =− 5°C = ( −5 + 273) K = 268 K ps
T1 = 35°C = ( 35 + 273) K = 308 K pv
0.60
5.63
= =
pv 0.60  5.63 3.378 kPa
Humidiy ratio,
0.622 pv 0.622  3.378
ω= =
p − pv 100 − 3.378

= 0.02174 kg of w.v./kg of dry air

= 21.74 gram of w. v./kg of dry air

Q.33

Given data :
= =
h1 402 kJ/kg, h 2 432 kJ/kg
h=
3 h=
4 237 kJ/kg

Q 2 = 2.5 kJ/s = 2.5 kW = 2500 W

T2
( COP )R =
T1 − T2
268
( COP )R
= = 6.7
308 − 268
Q
also ( COP ) = 2
W
h − h4 402 − 237 165
2500 COP =
= 1 =
∴ 6.7 = h2 − h1 432 − 402 30
W
2500 = 5.5
or W= = 373.13 watt
6.7 Q.34 (b)
Q.32 Reversed Brayton cycle is shown in the figure

© Copyright Reserved by Gateflix.in No part of this material should be copied or reproduced without permission
Mass of moist air = Mass of dry air + Mass of
water vapour

= m a + 0.01  m a
10.1
10.1 m a = 10.1

10.1
Mass of dry air m a = = 10 kg/s
1.01
Mass of water vapour,

m v1 = 10.1 − 10 = 0.1 kg/s


m v2 = 0.1 kg/s
Q.35

Given data :
(m )v total = m v1 + m v2 = 0.2 kg/s

=T2 6o C = ( 6 + 273) K= 279 K Humidity ratio ωfinal =


m v 0.2
=
ma 10
=T1 37 o C = ( 37 + 273) K= 310 K
= 0.02 kg/kg of dry air

Q.37

Given data :

Total pressure,

p = 750mm of Hg

Partial pressure, V

Pv = 20mm of Hg

We know that humidity ratio,

0.622 pv
ω= kg. w.v./kg d.a.
p − pv
0.622  20
=
750 − 20
= 0.01704 kg w.v./kg of d.a.
T1 310 = 17.04 g of w.v./kg of d.a.
( COP
= )HP =
T1 − T2 310 − 279
= 10
Q.38

Given data:
Q.36
m = 0.05kg/s
Mass flow rate of moist air = 10.1 kg/s
Pe = 0.14 MPa
mv h1 = 236.04 kJ/kg
Humidity ratio : =
ω = 0.01 kg/s
ma

© Copyright Reserved by Gateflix.in No part of this material should be copied or reproduced without permission
S1 = 0.9322 kJ/kgK T3
∴ s3 − s T =
c ln
p = 0.8 MPa T
T3
h3 = h4 = hf = 93.42 kJ/kg s=
3 s T + c ln
T
310
= s T + 4.8 ln
T
Similarly,
260
s=
0 s T + c ln
T
260
= s T + 4.8 ln
T
h fg 1054
∴ s 2 − s3 = =
h2 = 272.02 kJ/kg T 310
Rate of heat rejection to the environment,
1054
s=
2 s3 +
310
Q 2-3= m  h2 − h3 = 0.05  [ 275.02 − 93.42]Substitute the value of s3
= 8.93 kW in above equation, we get
s 2 = s1
Q.39
310 1054
Given data : s1 =
s T + 4.8 ln +
T 310
Te = 260K ∴ h1 − h=
0 Te  s1 − s0 
TC = 310K  310 1054 260 
= 260   sT + 4.8 ln + − sT − 4.8ln
cpl = 4.8 kJ/kgK  T 310 T 
  310 T  1054 
hfg = 1054 kJ/kg at TC = 310 K = 260  4.8ln   +
  T 260  310 
= 1103.51 kJ/kg

Q.40

Given data :

Relative humidity,

φ = 70 % = 0.70
Partial pressure,
Taking reference temperature T, at which
entropy pv = 1.6 kPa

is ST. Total pressure,

p0 = 101.325 kPa

We know that,

© Copyright Reserved by Gateflix.in No part of this material should be copied or reproduced without permission
pv Kg of water vapour
φ= ω =0.0149
ps Kg of dry air
mv
0.70 =
1.6 Qω=
ps ma

or =
ps
1.6
= 2.2857 kPa m v = ωm a
0.70
= 0.0149 ×103 ×1
Temperature corresponding to saturation
pressure m v = 14.9grm
is dry bulb temperature, Q.42 (a)

 ps  5304 For closed metallic vessel sp. Humidity


ln =
  14.317 − (ω) remain const
 po  Ts
 2.2857  5304
ln  =  14.317 −
101.325  Ts
5304
− 3.79166 =14.317 −
Ts
5304
= 14.317 + 3.79166
Ts
5304
= 18.10866
Ts
or Ts = 292.89 K
From the figure
= ( 292.89 − 273) o C
φ2 < φ1
= 19.89 o C
Q.43
Pv
Relative humidity φ =
Q.41 Ps
=
But, φ 60
= % 0.6=
, Ps 4.24 kPa
P
Relative humidity φ = v pa φ  =
⇒= ps 0.6  4.24
= 2.544 kPa
Pvs
pv
Specific humidity ω = 0.622
Pv pb − pv
0.55 =
4.246 where p b = ambient air pressure = 100 kPa
Pv = 2.335  2.544 
⇒ ω =0.622   
 100 − 2.544 
 M H2O  Pv  ⇒ ω = 0.016236kg/kg of dry air
ω =  
 M air   Pt − Pv  ⇒ ω = 16.236 g/kg of dry air

18  2.335 
ω=  
28.84  100 − 2.335 

© Copyright Reserved by Gateflix.in No part of this material should be copied or reproduced without permission
Q.44 (10)
RC = 15 kW

(
RC = m  ( h1 − h4 )
.
)
RC = m ( 400 − 250 )
.

.
m   kg/sec
Wisentropic =( h2 − h1 ) = ( 475 − 400 ) = 75 kJ/kg
Wisentropic
ηC =
Wactual
.
= =
Wactual m  Wactual 0.1  100
Pm. = 10 kW

© Copyright Reserved by Gateflix.in No part of this material should be copied or reproduced without permission

You might also like